You are on page 1of 182

Prefa

e
This book is a ontinuation Mathemati al Olympiads 1995-1996: Olympiad
Problems from Around the World, published by the Ameri an Mathemati s Competitions. It ontains solutions to the problems from 25 national
and regional ontests featured in the earlier pamphlet, together with sele ted problems (without solutions) from national and regional ontests
given during 1997.
This olle tion is intended as pra ti e for the serious student who
wishes to improve his or her performan e on the USAMO. Some of the
problems are omparable to the USAMO in that they ame from national ontests. Others are harder, as some ountries rst have a national
olympiad, and later one or more exams to sele t a team for the IMO. And
some problems ome from regional international ontests (\mini-IMOs").
Di erent nations have di erent mathemati al ultures, so you will nd
some of these problems extremely hard and some rather easy. We have
tried to present a wide variety of problems, espe ially from those ountries
that have often done well at the IMO.
Ea h ontest has its own time limit. We have not furnished this information, be ause we have not always in luded omplete ontests. As a
rule of thumb, most ontests allow a time limit ranging between one-half
to one full hour per problem.
Thanks to Walter Mientka for his ontinuing support of this proje t,
and to the students of the 1997 Mathemati al Olympiad Summer Program
for their help in preparing solutions.
The problems in this publi ation are opyrighted. Requests for reprodu tion permissions should be dire ted to:
Dr. Walter Mientka
Se retary, IMO Advisory Broad
1740 Vine Street
Lin oln, NE 68588-0658, USA.

Contents
1 1996 National Contests:
Problems and Solutions

1.1
1.2
1.3
1.4
1.5
1.6
1.7
1.8
1.9
1.10
1.11
1.12
1.13
1.14
1.15
1.16
1.17
1.18
1.19

Bulgaria . . . . . . . . . . .
Canada . . . . . . . . . . .
China . . . . . . . . . . . .
Cze h and Slovak Republi s
Fran e . . . . . . . . . . . .
Germany . . . . . . . . . .
Gree e . . . . . . . . . . . .
Iran . . . . . . . . . . . . .
Ireland . . . . . . . . . . . .
Italy . . . . . . . . . . . . .
Japan . . . . . . . . . . . .
Poland . . . . . . . . . . . .
Romania . . . . . . . . . . .
Russia . . . . . . . . . . . .
Spain . . . . . . . . . . . .
Turkey . . . . . . . . . . . .
United Kingdom . . . . . .
United States of Ameri a .
Vietnam . . . . . . . . . . .

.
.
.
.
.
.
.
.
.
.
.
.
.
.
.
.
.
.
.

.
.
.
.
.
.
.
.
.
.
.
.
.
.
.
.
.
.
.

.
.
.
.
.
.
.
.
.
.
.
.
.
.
.
.
.
.
.

.
.
.
.
.
.
.
.
.
.
.
.
.
.
.
.
.
.
.

.
.
.
.
.
.
.
.
.
.
.
.
.
.
.
.
.
.
.

.
.
.
.
.
.
.
.
.
.
.
.
.
.
.
.
.
.
.

.
.
.
.
.
.
.
.
.
.
.
.
.
.
.
.
.
.
.

.
.
.
.
.
.
.
.
.
.
.
.
.
.
.
.
.
.
.

.
.
.
.
.
.
.
.
.
.
.
.
.
.
.
.
.
.
.

.
.
.
.
.
.
.
.
.
.
.
.
.
.
.
.
.
.
.

.
.
.
.
.
.
.
.
.
.
.
.
.
.
.
.
.
.
.

.
.
.
.
.
.
.
.
.
.
.
.
.
.
.
.
.
.
.

.
.
.
.
.
.
.
.
.
.
.
.
.
.
.
.
.
.
.

.
.
.
.
.
.
.
.
.
.
.
.
.
.
.
.
.
.
.

.
.
.
.
.
.
.
.
.
.
.
.
.
.
.
.
.
.
.

.
.
.
.
.
.
.
.
.
.
.
.
.
.
.
.
.
.
.

.
.
.
.
.
.
.
.
.
.
.
.
.
.
.
.
.
.
.

.
.
.
.
.
.
.
.
.
.
.
.
.
.
.
.
.
.
.

Asian Pa i Mathemati s Olympiad . . . . .


Austrian-Polish Mathemati s Competition . .
Balkan Mathemati al Olympiad . . . . . . . .
Cze h-Slovak Mat h . . . . . . . . . . . . . .
Iberoameri an Olympiad . . . . . . . . . . . .
St. Petersburg City Mathemati al Olympiad

.
.
.
.
.
.

.
.
.
.
.
.

.
.
.
.
.
.

.
.
.
.
.
.

.
.
.
.
.
.

.
.
.
.
.
.

.
.
.
.
.
.

.
.
.
.
.
.

.
.
.
.

.
.
.
.

.
.
.
.

.
.
.
.

.
.
.
.

.
.
.
.

.
.
.
.

.
.
.
.

2 1996 Regional Contests:


Problems and Solutions

2.1
2.2
2.3
2.4
2.5
2.6

3 1997 National Contests:


Problems

3.1
3.2
3.3
3.4

Austria
Bulgaria
Canada
China .

.
.
.
.

.
.
.
.

.
.
.
.

.
.
.
.

.
.
.
.

.
.
.
.

.
.
.
.

.
.
.
.

.
.
.
.

.
.
.
.

.
.
.
.
1

.
.
.
.

.
.
.
.

.
.
.
.

.
.
.
.

.
.
.
.

.
.
.
.

.
.
.
.

.
.
.
.

.
.
.
.

.
.
.
.

3
9
12
17
22
25
27
29
34
38
41
44
47
57
76
81
84
89
96
100

100
103
108
110
114
118

131

131
132
136
137

3.5
3.6
3.7
3.8
3.9
3.10
3.11
3.12
3.13
3.14
3.15
3.16
3.17
3.18
3.19
3.20
3.21
3.22
3.23
3.24
3.25
3.26

Colombia . . . . . . . . . .
Cze h and Slovak Republi s
Fran e . . . . . . . . . . . .
Germany . . . . . . . . . .
Gree e . . . . . . . . . . . .
Hungary . . . . . . . . . . .
Iran . . . . . . . . . . . . .
Ireland . . . . . . . . . . . .
Italy . . . . . . . . . . . . .
Japan . . . . . . . . . . . .
Korea . . . . . . . . . . . .
Poland . . . . . . . . . . . .
Romania . . . . . . . . . . .
Russia . . . . . . . . . . . .
South Afri a . . . . . . . .
Spain . . . . . . . . . . . .
Taiwan . . . . . . . . . . . .
Turkey . . . . . . . . . . . .
Ukraine . . . . . . . . . . .
United Kingdom . . . . . .
United States of Ameri a .
Vietnam . . . . . . . . . . .

.
.
.
.
.
.
.
.
.
.
.
.
.
.
.
.
.
.
.
.
.
.

.
.
.
.
.
.
.
.
.
.
.
.
.
.
.
.
.
.
.
.
.
.

.
.
.
.
.
.
.
.
.
.
.
.
.
.
.
.
.
.
.
.
.
.

.
.
.
.
.
.
.
.
.
.
.
.
.
.
.
.
.
.
.
.
.
.

Asian Pa i Mathemati s Olympiad . . . . . . . . . .


Austrian-Polish Mathemati al Competition . . . . . .
Cze h-Slovak Mat h . . . . . . . . . . . . . . . . . . .
Hungary-Israel Mathemati s Competition . . . . . . .
Iberoameri an Mathemati al Olympiad . . . . . . . .
Nordi Mathemati al Contest . . . . . . . . . . . . . .
Rio Plata Mathemati al Olympiad . . . . . . . . . . .
St. Petersburg City Mathemati al Olympiad (Russia)

.
.
.
.
.
.
.
.

.
.
.
.
.
.
.
.

.
.
.
.
.
.
.
.

4 1997 Regional Contests:


Problems

4.1
4.2
4.3
4.4
4.5
4.6
4.7
4.8

.
.
.
.
.
.
.
.
.
.
.
.
.
.
.
.
.
.
.
.
.
.

.
.
.
.
.
.
.
.
.
.
.
.
.
.
.
.
.
.
.
.
.
.

.
.
.
.
.
.
.
.
.
.
.
.
.
.
.
.
.
.
.
.
.
.

.
.
.
.
.
.
.
.
.
.
.
.
.
.
.
.
.
.
.
.
.
.

.
.
.
.
.
.
.
.
.
.
.
.
.
.
.
.
.
.
.
.
.
.

.
.
.
.
.
.
.
.
.
.
.
.
.
.
.
.
.
.
.
.
.
.

.
.
.
.
.
.
.
.
.
.
.
.
.
.
.
.
.
.
.
.
.
.

.
.
.
.
.
.
.
.
.
.
.
.
.
.
.
.
.
.
.
.
.
.

.
.
.
.
.
.
.
.
.
.
.
.
.
.
.
.
.
.
.
.
.
.

.
.
.
.
.
.
.
.
.
.
.
.
.
.
.
.
.
.
.
.
.
.

.
.
.
.
.
.
.
.
.
.
.
.
.
.
.
.
.
.
.
.
.
.

.
.
.
.
.
.
.
.
.
.
.
.
.
.
.
.
.
.
.
.
.
.

.
.
.
.
.
.
.
.
.
.
.
.
.
.
.
.
.
.
.
.
.
.

.
.
.
.
.
.
.
.
.
.
.
.
.
.
.
.
.
.
.
.
.
.

139
140
141
142
144
145
146
147
148
149
150
152
153
155
161
162
163
165
166
167
168
169
170

170
171
173
174
175
177
178
179

1996 National Contests:


Problems and Solutions

1.1 Bulgaria

1. Prove that for all natural numbers n  3 there exist odd natural
numbers xn ; yn su h that 7x2n + yn2 = 2n .
Solution: For n = 3 we have x3 = y3 = 1. Now suppose that
for a given natural number n we have odd natural numbers xn ; yn
su h that 7x2n + yn2 = 2n; we shall exhibit a pair (X; Y ) su h that
7X 2 + Y 2 = 2n+1. In fa t,

xn  yn
2

2

7xn  yn
2

2

= 2(7x2n + yn2 ) = 2n+1 :

One of (xn + yn)=2 and jxn ynj=2 is odd (as their sum is the larger
of xn and yn , whi h is odd), giving the desired pair.
2. The ir les k1 and k2 with respe tive enters O1 and O2 are externally tangent at the point C , while the ir le k with enter O is
externally tangent to k1 and k2 . Let ` be the ommon tangent of k1
and k2 at the point C and let AB be the diameter of k perpendi ular
to `. Assume that O and A lie on the same side of `. Show that the
lines AO2 ; BO1 ; ` have a ommon point.
Solution: Let r; r1 ; r2 be the respe tive radii of k; k1 ; k2 . Also let

M and N be the interse tions of AC and BC with k. Sin e AMB


is a right triangle, the triangle AMO is isos eles and

\AMO = \OAM = \O1 CM = \CMO1 :


Therefore O; M; O1 are ollinear and AM=MC = OM=MO1 = r=r1 .
Similarly O; N; O2 are ollinear and BN=NC = ON=NO2 = r=r2 .
Let P be the interse tion of ` with AB ; the lines AN; BM; CP on ur at the ortho enter of ABC , so by Ceva's theorem, AP=P B =
(AM=MC )(CN=NB ) = r2 =r1 . Now let D1 and D2 be the interse tions of ` with BO1 and AO2 . Then CD1 =D1 P = O1 C=P B =
r1 =P B , and similarly CD2 =D2P = r2 =P A. Thus CD1 =D1P =
CD2 =D2P and D1 = D2 , and so AO2 ; BO1 ; ` have a ommon point.
3

3. Let a; b; be real numbers and let M be the maximum of the fun tion
y = j4x3 + ax2 + bx + j in the interval [ 1; 1. Show that M  1
and nd all ases where equality o urs.
Solution: For a = 0; b = 3; = 0, we have M = 1, with the
maximum a hieved at 1; 1=2; 1=2; 1. On the other hand, if M < 1
for some hoi e of a; b; , then

(4x3 + ax2 + bx + ) (4x3 + 3x)


must be positive at 1, negative at 1=2, positive at 1=2, and
negative at 1, whi h is impossible for a quadrati fun tion. Thus
M  1, and the same argument shows that equality only o urs for
(a; b; ) = (0; 3; 0). (Note: this is a parti ular ase of the minimum
deviation property of Chebyshev polynomials.)

4. The real numbers a1 ; a2 ; : : : ; an (n  3) form an arithmeti progression. There exists a permutation ai1 ; ai2 ; : : : ; ain of a1 ; a2 ; : : : ; an
whi h is a geometri progression. Find the numbers a1 ; a2 ; : : : ; an if
they are all di erent and the largest of them is equal to 1996.
Solution: Let a1 < a2 <    < an = 1996 and let q be the ratio of

the geometri progression ai1 ; : : : ain ; learly q 6= 0; 1. By reversing


the geometri progression if needed, we may assume jqj > 1, and so
jai1 j < jai2 j <    < jain j. Note that either all of the terms are
positive, or they alternate in sign; in the latter ase, the terms of
either sign form a geometri progression by themselves.
There annot be three positive terms, or else we would have a threeterm geometri progression a; b; whi h is also an arithmeti progression, violating the AM-GM inequality. Similarly, there annot
be three negative terms, so there are at most two terms of ea h sign
and n  4.
If n = 4, we have a1 < a2 < 0 < a3 < a4 and 2a2 = a1 + a3 ,
2a3 = a2 + a4 . In this ase, q < 1 and the geometri progression is
either a3 ; a2 ; a4 ; a1 or a2 ; a3 ; a1 ; a4 . Suppose the former o urs (the
argument is similar in the latter ase); then 2a3q = a3 q3 + a3 and
2a3 + a3 q + a3 q2 , giving q = 1, a ontradi tion.
We dedu e n = 3 and onsider two possibilities. If a1 < a2 <
0 < a3 = 1996, then 2a2 = a2 q2 + a2 q, so q2 + q 2 = 0 and
4

q = 2, yielding (a1 ; a2 ; a3 ) = ( 3992; 998; 1996). If a1 < 0 <


a2 < a3 = 1996, then 2a2 = a2 q + a2 q2 , so again q = 2, yielding
(a1 ; a2 ; a3 ) = ( 998; 499; 1996).
5. A onvex quadrilateral ABC is given for whi h \ABC + \BCD <
180. The ommon point of the lines AB and CD is E . Prove that
\ABC = \ADC if and only if

AC 2 = CD  CE

AB  AE:

Solution: Let C1 be the ir um ir le of ADE , and let F be its


se ond interse tion with CA. In terms of dire ted lengths, we have
AC 2 = CD  CE + AB  AE if and only if

AB  AE = AC 2

CD  CE = CA2

CA  AF = AC  AF;

that is, if and only if B; C; E; F are on y li . But this happens if


and only if \EBC = \EF C , and

\EF C = \EF A =  \ADE = \CDA


(in dire ted angles modulo ), so B; C; E; F are on y li if and only
if \ABC = \ADC (as undire ted angles), as desired.
6. Find all prime numbers p; q for whi h pq divides (5p

2p )(5q

2q ).

Solution: If pj5p 2p , then pj5 2 by Fermat's theorem, so p = 3.

Suppose p; q 6= 3; then pj5q 2q and qj5p 2p . Without loss of


generality, assume p > q, so that (p; q 1) = 1. Then if a is an
integer su h that 2a  5 (mod q), then the order of a mod q divides
p as well as q 1, a ontradi tion.
Hen e one of p; q is equal to 3. If q 6= 3, then qj53 23 = 9  13, so
q = 13, and similarly p 2 f3; 13g. Thus the solutions are (p; q) =
(3; 3); (3; 13); (13; 3):
7. Find the side length of the smallest equilateral triangle in whi h
three dis s of radii 2; 3; 4 an be pla ed without overlap.
Solution: A short omputation shows that dis s of radii 3 p
and 4

an be t into two orners of an equilateral triangle of side 11 3 so


5

as to just tou h, and that a dis of radius 2 easily ts into the third
orner without overlap. On the other hand, if the dis s of radii 3
and 4 t into an equilateral triangle without overlap, there exists a
line separating them (e.g. a tangent to one perpendi ular to their
line of enters) dividing the triangle into a triangle and a (possibly
degenerate) onvex quadrilateral. Within ea h pie e, the dis an be
moved into one of the orners of the original triangle. Thus the two
dis s t into the orners without
p overlap, so the side length of the
triangle must be at least 11 3.
8. The quadrati polynomials f and g with real oe ients are su h
that if g(x) is an integer for some x > 0, then so is f (x). Prove that
there exist integers m; n su h that f (x) = mg(x) + n for all x.
Let f (x) = ax2 + bx + and g(x) = px2 + qx + r;
assume without loss of generality p > 0 and q = 0 (by the hange
of variable
px ! x q=(2p)). Let k be an integer su h that k > s
and t = (k s)=p > q=(2p). Sin e g(t) = k is an integer, so is
f (t) = a(k s)=p + bt + , as is

Solution:

k+1 s
p

b
1
k s
=p p
p
p k+1 s

a
+ :
s p

This tends to a=p as k in reases, so a=p must be an integer; moreover,


b must equal 0, or else the above expression will equal a=p plus a
small quantity for large k, whi h annot be an integer. Now put
m = a=p and n = ms; then f (x) = mg(x) + n.
9. The sequen e fan g1
n=1 is de ned by
n
a
a1 = 1; an+1 = n + ; n  1:
n an
Prove that for n  4, ba2n = n.
Solution: We will show by indu tion that

pn  a  n=pn
n

1
for n  1, whi h will imply the laim. These inequalities learly
hold for n = 1; 2; 3. Now assume the inequality for some n. Let
fn(x) = x=n + n=x. We rst have for n  3,


p
n
n
=p
> n + 1:
an+1 = fn (an )  fn p
n 1
n 1
6

On the other hand, using that an > (n 1)= n 2 (whi h we just


proved), we get for n  4,

n 1
(n 1)2 + n2 (n 2) p
p
an+1 = fn (an ) < fn p
=
< n + 2:
n 2
(n 1)n n 2


10. The quadrilateral ABCD is ins ribed in a ir le. The lines AB


and CD meet at E , while the diagonals AC and BD meet at F .
The ir um ir les of the triangles AF D and BF C meet again at H .
Prove that \EHF = 90.
(We use dire ted angles modulo .) Let O be the
ir um enter of ABCD; then

Solution:

\AHB = \AHF +\F HB = \ADF +\F CB = 2\ADB = \AOB;


so O lies on the ir um ir le of AHB , and similarly on the ir um ir le of CHD. The radi al axes of the ir um ir les of AHB; CHD
and ABCD on ur; these lines are AB , CD and HO, so E; H; O are
ollinear. Now note that
\OHF = \OHC +\CHF = \ODC +\CBF = 2 \CAD+\CBD;
so \EHF = \OHF = =2 as desired. (Compare IMO 1985/5.)
11. A 7  7 hessboard is given with its four orners deleted.
(a) What is the smallest number of squares whi h an be olored
bla k so that an un olored 5-square (Greek) ross annot be
found?
(b) Prove that an integer an be written in ea h square su h that
the sum of the integers in ea h 5-square ross is negative while
the sum of the numbers in all squares of the board is positive.
Solution:

(a) The 7 squares


(2; 5); (3; 2); (3; 3); (4; 6); (5; 4); (6; 2); (6; 5)
7

su e, so we need only show that 6 or fewer will not su e.


The rosses entered at
(2; 2); (2; 6); (3; 4); (5; 2); (5; 6); (6; 4)
are disjoint, so one square must be olored in ea h, hen e 5
or fewer squares do not su e. Suppose exa tly 6 squares are
olored. Then none of the squares (1; 3); (1; 4); (7; 2) an be olored; by a series of similar arguments, no square on the perimeter an be olored. Similarly, (4; 3) and (4; 5) are not overed,
and by a similar argument, neither is (3; 4) or (5; 4). Thus the
enter square (4; 4) must be overed.
Now the rosses entered at
(2; 6); (3; 3); (5; 2); (5; 6); (6; 4)
are disjoint and none ontains the enter square, so ea h ontains one olored square. In parti ular, (2; 2) and (2; 4) are not
olored. Repla ing (3; 3) with (2; 3) in the list shows that (3; 2)
and (3; 4) are not olored. Similar symmetri arguments now
show that no squares besides the enter square an be overed,
a ontradi tion. Thus 7 squares are needed.
(b) Write 5 in the 7 squares listed above and 1 in the remaining
squares. Then learly ea h ross has negative sum, but the total
of all of the numbers is 5( 7) + (45 7) = 3.

1.2 Canada
1. If ; ; are the roots of x3

x 1 = 0, ompute

1 1 1
+
+
:
1+ 1+ 1+
Solution: The given quantity equals

1
1
1
+
+
+1 +1 +1

3:

Sin e P (x) = x3 x 1 has roots ; ; , the polynomial P (x 1) =


x3 3x2 +2x 1 has roots +1; +1; +1. By a standard formula,
the sum of the re ipro als of the roots of x3 + 2 x2 + 1 x + 0 is
1= 0 , so the given expression equals 2(2) 3 = 1:
2. Find all real solutions to the following system of equations:
4x2
= y
1 + 4x2
4y2
= z
1 + 4y2
4z 2
= x:
1 + 4z 2
De ne f (x) = 4x2 =(1 + 4x2 ); the range of f is [0; 1),
so x; y; z must lie in that interval. If one of x; y; z is zero, then all
three are, so assume they are nonzero. Then f (x)=x = 4x=(1 +
4x2 ) is at least 1 by the AM-GM inequality, with equality for x =
1=2. Therefore x  y  z  x, and so equality holds everywhere,
implying x = y = z = 1=2. Thus the solutions are (x; y; z ) =
(0; 0; 0); (1=2; 1=2; 1=2).
Solution:

3. Let f (n) be the number of permutations a1 ; : : : ; an of the integers


1; : : : ; n su h that
(i) a1 = 1;
(ii) jai ai+1 j  2, i = 1; : : : ; n 1.
9

Determine whether f (1996) is divisible by 3.


Solution: Let g (n) be the number of permutations of the desired

form with an = n. Then either an 1 = n 1 or an 1 = n 2; in


the latter ase we must have an 2 = n 1 and an 3 = n 3. Hen e
g(n) = g(n 1)+ g(n 3) for n  4. In parti ular, the values of g(n)
modulo 3 are g(1) = 1; 1; 1; 2; 0; 1; 0; 0; : : : repeating with period 8.
Now let h(n) = f (n) g(n); h(n) ounts permutations of the desired
form where n o urs in the middle, sandwi hed between n 1 and n
2. Removing n leaves an a eptable permutation, and any a eptable
permutation on n 1 symbols an be so produ ed ex ept those ending
in n 4; n 2; n 3; n 1. Hen e h(n) = h(n 1)+g(n 1) g(n 4) =
h(n 1)+g(n 2); one he ks that h(n) modulo 3 repeats with period
24.
Sin e 1996  4 (mod 24), we have f (1996)  f (4) = 4 (mod 3), so
f (1996) is not divisible by 3.
4. Let 4ABC be an isos eles triangle with AB = AC . Suppose that
the angle bise tor of \B meets AC at D and that BC = BD + AD.
Determine \A.
Solution: Let = \A; = (

by the Law of Sines,

BC =

)=4 and assume AB = 1. Then

sin
sin
sin
; BD =
; AD =
:
sin 2
sin 3
sin 3

Thus we are seeking a solution to the equation


sin(

4 ) sin 3 = (sin(

4 ) + sin ) sin 2 :

Using the sum-to-produ t formula, we rewrite this as


os

os 7 = os 2

os 6 + os

Can elling os , we have os 3 os 7 = os 2


implies
sin 2 sin 5 = sin 2 sin 4 :
Now sin 5 = sin 4 , so 9 =  and = =9.
10

os 3 :
os 6 , whi h

5. Let r1 ; r2 ; : : : ; rm be a given set of positive rational


Pm numbers whose
sum is 1. De ne the fun tion f by f (n) = n
k=1 brk n for ea h
positive integer n. Determine the minimum and maximum values of
f (n).
Solution: Of ourse brk n  rk n, so f (n)  0, with equality for
n = 0, so 0 is the minimum value. On the other hand, we have
rk n brk n < 1, so f (n)  m 1. Here equality holds for n = t 1
if t is the least ommon denominator of the rk .

11

1.3 China
1. Let H be the ortho enter of a ute triangle ABC . The tangents from
A to the ir le with diameter BC tou h the ir le at P and Q. Prove
that P; Q; H are ollinear.
Solution: The line P Q is the polar of A with respe t to the ir le,
so it su es to show that A lies on the pole of H . Let D and E
be the feet of the altitudes from A and B , respe tively; these also
lie on the ir le, and H = AD \ BE . The polar of the line AD
is the interse tion of the tangents AA and DD, and the polar of
the line BE is the interse tion of the tangents BB and EE . The
ollinearity of these two interse tions with C = AE \BD follows from
applying Pas al's theorem to the y li hexagons AABDDE and
ABBDEE . (An elementary solution with ve tors is also possible
and not di ult.)

2. Find the smallest positive integer K su h that every K -element subset of f1; 2; : : : ; 50g ontains two distin t elements a; b su h that a + b
divides ab.
Solution: The minimal value is k = 39. Suppose a; b 2 S are su h
that a + b divides ab. Let = g d(a; b), and put a = a1 ; b = b1 , so
that a1 and b1 are relatively prime. Then (a1 + b1 ) divides 2 a1 b1,
so a1 + b1 divides a1 b1 . Sin e a1 and b1 have no ommon fa tor,
neither do a1 and a1 + b1 , or b1 and a1 + b1. In short, a1 + b1 divides
.
Sin e S  f1; : : : ; 50g, we have a + b  99, so (a1 + b1 )  99, whi h
implies a1 + b1  9; on the other hand, of ourse a1 + b1  3. An
exhaustive sear h produ es 23 pairs a; b satisfying the ondition:

a1 + b1 = 3
a1 + b1 = 4
a1 + b1 = 5
a1 + b1 = 6
a1 + b1 = 7
a1 + b1 = 8
a1 + b1 = 9

(6; 3); (12; 6); (18; 9); (24; 12);


(30; 15); (36; 18); (42; 21); (48; 24)
(12; 4); (24; 8); (36; 12); (48; 16)
(20; 5); (40; 10); (15; 10); (30; 20); (45; 30)
(30; 6)
(42; 7); (35; 14); (28; 21)
(40; 24)
(45; 36)
12

Let M = f6; 12; 15; 18; 20; 21; 24; 35; 40; 42; 45; 48g and T = f1; : : : ; 50g
M . Sin e ea h pair listed above ontains an element of M , T does
not have the desired property. Hen e we must take k  jT j +1 = 39.
On the other hand, from the 23 pairs mentioned above we an sele t
12 pairs whi h are mutually disjoint:
(6; 3); (12; 4); (20; 5); (42; 7); (24; 8); (18; 9);
(40; 10); (35; 14); (30; 15); (48; 16); (28; 21); (45; 36):
Any 39-element subset must ontain both elements of one of these
pairs. We on lude the desired minimal number is k = 39.
3. Let f : R ! R be a fun tion su h that for all x; y 2 R,

f (x3 + y3) = (x + y)(f (x)2

f (x)f (y) + f (y)2 ):

(1)

Prove that for all x 2 R, f (1996x) = 1996f (x).

Setting x = y = 0 in the given equation, we have


f (0) = 0. Setting y = 0, we nd f (x3 ) = xf (x)2 , or equivalently,

Solution:

f (x) = x1=3 f (x1=3 )2 :

(2)

In parti ular, x and f (x) always have the same sign, that is, f (x)  0
for x  0 and f (x)  0 for x  0.
Let S be the set

S = fa > 0 : f (ax) = af (x)8x 2 Rg:

Clearly 1 2 S ; we will show a1=3 2 S whenever a 2 S . In fa t,

axf (x)2 = af (x3 ) = f (ax3 ) = f ((a1=3 x)3 ) = a1=3 f (a1=3 x)2

and so

[a1=3 f (x)2 = f (a1=3 x)2 :


Sin e x and f (x) have the same sign, we on lude f (a1=3 x) = a1=3 f (x).
Now we show that a; b 2 S implies a + b 2 S :

f ((a + b)x) =
=
=
=

f ((a1=3 x1=3 )3 + (b1=3 x1=3 )3 )


(a1=3 + b1=3 )[f (a1=3 x1=3 )2 f (a1=3 x1=3 )f (b1=3 x1=3 ) + f (b1=3 x1=3 )2
(a1=3 + b1=3 )(a2=3 a1=3 b1=3 + b2=3 )x1=3 f (x1=3 )2
(a + b)f (x):
13

By indu tion, we have n 2 S for ea h positive integer n, so in parti ular, f (1996x) = 1996f (x) for all x 2 R.
4. Eight singers parti ipate in an art festival where m songs are performed. Ea h song is performed by 4 singers, and ea h pair of singers
performs together in the same number of songs. Find the smallest
m for whi h this is possible.
Solution: Let r be the number of songs ea h pair of singers performs together, so that

 

 

4
8
=r
2
2

and so m = 14r=3; in parti ular, m  14. However, m = 14 is indeed


possible, using the arrangement

f1; 2; 3; 4g
f3; 4; 5; 6g
f2; 4; 5; 7g
f1; 2; 7; 8g

f5; 6; 7; 8g f1; 2; 5; 6g f3; 4; 7; 8g


f1; 3; 5; 7g f2; 4; 6; 8g f1; 3; 6; 8g
f1; 4; 5; 8g f2; 3; 6; 7g f1; 4; 6; 7g
f2; 3; 5; 8g:

5. Suppose n 2 N, x0 = 0, xi > 0 for i = 1; 2; : : : ; n, and


Prove that
1

n
X
i=1

Pn

i=1 xi

= 1.

xi p

< :
xi +    + xn 2
0 +    + xi 1 

p1 + x

Solution: The left inequality follows from the fa t that

px

1
+    + xn  (1+ x0 +    + xn ) = 1;
2
P
so that the middle quantity is at least xi = 1. For the right
inequality, let
p

1 + x0 + x1 +    + xi

i = ar sin(x0 +    + xi ) (i = 0; : : : ; n)
so that

1 + x0 + x1 +    + xi
14

px +    + x = os 
i
n
i

and the desired inequality is


n
X

sin i sin i
os i 1
i=1


< :
2

Now note that

 
 +
= 2 os i i 1 sin i i 1 < osi 1 (i i 1 );
2
2
using the fa ts that i 1 < i and that sin x < x for x > 0, so that
sin i

sin i

n
X

sin i sin i
os i 1
i=1

<

n
X
i=1

i

i

= n


0 < ;
2

as laimed.
6. In triangle ABC , \C = 90, \A = 30 and BC = 1. Find the
minimum of the length of the longest side of a triangle ins ribed in
ABC (that is, one su h that ea h side of ABC ontains a di erent
vertex of the triangle).
Solution: We rst nd the minimum side length of an equilateral

triangle ins ribed in ABC . Let D be a point on BC and put x =


BD. Then
p take points E; F on CA; AB , respe tively, su h that
CE = 3x=2 and BF = 1 x=2. A al ulation using the Law of
Cosines shows that
7
DF = DE = EF = x2
4
2

7
x
2x + 1 =
4

4
7

2

3
+ :
7

Hen e the triangle


DEF is equilateral, and its minimum possible
p
side length is 3=7.
We now argue that the minimum possible longest side must o ur for
some equilateral triangle. Starting with an arbitrary triangle, rst
suppose it is not isos eles. Then we an slide one of the endpoints
of the longest side so as to de rease its length; we do so until there
are two longest sides, say DE and EF . We now x D, move E so
as to de rease DE and move F at the same time so as to de rease
EF ; we do so until all three sides be ome equal in length. (It is ne
15

if the verti es move onto the extensions of the sides, sin e the bound
above applies in that ase as well.)
p
Hen e the mininum is indeed 3=7, as desired.

16

1.4 Cze h and Slovak Republi s


1. Prove that if a sequen e fG(n)g1
n=0 of integers satis es
G(0) = 0;
G(n) = n G(G(n))

(n = 1; 2; 3; : : :);

then
(a) G(k)  G(k 1) for any positive integer k;
(b) no integer k exists su h that G(k 1) = G(k) = G(k + 1).
Solution:

(a) We show by indu tion that G(n) G(n 1) 2 f0; 1g for all n.
If this holds up to n, then

G(n + 1) G(n) = 1 + G(G(n 1)) G(G(n)):


If G(n 1) = G(n), then G(n + 1) G(n) = 1; otherwise,
G(n 1) and G(n) are onse utive integers not greater than
n, so G(G(n)) G(G(n 1)) 2 f0; 1g, again ompleting the
indu tion.
(b) Suppose that G(k 1) = G(k) = G(k + 1) + A for some k; A.
Then

A = G(k + 1) = k + 1 G(G(k)) = k + 1 G(A)


and similarly A = k
ontradi tion.

G(A) (repla ing k + 1 with k above), a

Note: It an be shown that G(n) = bnw for w = ( 5

1)=2.

2. Let ABC be an a ute triangle with altitudes AP; BQ; CR. Show
that for any point P in the interior of the triangle P QR, there exists
a tetrahedron ABCD su h that P is the point of the fa e ABC at
the greatest distan e (measured along the surfa e of the tetrahedron)
from D.

17

We rst note that if S is the ir um ir le of an a ute


triangle KLM , then for any point X 6= S inside the triangle, we
have
minfXK; XL; XM g < SK = SL = SM;
sin e the dis s entered at K; L; M whose bounding ir les pass
through S over the entire triangle.
Fix a point V in the interior of the triangle P QR; we rst assume
the desired tetrahedron exists and determine some of its properties.
Rotate the fa es ABD; BCD; CAD around their ommon edges with
fa e ABC into the plane ABC , so that the images D1 ; D2 ; D3 of D
lie outside of triangle ABC . We shall hoose D so that triangle
D1 D2 D3 is a ute, ontains triangle ABC and has ir um enter V ;
this su es by the above observation.
In other words, we need a point D su h that AV is the perpendi ular bise tor of D1 D3 , BV that of D1 D2 , and CV that of D2 D3 . We
thus need \D1 D2 D3 =  \BV C and so on. Sin e V lies inside
P QR, the angle BV C is a ute, and so \D1 D2 D3 is xed and a ute.
We may then onstru t an arbitrary triangle D10 D20 D30 similar to
the unknown triangle D1 D2 D3 , let V 0 be its ir um enter, and onstru t points A0 ; B 0 ; C 0 on the rays from V through the midpoints of
D30 D10 ; D10 D20 ; D20 D30 , respe tively, so that triangles A0 B 0 C 0 and ABC
are similar. We an also ensure that the entire triangle A0 B 0 C 0 lies
inside D10 D20 D30 . Then folding up the hexagon A0 D10 B 0 D20 C 0 D30 along
the edges of triangle A0 B 0 C 0 produ es a tetrahedron similar to the
required tetrahedron.

Solution:

3. Given six three-element subsets of a nite set X , show that it is


possible to olor the elements of X in two olors su h that none of
the given subsets is all in one olor.
Solution: Let A1 ; : : : ; A6 be the subsets; we indu t on the number

n of elements of X , andthere is no loss of generality in assuming


n  6. If n = 6, sin e 63 = 20 > 2  6, we an nd a three-element
subset Y of X not equal to any of A1 ; : : : ; A6 or their omplements;
oloring the elements of Y in one olor and the other elements in the
other olor meets the desired ondition.
18

Now suppose n > 6. There must be two elements u; v of X su h


that fu; vg is not a subset of any Ai , sin e there are at least 72 = 21
pairs, and at most 6  3 = 18 lie in an Ai . Repla e all o urren es of
u and v by a new element w, and olor the resulting elements using
the indu tion hypothesis. Now olor the original set by giving u and
v the same olor given to w.
4. An a ute angle XCY and points A and B on the rays CX and
CY , respe tively, are given su h that jCX j < jCAj = jCB j < jCY j.
Show how to onstru t a line meeting the ray CX and the segments
AB; BC at the points K; L; M , respe tively, su h that

KA  Y B = XA  MB = LA  LB 6= 0:
Solution: Suppose K; L; M have already been onstru ted. The
triangles ALK and BY L are similar be ause \LAK = \Y BL and
KA=LA = LB=Y B . Hen e \ALK = \BY L. Similarly, from the
similar triangles ALX and BML we get \AXL = \MLB . We
also have \MLB = \ALK sin e M; L; K are ollinear; we on lude
\LY B = \AXL. Now

\XLY = \XLB +\BLY = \XAL+\AXL+\ABM \LY B = 2\ABC:


We now onstru t the desired line as follows: draw the ar of points
L su h that \XLY = 2\ABC , and let L be its interse tion with
AB . Then onstru t M on BC su h that \BLM = \AXL, and let
K be the interse tion of LM with CA.
5. For whi h integers k does there exist a fun tion f : N

! Z su h that

(a) f (1995) = 1996, and


(b) f (xy) = f (x) + f (y) + kf (g d(x; y)) for all x; y 2 N ?
Solution: Su h f exists for k = 0 and k = 1. First take x = y in

(b) to get f (x2 ) = (k + 2)f (x). Applying this twi e, we get

f (x4 ) = (k + 2)f (x2 ) = (k + 2)2 f (x):

19

On the other hand,

f (x4 ) = f (x) + f (x3 ) + kf (x) = (k + 1)f (x) + f (x3 )


= (k + 1)f (x) + f (x) + f (x2 ) + kf (x) = (2k + 2)f (x) + f (x2 )
= (3k + 4)f (x):
Setting x = 1995 so that f (x) 6= 0, we dedu e (k + 2)2 = 3k + 4,
whi h has roots k = 0; 1. For k = 0, an example is given by

f (pe11    penn ) = e1 g(p1 ) +    + en g(pn );

where g(5) = 1996 and g(p) = 0 for all primes p 6= 5. For k = 1, an


example is given by

f (pe11    penn ) = g(p1 ) +    + g(pn ):


6. A triangle ABC and points K; L; M on the sides AB; BC; CA, respe tively, are given su h that
AK BL CM 1
=
=
= :
AB BC CA 3
Show that if the ir um ir les of the triangles AKM; BLK; CML
are ongruent, then so are the in ir les of these triangles.
Solution: We will show that ABC is equilateral, so that AKM; BLK; CML

are ongruent and hen e have the same inradius. Let R be the ommon ir umradius; then

KL = 2R sin A; LM = 2R sin B; MK = 2R sin C;


so the triangles KLM and ABC are similar. Now we ompare areas:
2
[AKM = [BLK = [CLM = [ABC ;
9
and the oe ient of similarity between KLM
so [KLM = 31 [ABC
p
and ABC must be 1=3. By the law of osines applied to ABC and
AKM ,

a2 = b2 + 2 2b os A
 2  
2 2b
1 2
2p
+
a =
2
os A:
3
3
3
33
20

From these we dedu e a2 = 2b2 2 , and similarly b2 = 2 2 a2 ,


2 = 2a2 b2 . Combining these gives a2 = b2 = 2 , so ABC is
equilateral, as desired.

21

1.5 Fran e
1. Let ABC be a triangle and onstru t squares ABED; BCGF; ACHI
externally on the sides of ABC . Show that the points D; E; F; G; H; I
are on y li if and only if ABC is equilateral or isos eles right.
Suppose D; E; F; G; H; I are on y li ; the perpendi ular bise tors of DE; F G; HI oin ide with those of AB; BC; CA,
respe tively, so the enter of the ir le must be the ir um enter O
of ABC . By equating the distan es OD and OF , we nd
Solution:

( os B + 2 sin B )2 + sin2 B = ( os C + 2 sin C )2 = sin2 C:


Expanding this and an elling like terms, we determine
sin2 B + sin B os B = sin2 C + sin C os C:
Now note that

2(sin2  + sin  os ) = 1 os 2 + sin  = 1 + 2 sin(2

=4):

Thus we either have B = C or 2B =4+2C =4 = , or B + C =


3=4. In parti ular, two of the angles must be equal, say A and B ,
and we either have A = B = C , so the triangle is equilaterla, or
B + ( 2B ) = 3=4, in whi h ase A = B = =4 and the triangle
is isos eles right.
2. Let a; b be positive integers with a odd. De ne the sequen e fung
as follows: u0 = b, and for n 2 N ,

un+1 =

un if un is even
un + a otherwise.
1
2

(a) Show that un  a for some n 2 N.


(b) Show that the sequen e fung is periodi from some point onwards.
Solution:

(a) Suppose un > a. If un is even, un+1 = un =2 < un; if un is odd,


un+2 = (un + a)=2 < un . Hen e for ea h term greater than
22

a, there is a smaller subsequent term. These form a de reasing subsequen e whi h must eventually terminate, whi h only
o urs on e un  a.
(b) If um  a, then for all n  m, either un  a, or un is even
and un  2a, by indu tion on n. In parti ular, un  2a for all
m  n, and so some value of un eventually repeats, leading to
a periodi sequen e.
hoose
3. (a) Find the minimum value of xx for x a positive real number.
(b) If x and y are positive real numbers, show that xy + yx > 1.
Solution:

(a) Sin e xx = ex log x and ex is an in reasing fun tion of x , it


su es to determine the minimum of x log x. This is easily done
by setting its derivative 1 + log x to zero, yielding x = 1=e. The
se ond derivative 1=x is positive for x > 0, so the fun tion is
everywhere onvex, and the unique extremum is indeed a global
minimum. Hen e xx has minimum value e 1=e .
(b) If x  1, then xy  1 for y > 0, so we may assume 0 < x; y < 1.
Without loss of generality, assume x  y; now note that the
fun tion f (x) = xy + yx has derivative f 0(x) = xy log x + yx 1.
Sin e yx  xx  xy for x  y and 1=x  log x, we see that
f 0 (x) > 0 for 0  x  y and so the minimum of f o urs with
x = 0, in whi h ase f (x) = 1; sin e x > 0, we have stri t
inequality.
4. Let n be a positive integer. We say a positive integer k satis es the
ondition Cn if there exist 2k distin t positive integers a1 ; b1 ; : : :,
ak ; bk su h that the sums a1 + b1; : : : ; ak + bk are all distin t and less
than n.
(a) Show that if k satis es the ondition Cn , then k  (2n 3)=5.
(b) Show that 5 satis es the ondition C14 .
( ) Suppose (2n 3)=5 is an integer. Show that (2n 3)=5 satis es
the ondition Cn .
23

(a) If k satis es the ondition Cn , then


1 + 2 +    + 2k  (n 1) + (n 2) +    + (n k);
or k(2k + 1)  k(2n k 1)=2, or 4k + 2  2n
5k  2n 3.
(b) We obtain the sums 9; 10; 11; 12; 13 as follows:

1, or

9 = 7 + 2; 10 = 6 + 4; 11 = 10 + 1; 12 = 9 + 3; 13 = 8 + 5:
( ) Imitating the above example, we pair 2k with 1, 2k 1 with 3,
and so on, up to 2k (k 1)=2 with k (where k = (2n 3)=5),
giving the sums 2k + 1; : : : ; n 1. Now we pair 2k (k + 1)=2
with 2, 2k (k + 3)=2 with 4, and so on, up to k + 1 with k 1,
giving the sums from (5k + 1)=2 to 2k.

24

1.6 Germany
1. Starting at (1; 1), a stone is moved in the oordinate plane a ording
to the following rules:
(i) From any point (a; b), the stone an move to (2a; b) or (a; 2b).
(ii) From any point (a; b), the stone an move to (a b; b) if a > b,
or to (a; b a) if a < b.
For whi h positive integers x; y an the stone be moved to (x; y)?
Solution: It is ne essary and su ient that g d(x; y ) = 2s for some

nonnegative integer s. We show ne essity by noting that g d(p; q) =


g d(p; q p), so an odd ommon divisor an never be introdu ed,
and noting that initially g d(1; 1) = 1.
As for su ien y, suppose g d(x; y) = 2s . Of those pairs (p; q) from
whi h (x; y) an be rea hed, hoose one to minimize p + q. Neither p
nor q an be even, else one of (p=2; q) or (p; q=2) is an admissible pair.
If p > q, then (p; q) is rea hable from ((p + q)=2; q), a ontradi tion;
similarly p < q is impossible. Hen e p = q, but g d(p; q) is a power
of 2 and neither p nor q is even. We on lude p = q = 1, and so
(x; y) is indeed rea hable.
2. Suppose S is a union of nitely many disjoint subintervals of [0; 1
su h that no two points in S have distan e 1=10. Show that the total
length of the intervals omprising S is at most 1=2.
Cut the given segment into 5 segments of length 1=5.
Let AB be one of these segments and M its midpoint. Translate
~ . No olored point an have a
ea h point of AM by the ve tor MB
olored image, so all of the olored intervals of AB an be pla ed in
MB without overlap, and their total length therefore does not ex eed
1=10. Applying this reasoning to ea h of the 5 segments gives the
desired result.
Solution:

3. Ea h diagonal of a onvex pentagon is parallel to one side of the


pentagon. Prove that the ratio of the length of a diagonal to that of
its orresponding side is the same for all ve diagonals, and ompute
this ratio.
25

Let CE and BD interse t in S , and hoose T on AB


with CT k BD. Clearly S lies inside the pentagon and T lies outside.
Put d = AB; = AE; and s = SC=AB ; then the similar triangles
SCD and ABE give SC = sd and SD = s . The parallelograms
ABSE , AT CE , BT CS give SE = d, T C = , BT = sd. From the
similar triangles ESD and AT C we get SD=T C = SE=T A, and so
s = = d=(d +p
sd). We on lude s is the positive root of s(1 + s) = 1,
whi h is s = ( 5 1)=2.
Finally,
p we determine EC = d(1+ s) and the ratio EC=AB = 1+ s =
(1 + 5)=2, and the value is learly the same for the other pairs.
Solution:

4. Prove that every integer k > 1 has a multiple less than k4 whose
de imal expansion has at most four distin t digits.
Let n be the integer su h that 2n 1  k < 2n. For
n  6 the result is immediate, so assume n > 6.
Let S be the set of nonnegative integers less than 10n whose de imal
digits are all 0s or 1s. Sin e jS j = 2n > k, we an nd two elements
a < b of S whi h are ongruent modulo k, and b a only has the
digits 8, 9, 0, 1 in its de imal representation. On the other hand,
Solution:

b a  b  1 + 10 +    + 10n
hen e b a is the desired multiple.

26

< 10n < 16n

k ;
4

1.7 Gree e
1. In a triangle ABC the points D; E; Z; H;  are the midpoints of the
segments BC; AD; BD; ED; EZ , respe tively. If I is the point of
interse tion of BE and AC , and K is the point of interse tion of
H  and AC , prove that
(a)
(b)
( )
(d)

AK = 3CK ;
HK = 3H ;
BE = 3EI ;
the area of ABC is 32 times that of E H .

Introdu e oblique oordinates with B = (0; 0), C =


(24; 0), A = (0; 24). We then ompute D = (12; 0), E = (6; 12),
Z = (6; 0), H = (9; 6),  = (6; 6), I = (8; 16), K = (18; 6), from
whi h the relations AK = 3CK , HK = 3H , BE = 3EI are
evident. As for E H , it has base H whose length is half that of
ZD, and ZD is 1=4 as long as BC , so H = 1=8BC . The altitude
from E to H is 1=4 the altitude from A to BC , so we on lude the
area of E H is 1=32 times that of ABC .
Solution:

2. Let ABC be an a ute triangle, AD; BE; CZ its altitudes and H its
ortho enter. Let AI; A be the internal and external bise tors of
angle A. Let M; N be the midpoints of BC; AH , respe tively. Prove
that
(a) MN is perpendi ular to EZ ;
(b) if MN uts the segments AI; A at the points K; L, then KL =
AH .
Solution:

(a) The ir le with diameter AH passes through Z and E , and


so ZN = ZE . On the other hand, MN is a diameter of the
nine-point ir le of ABC , and Z and E lie on that ir le, so
ZN = ZE implies that ZE ? MN .
(b) As determined in (a), MN is the perpendi ular bise tor of segment ZE . The angle bise tor AI of \EAZ passes through
27

the midpoint of the minor ar EZ , whi h learly lies on MN ;


therefore this midpoint is K . By similar reasoning, L is the
midpoint of the major ar EZ . Thus KL is also a diameter of
ir le EAZ , so KL = MN .
3. Given 81 natural numbers whose prime divisors belong to the set
f2; 3; 5g, prove there exist 4 numbers whose produ t is the fourth
power of an integer.
Solution: It su es to take 25 su h numbers. To ea h number,
asso iate the triple (x2 ; x3 ; x5 ) re ording the parity of the exponents
of 2, 3, and 5 in its prime fa torization. Two numbers have the same
triple if and only if their produ t is a perfe t square. As long as there
are 9 numbers left, we an sele t two whose produ t is a square; in
so doing, we obtain 9 su h pairs. Repeating the pro ess with the
square roots of the produ ts of the pairs, we obtain four numbers
whose produ t is a fourth power. (See IMO 1985/4.)

4. Determine the number of fun tions f : f1; 2; : : : ; ng ! f1995; 1996g


whi h satsify the ondition that f (1) + f (2) +    + f (1996) is odd.
Solution: We an send 1; 2; : : : ; n

1 anywhere, and the value of


f (n) will then be uniquely determined. Hen e there are 2n 1 su h
fun tions.

28

1.8 Iran
1. Prove the following inequality for positive real numbers x; y; z :
(xy + yz + zx)
Solution:

omes

sym

1
1
1
+
+
2
2
(x + y)
(y + z )
(z + x)2

 49 :

After learing denominators, the given inequality be-

4x5 y

x4 y2 3x3 y3 + x4 yz 2x3 y2 z + x2 y2 z 2  0;

where the symmetri sum runs over all six permutations of x; y; z . (In
parti ular, this means the oe ient of x3 y3 in the nal expression
is -6, and that of x2 y2z 2 is 6.)
Re all S hur's inequality:

x(x y)(x z ) + y(y z )(y x) + z (z x)(z

y)  0:

Multiplying by 2xyz and olle ting symmetri terms, we get


X

sym

x4 yz 2x3 y2 z + x2 y2 z 2  0:

On the other hand,


X

(x5 y
sym

x4 y2 ) + 3(x5 y x3 y3 )  0

by two appli ations of AM-GM; ombining the last two displayed


inequalities gives the desired result.
2. Prove that for every pair m; k of natural numbers, m has a unique
representation in the form


 

a
a
a
m = k + k 1 ++ t ;
t
k 1
k
where

ak > a k

>    > at  t  1:
29

We rst show uniqueness. Suppose m is represented


by two sequen es ak ; : : : ; at and bk ; : : : ; bt. Find the rst position in
whi h they di er; without loss of generality, assume this position is
k and that ak > bk . Then
Solution:

m

b 1
b k+1
b +1
bk
+ k
+ + k
< k
k
k 1
1
k

 m;

a ontradi tion.
To show existen e,
apply the greedy algorithm: nd the largest ak

su h that akk  m, and apply the same algorithm with m and k
repla ed by m akk and k 1. We need only make sure that the
sequen e obtained is indeed
de reasing, but
be ause by

 this follows

assumption, m < akm+1 , and so m akk < kak1 .
3. In triangle ABC , we have \A = 60 . Let O; H; I; I 0 be the ir um enter, ortho enter, in enter, and ex enter opposite A, respe tively,
of ABC . Let B 0 and C 0 be points on the segments AC and AB su h
that AB = AB 0 and AC = AC 0 . Prove that:
(a) The eight points B; C; H; O; I; I 0 ; B 0 ; C 0 are on y li .
(b) If OH interse ts AB and AC at E and F , respe tively, the
perimeter of triangle AEF equals AB + AC .
( ) OH = jAB AC j.

Solution:

(a) The ir le through B; C; H onsists of all points P su h that


\BP C = \BHC = 180 \CAB = 120 (as dire ted angles mod 180). Thus O lies on this ir le, as does I be ause
\BIC = 90 + 21 \A = 30. Note that the ir le with diameter II 0 passes through B and C (sin e internal and external
angle bise tors are perpendi ular). Hen e I 0 also lies on the ir le, whose enter lies on the internal angle bise tor of A. This
means re e ting B and C a ross this bise tor gives two more
points B 0 ; C 0 on the ir le.
(b) Let R be the ir umradius of triangle ABC . The re e tion
a ross AI maps B and C to B 0 and C 0 , and preserves I . By
30

(a), the ir le BCHO is then preserved, and hen e H maps to


O. In other words, AHO is isos eles with AH = AO = R and
\HAO = j j, writing for \B and for \C .
In parti ular, the altitude of AHO has length R os and
so the equilateral triangle AEF has perimeter

3R os( ) = 2R sin( + ) os( ) = 2R(sin +sin ) = AB +AC:

( ) We use a; b; to denote the lengths of BC; CA; AB . By a standard omputation using ve tors, we nd OH 2 = 9R2 (a2 +b2 +
2 ), but sin e a = 2R sin 60, we have OH 2 = 2a2 b2 2 . By
the Law of Cosines, a2 = b2 + 2 b , so OH 2 = b2 + 2 2b =
(b )2 , and so OH = jb j.
4. Let ABC be a s alene triangle. The medians from A; B; C meet
the ir um ir le again at L; M; N , respe tively. If LM = LN , prove
that 2BC 2 = AB 2 + AC 2 .
Let G be the entroid of triangle ABC ; then triangles NLG and AGL are similar, so LN=AC = LG=CG. Similarly
LM=AB = GL=BG. Thus if LM = LN , then AB=AC = BG=CG.
Using Stewart's theorem to ompute the lengths of the medians, we
have
AB 2 2AB 2 + 2BC 2 AC 2
=
AC 2 2AC 2 + 2BC 2 AB 2
whi h redu es to (AC 2 AB 2 )(2BC 2 AB 2 AC 2 ) = 0. Sin e the
triangle is s alene, we on lude 2BC 2 = AB 2 + AC 2 .
Solution:

5. The top and bottom edges of a hessboard are identi ed together,


as are the left and right edges, yielding a torus. Find the maximum
number of knights whi h an be pla ed so that no two atta k ea h
other.
Solution: The maximum is 32 knights; if the hessboard is alternately olored bla k and white in the usual fashion, an optimal
arrangement puts a knight on ea h bla k square. To see that this
annot be improved, suppose that k knights are pla ed. Ea h knight
atta ks 8 squares, but no uno upied square an be atta ked by more
than 8 knights. Therefore 8k  8(64 k), when e k  32.

31

6. Find all nonnegative real numbers a1  a2  : : :  an satisfying


n
X
i=1

ai = 96;

n
X
i=1

a2i = 144;

n
X
i=1

a3i = 216:

Adding or removing zeroes has no e e t, so we may


assume the ai are positive. By Cau hy-S hwarz,

Solution:

(a1 +    + an )(a31 +    + a3n )  (a21 +    + a2n ):


Sin e 96  216 = 1442, we have equality, so the sequen es a1 ;    ; an
and a31 ;    ; a3n are proportional, so that a1 =    = an = a. Now
na = 96; na2 = 144 so that a = 3=2; n = 32.
7. Points D and E lie on sides AB and AC of triangle ABC su h
that DE jjBC . Let P be an arbitrary point inside ABC . The lines
P B and P C interse t DE at F and G, respe tively. If O1 is the
ir um enter of P DG and O2 is the ir um enter of P F E , show
that AP ? O1 O2 .
(Note: angles are dire ted modulo .) Let M be the
se ond interse tion of AB with the ir um ir le of DP G, and let
N be the se ond interse tion of N with the ir um ir le of EP F .
Now \DMP = \DGP by y li ity, and \DGP = \BCP by parallelism, so \DMP = \BCP and the points B; C; P; M are on y li . Analogously, B; C; P; N are on y li . Therefore the points
B; C; M; N are on y li , so \DMN = \BCN . Again by parallels,
\BCN = \DEN , so the points D; E; M; N are on y li .
We now apply the radi al axis theorem to the ir um ir les of DGP ,
EP F , and DEMN to on lude that DM \ EN = A lies on the
radi al axis of the ir les P DG and P EF , so AP ? O1 O2 as desired.

Solution:

8. Let P (x) be a polynomial with rational oe ients su h that


P 1 (Q )  Q . Show that P is linear.
Solution: By a suitable variable substitution and onstant fa tor,

we may assume P (x) is moni and has integer oe ients; let P (0) =
0 . If p is a su iently large prime, the equation P (x) = p + 0
has a single real root, whi h by assumption is rational and whi h we
32

may also assume is positive (sin e P has positive leading oe ient).


However, by the rational root theorem, the only rational roots of
P (x) p 0 an be 1 and p. Sin e the root must be positive
and annot be 1 for large p, we have P (p) p 0 = 0 for in nitely
many p, so P (x) = x + 0 is linear.
9. For S = fx1 ; x2 ; : : : ; xn g a set of n real numbers, all at least 1, we
ount the number of reals of the form
n
X
i=1

i xi ; i 2 f0; 1g

lying in an open interval I of length 1. Find the maximum value of


this ount over all I and S .


Solution: The maximum is bn=n2 , a hieved by taking xi = 1 +

i=(n + 1). To see that this annot be improved, note that for any
permutation  of f1; : : : ; ng, at most one of the sets f(1); : : : ; (i)g
for i = 1; : : : ; n has sum lying in I . Thus if T is the set of subsets
whose sum lies in I , we have
X

t2T

t!(n t)!  n! ,


In parti ular, we have jT j  bn=n2 .

33

 

n
t
t2T

 1:

1.9 Ireland
1. For ea h positive integer n, nd the greatest ommon divisor of n!+1
and (n + 1)!.
Solution: If n + 1 is omposite, then ea h prime divisor of (n + 1)!
is a prime less than n, whi h also divides n! and so does not divide
n! + 1. Hen e f (n) = 1. If n + 1 is prime, the same argument
shows that f (n) is a power of n + 1, and in fa t n + 1jn! + 1 by
Wilson's theorem. However, (n + 1)2 does not divide (n + 1)!, and
thus f (n) = n + 1.

2. For ea h positive integer n, let S (n) be the sum of the digits in the
de imal expansion of n. Prove that for all n,

S (2n)  2S (n)  10S (2n)


and show that there exists n su h that S (n) = 1996S (3n).
Solution: It is lear that S (a + b)  S (a) + S (b), with equality if
and only if there are no arries in the addition of a and b. Therefore
S (2n)  2S (n). Similarly S (2n)  5S (10n) = 5S (n). An example
with S (n) = 1996S (3n) is 133    35 (with 5968 threes).

3. Let f : [0; 1 ! R be a fun tion su h that

(i) f (1) = 1,
(ii) f (x)  0 for all x 2 [0; 1,
(iii) if x; y and x + y all lie in [0; 1, then f (x + y)  f (x) + f (y).

Prove that f (x)  2x for all x 2 K .

Solution: If y > x, then f (y )  f (x) + f (y x), so f is in reasing.

We note that f (2 k )  2 k by indu tion on k (with base ase k = 0),


as 2f (2 k )  f (2 (k 1) ). Thus for x > 0, let k be the positive integer
su h that 2 k < x < 2 (k 1) ; then f (x)  f (2 (k 1) )  2 (k 1) <
2x. Sin e f (0) + f (1)  f (1), we have f (0) = 0 and so f (x)  2x in
all ases.

34

4. Let F be the midpoint of side BC of triangle ABC . Constru t


isos eles right triangles ABD and ACE externally on sides AB and
AC with the right angles at D and E , respe tively. Show that DEF
is an isos eles right triangle.
Solution: Identifying A; B; C with numbers on the omplex plane,

we have F = (B + C )=2, D = B + (A B )r, E = A + (C A)r,


where r = (1 + i)=2. Then E F = A(1 i)=2 B=2 + Ci=2 and
D F = A(1 + i)=2 Bi=2 C=2; in parti ular, D F = i(E F )
and so DEF is an isos eles right triangle.

5. Show, with proof, how to disse t a square into at most ve pie es in


su h a way that the pie es an be reassembled to form three squares
no two of whi h have the same area.
Solution: We disse t a 7  7 square into a 2  2 square A, a 3  3
square B , and three pie es C; D; E whi h form a 6  6 square, as
shown below.
C C C C C A A
C C C C C A A
C C C C C D D
C C C C C D D
C C C C B B B
C C C C B B B
E E E E B B B

6. Let Fn denote the Fibona i sequen e, so that F0 = F1 = 1 and


Fn+2 = Fn+1 + Fn for n  0. Prove that
(i) The statement \Fn+k Fn is divisible by 10 for all positive
integers n" is true if k = 60 and false for any positive integer
k < 60;
(ii) The statement \Fn+t Fn is divisible by 100 for all positive
integers n" is true if t = 300 and false for any positive integer
t < 300.
Solution: A dire t omputation shows that the Fibona i sequen e

has period 3 modulo 2 and 20 modulo 5 ( ompute terms until the


initial terms 0; 1 repeat, at whi h time the entire sequen e repeats),
35

yielding (a). As for (b), one omputes that the period mod 4 is 6.
The period mod 25 turns out to be 100, whi h is awfully many terms
to ompute by hand, but knowing that the period must be a multiple
of 20 helps, and verifying the re urren e Fn+8 = tFn+4 + Fn , where t
is an integer ongruent to 2 modulo 5, shows that the period divides
100; nally, an expli it omputation shows that the period is not 20.
7. Prove that for all positive integers n,
n

21=2  41=4    (2n )1=2 < 4:


Solution: It su es to show

P1

1
X

n
n=1 n=2

= 2:

1 X
1 1 X
1 1
n X
=
=
n
k
n 1 = 2:
n=1 2
n=1 k=n 2
n=1 2
8. Let p be a prime number and a; n positive integers. Prove that if
2p + 3p = an ;
then n = 1.
Solution: If p = 2, we have 22 + 32 = 13 and n = 1. If p > 2, then

p is odd, so 5 divides 2p + 3p and so 5 divides a. Now if n > 1, then


25 divides an and 5 divides
2p + 3p
= 2p 1 2p 2  3 +    + 3p 1  p2p 1 (mod 5);
2+3

a ontradi tion if p 6= 5. Finally, if p = 5, then 25 + 35 = 753 is not


a perfe t power, so n = 1 again.
9. Let ABC be an a ute triangle and let D; E; F be the feet of the
altitudes from A; B; C , respe tively. Let P; Q; R be the feet of the
perpendi ulars from A; B; C to EF; F D; DE , respe tively. Prove
that the lines AP; BQ; CR are on urrent.
Solution: It is a routine exer ise to show that ea h of AP; BQ; CR

passes through the ir um enter of ABC , so they all on ur.


36

10. On a 5  9 re tangular hessboard, the following game is played. Initially, a number of dis s are randomly pla ed on some of the squares,
no square ontaining more than one dis . A turn onsists of moving
all of the dis s subje t to the following rules:
(i) ea h dis may be moved one square up, down, left, or right;
(ii) if a dis moves up or down on one turn, it must move left or
right on the next turn, and vi e versa;
(iii) at the end of ea h turn, no square an ontain two or more
dis s.
The game stops if it be omes impossible to omplete another turn.
Prove that if initially 33 dis s are pla ed on the board, the game
must eventually stop. Prove also that it is possible to pla e 32 dis s
on the board so that the game an ontinue forever.
Solution: If 32 dis s are pla ed in an 8  4 re tangle, they an all

move up, left, down, right, up, et . To show that a game with 33
dis s must stop, label the board as shown:
1
2
1
2
1

2
3
2
3
2

1
2
1
2
1

2
3
2
3
2

1
2
1
2
1

2
3
2
3
2

1
2
1
2
1

2
3
2
3
2

1
2
1
2
1

Note that a dis on 1 goes to a 3 after two moves, a dis on 2 goes to


a 1 or 3 immediately, and a dis on 3 goes to a 2 immediately. Thus
if k dis s start on 1 and k > 8, the game stops be ause there are not
enough 3s to a ommodate these dis s. Thus we assume k  8, in
whi h ase there are at most 16 squares on 1 or 3 at the start, and
so at least 17 on 2. Of these 17, at most 8 an move onto 3 after
one move, so at least 9 end up on 1; these dis s will not all be able
to move onto 3 two moves later, so the game will stop.

37

1.10 Italy
1. Among triangles with one side of a given length ` and with given
area S , determine all of those for whi h the produ t of the lengths
of the three altitudes is maximum.
Solution: Let A; B be two xed points with AB = `, and vary
C along a line parallel to AB at distan e 2S=`. The produ t of the
altitudes of ABC is 8S 3 divided by the lengths of the three sides, so
it su es to minimize AC  BC , or equivalently to maximize sin C .
Let D be the interse tion of the perpendi ular bise tor of AB with
the line through C . If \D is not a ute, the optimal triangles are
learly those with a right angle at C .
Suppose \D is a ute and C 6= D, and assume C is on the same
side of the perpendi ular bise tor of AB as B : we show \D  \C ,
and so the optimal triangle is ABD. The triangles DAC and DBC
have equal base and height, so equal altitude. However, AC > BC
sin e \CAB > \CBA, so sin \DAC < sin \DBC , and sin e the
former is a ute, we have \DAC < \DBC . Adding \CAB + \ABD
to both sides, we get \DAB + \DBA < \CAB + \CBA, and so
\ADB > \ACB , as laimed.

2. Prove that the equation a2 + b2 = 2 + 3 has in nitely many integer


solutions fa; b; g.
Let a be any odd number, let b = (a2
1)=2. Then

Solution:

= (a

b2 = ( + b)( b) = a2

5)=2 and

3:

3. Let A and B be opposite verti es of a ube of edge length 1. Find


the radius of the sphere with enter interior to the ube, tangent to
the three fa es meeting at A and tangent to the three edges meeting
at B .
Solution: Introdu e oordinates so that A = (0; 0; 0); B = (1; 1; 1)
and the edges are parallel to the oordinate axes. If r is the radius
of the sphere, then (r; r; r) is its enter, and (r; 1; 1) is the point of
tangen y of one of the edges at B . Therefore r2 = 2(1 r)2 , giving

38

r2 4r + 2 = 0 and so r = 2
outside of the ube).

2 (the other root puts the enter

4. Given an alphabet with three letters a; b; , nd the number of words


of n letters whi h ontain an even number of a's.

Solution: If there are 2k o uren es of a, these an o ur in 2nk

n 2k ways. So
pla es, and thePremaining
 n 2kpositions an be lled in 2
n
the answer is k 2k 2
. To ompute this, note that

(1 + x)n + (1

x)n

=2

X n 

2k

x2k ;

so the answer is
1 n
1
2 [(1 + 1=2)n + (1 1=2)n = (3n + 1):
2
2
5. Let C be a ir le and A a point exterior to C . For ea h point P on
C , onstru t the square AP QR, where the verti es A; P; Q; R o ur
in ounter lo kwise order. Find the lo us of Q as P runs over C .
Solution: Take the ir le to be the unit ir le in the omplex
plane. Then (Q P )i = A P , so Q = A + (1 i)P . We on lude
the lo us of Q ispthe ir le entered at A whose radius is the norm
of 1 i, namely 2.

6. Whas is the minimum number of squares that one needs to draw on


a white sheet in order to obtain a omplete grid with n squares on
a side?
Solution: It su es to draw 2n 1 squares: in terms of oordinates,
we draw a square with opposite orners (0; 0) and (i; i) for 1  i  n
and a square with opposite orners (i; i) and (n; n) for 1  i  n 1.
To show this many squares are ne essary, note that the segments
from (0; i) to (1; i) and from (n 1; i) to (n; i) for 0 < i < n all must
lie on di erent squares, so surely 2n 2 squares are needed. If it were
possible to obtain the omplete grid with 2n 2 squares, ea h of these
segments would lie on one of the squares, and the same would hold

39

for the segments from (i; 0) to (i; 1) and from (i; n 1) to (i; n) for
0 < i < n. Ea h of the aforementioned horizontal segments shares a
square with only two of the verti al segments, so the only possible
arrangements are the one we gave above without the square with
orners (0; 0) and (n; n), and the 90 rotation of this arrangement,
both of whi h are insu ient. Hen e 2n 1 squares are ne essary.

40

1.11 Japan
1. Consider a triangulation of the plane, i.e. a overing of the plane
with triangles su h that no two triangles have overlapping interiors,
and no vertex lies in the interior of an edge of another triangle.
Let A; B; C be three verti es of the triangulation and let  be the
smallest angle of the triangle 4ABC . Suppose no verti es of the
triangulation lie inside the ir um ir le of 4ABC . Prove there is a
triangle  in the triangulation su h that  \ 4ABC 6= ; and every
angle of  is greater than .
Solution: We may assume  = \A. The ase where ABC belongs

to the triangulation is easy, so assume this is not the ase. If BC


is an edge of the triangulation, one of the two triangles bounded
by BC has ommon interior points with ABC , and this triangle
satis es the desired ondition. Otherwise, there is a triangle BEF
in the triangulation whose interior interse ts BC . Sin e EF rosses
BC at an interior point, \BEF < \BAF < \BAC , so triangle
BEF satis es the desired ondition.
2. Let m and n be positive integers with g d(m; n) = 1. Compute
g d(5m + 7m ; 5n + 7n ).
Solution: Let sn = 5n + 7n . If n  2m, note that

sn = sm sn m

5m7m sn 2m ;

so g d(sm ; sn ) = g d(sm ; sn 2m ):. Similarly, if m < n < 2m, we


have g d(sm ; sn ) = g d(sm ; s2m n ). Thus by the Eu lidean algorithm, we on lude that if m + n is even, then g d(sm ; sn ) =
g d(s1 ; s1 ) = 12, and if m+n is odd, then g d(sm ; sn ) = g d(s0 ; s1 ) =
2.
3. Let x > 1 be a real number whi h is not an integer. For n =
1; 2; 3; : : :, let an = bxn+1 xbxn . Prove that the sequen e fan g
is not periodi .
Solution: Assume, on the ontrary, that there exists p > 0 su h

that ap+n = an for every n. Sin e bxn ! 1 as n ! 1, we have


41

bxn p bxn > 0 for some n; then setting an


for x, we get
bxn p bxn
x=
bxn p bxn
+

+ +1

= an and solving

+1

and so x is rational.
Put y = xp and

bm =

pX1
k=0

xp k 1 amp+k = bxmp+p xp bxm r = bym+1 ybym :

Sin e ap+n = ap , we have bm+1 = bm, and y is also a rational


number whi h is not an integer. Now put m = bym+1 ym ; then
m+1 = y m = ym 1 . This means m annot be an integer for large
m, a ontradi tion.
4. Let  be the maximum of the six angles between the edges of a
regular tetrahedron and a given plane. Find the minimum value of
 over all positions of the plane.
= ABCD have
length 1. If we pla e the tetrahedron so that AC and BC are parallel
to the horizontal plane H , we obtain  = 45 , and we shall show this
is the minimum angle.
Let a; b; ; d be the proje tions of A; B; C; D to the horizontal plane
H , and `1 ; : : : ; `6 the proje tions of the edges L1 ; : : : ; L6 . Sin e the
angle between Li and H has osine `, it su es to onsider the
shortest `i .
If a; b; ; d form a onvex quadrilateral
with largest angle at a, then
p
one of ab or ad is at most 1= 2 sin e bd  1. Otherwise, it is easily
shown that one of the `i originating
from the vertex inside the onvex
p
hull has length at most 1= 3.
Solution: Assume the edges of the tetrahedron

5. Let q be a real number with (1 + 5)=2 < q < 2. For a number n


with binary representation

 2k

pn = qk + ak 1 qk

n = 2k + ak

+    + a1  2 + a0

with ai 2 f0; 1g, we de ne pn as follows:

42

+    + a1 q + a0 :

Prove that there exist in nitely many positive integers k for whi h
there does not exist a positive integer l su h that p2k < pl < p2k+1 .
Solution: De ne the sequen e an as follows:

a2 m =

m
X
k=0

22k ;

a2m+1 =

m
X
k=0

22k+1 :

We will show that k = an satis es the given ondition by indu tion


on n. The ases n = 0; 1 follow by noting
1 < q < q + 1 < q2 < q2 + 1 < q2 + q < q2 + q + 1
and pl  qp  q3 > q2 + q = p6 for l  8.
Now suppose n  2, assume the indu tion hypothesis, and suppose
by way of ontradi tion that there exists l su h that p2an < pl <
p2an+1 . The argument falls into six ases, whi h we summarize in a
table. The rst olumn gives the onditions of the ase, the se ond
gives a lower bound for p2an , the third is always equal to pl , and the
fourth gives an upper bound for p2an+1 ; from these a ontradi tion
to the indu tion hypothesis will be ome evident.

qpr + 1
qp2an 1 +1 + 1
n even, l = 2r + 1
qp2an 1 + 1
q2 pr
q2 p2an 1 +1
n even, l = 4r
q2 p2an 2
q 2 pr + q
q2 p2an 2+1 + q
n even, l = 4r + 2
q2 p2an 2 + q
qpr
qp2an 1 +1
n odd, l = 2r
qp2an 1
q 2 pr + 1
q2 p2an 2 +1 + 1
n even, l = 4r + 1
q2 p2an 2 + 1
n even, l = 4r + 3 q2 p2an 2 + q + 1 q2 pr + q + 1 q2 p2an 2+1 + q + 1.

43

1.12 Poland
1. Find all pairs (n; r), with n a positive integer and r a real number,
for whi h the polynomial (x + 1)n r is divisible by 2x2 + 2x + 1.
Solution: Let t = ( 1 + i)=2 be one of the roots of 2x2 + 2x + 1;

then (x + 1)n r is divisible by 2x2 + 2x + 1 for r real if and only


if (t + 1)n = r. Sin e the argument of t + 1 is =4, this is possible
if and only if n = 4m, in whi h ase (t + 1)4 m = ( 4)m . Hen e
(4m; ( 4)m) are the only solutions.
2. Let ABC be a triangle and P a point inside it su h that \P BC =
\P CA < \P AB . The line P B uts the ir um ir le of ABC at B
and E , and the line CE uts the ir um ir le of AP E at E and F .
Show that the ratio of the area of the quadrilateral AP EF to the
area of the triangle ABP does not depend on the hoi e of P .
Solution: Note that \AEP = \AEB = \ACB = \CBP , so the

lines AE and CP are parallel. Thus [AP E = [ACE and [AP EF =


[ACF . Now note that \AF C =  \EP A = \AP B and \ACF =
\ACE = \ABE . Therefore triangles ACF and ABP are similar
and [ACF =[AB = (AC=AB )2 independent of the hoi e of P .
3. Let n  2 be a xed natural number and let a1 ; a2 ; : : : ; an be positive numbers whose sum is 1. Prove that for any positive numbers
x1 ; x2 ; : : : ; xn whose sum is 1,
n
X
ai x2i
n 2 X
;
+
2 xi xj 
n 1 i=1 1 ai
i<j
and determine when equality holds.
P

i xi ,
n
X

Solution: The left side is 1

result as

By Cau hy-S hwarz,


n
X

x2i
i=1 1 ai


n 1 i
!

so we an rewrite the desired

x2i
:
1 ai
=1
!

n
X
i=1

(1 ai )
44

n
X
i=1

!2

xi

= 1:

ai ) = n 1, we have the desired result.


4. Let ABCD be a tetrahedron with \BAC = \ACD and \ABD =
\BDC . Show that edges AB and CD have the same length.
Sin e

i (1

Solution: Assume AB 6= CD. Draw the plane through AC bise ting the dihedral angle formed by the planes ABC and ACD,
then draw a line ` in that plane perpendi ular to AC through the
midpoint O of AC . Now let B 0 and D0 be the images of B and D,
respe tively, under the half-turn around the line `; by assumption,
B 0 6= D and D0 6= B . Sin e \BAC = \ACD, B 0 lies on CD and
D0 lies on AB . Now note that the quadrilateral BB 0 D0 D has total angular sum 2. However, a nonplanar quadrilateral always has
total angular sum less than 2 (divide it into two triangles, whi h
ea h have angular sum , and apply the spheri al triangle inequality
\ABC + \CBD > \ABD), so the lines AB and CD are oplanar,
ontradi ting the assumption that ABCD is a tetrahedron.

5. For a natural number k, let p(k) denote the smallest prime number
whi h does not divide k. If p(k) > 2, de ne q(k) to be the produ t
of all primes less than p(k), otherwise let q(k) = 1. Consider the
sequen e
x p(x )
x0 = 1;
xn+1 = n n
n = 0; 1; 2; : : : :
q(xn )
Determine all natural numbers n su h that xn = 111111.
Solution: An easy indu tion shows that, if p0 ; p1 ; : : : are the primes

in in reasing order and n has base 2 representation 0 +2 1 +4 2 +  ,


then xn = p 00 p 11   . In parti ular, 111111 = 3  7  11  13  37 =
p1 p3 p4 p5 p10 , so xn = 111111 if and only if n = 210 +25 +24 +23 +21 =
1082.

6. From the set of all permutations f of f1; 2; : : : ; ng that satisfy the


ondition
f (i)  i 1
i = 1; 2; : : : ; n;
one is hosen uniformly at random. Let pn be the probability that
the hosen permutation f satis es

f (i)  i + 1

45

i = 1; 2; : : : ; n:

Find all natural numbers n su h that pn > 1=3.


We have pn > 1=3 for n  6. Let n be the number
of permutations of the rst type. For su h a permutation, either
f (1) = 1, or f (2) = 1. In the rst ase, ignoring 1 gives a valid permutation of f2; : : : ; ng; in the latter ase, we get a valid permutation
of f2; : : : ; ng by identifying 1 and 2 together. Hen e n = 2 n 1 and
so n = 2n 1 sin e 1 = 1.
Let dn be the number of permutations of the se ond type. For su h
a permutation, either f (n) = n or f (n) = n 1. In the rst ase,
ignoring n gives a valid permutation of f1; : : : ; n 1g. In the latter
ase, we must have f (n 1) = n, so ignoring n and n 1 gives a
valid permutation of f1; : : : ; n 2g. Thus dn = dn 1 + dn 2 , and
the initial onditions d1 = 1; d2 = 2 yield dn = Fn+1 , the n + 1-st
Fibona i number.
It is easily shown (using the formula for Fn or by indu tion) that
n =dn < 1=3 for n  7. Hen e the desired n are 1; : : : ; 6.

Solution:

46

1.13 Romania

1. Let n > 2 be an integer and f : R2 ! R be a fun tion su h that for


any regular n-gon A1 A2 : : : An ,
f (A1 ) + f (A2 ) +    + f (An ) = 0:
Prove that f is the zero fun tion.
We identify R2 with the omplex plane and let  =
Then the ondition is that for any z 2 C and any positive

Solution:
i=n .

e
real t,
2

n
X
j =1

f (z + t j ) = 0:

In parti ular, for ea h of k = 1; : : : ; n, we have


n
X
j =1

f (z  k +  j ) = 0:

Summing over k, we have

n X
n
X

m=1 k=1

f (z (1  m ) k ) = 0:

For m = n the inner sum is nf (z ); for other m, the inner sum again
runs over a regular polygon, hen e is 0. Thus f (z ) = 0 for all z 2 C .
2. Find the greatest positive integer n for whi h there exist n nonnegative integers x1 ; x2 ; : : : ; xn , not all zero, su h that for any sequen e
1; 2 ; : : : ; n of elements of f 1; 0; 1g, not all zero, n3 does not divide
1x1 + 2 x2 + : : : + n xn .
Solution: The statement holds for n = 9 by hoosing 1; 2; 22 ; : : : ; 28 ,

sin e in that ase


j1 +    + 9 28 j  1 + 2 +    + 28 < 93:
However, if n = 10, then 210 > 103, so by the pigeonhole prin iple,
there are two subsets A and B of fx1 ; : : : ; x10 g whose sums are ongruent modulo 103. Let i = 1 if xi o urs in A but
P not in B , 1 if
xi o urs in B but not in A, and 0 otherwise; then i xi is divisible
by n3 .
47

3. Let x; y be real numbers. Show that if the set

f os(nx) + os(ny)jn 2 N g
is nite, then x; y 2 Q .
Solution: Let an = os nx and bn = sin nx. Then

(an + bn )2 + (an

bn )2 = 2(a2n + b2n ) = 2 + (a2n + b2n ):

If fan + bn g is nite, it follows that fan bn g is also a nite set, and


hen e that fan g is nite, sin e
1
an = [(an + bn ) + (an bn);
2
and similarly fbn g is nite. In parti ular, am = an for some m < n,
and so (n m)x is an integral multiple of . We on lude x and y
are both rational.
4. Let ABCD be a y li quadrilateral and let M be the set of in enters
and ex enters of the triangles BCD; CDA; DAB; ABC (for a total
of 16 points). Show that there exist two sets of parallel lines K and
L, ea h onsisting of four lines, su h that any line of K [ L ontains
exa tly four points of M .
Solution: Let T be the midpoint of the ar AB of the ir um ir le

of ABC , I the in enter of ABC , and IB ; IC the ex enters of ABC


opposite B and C , respe tively. We rst show T I = T A = T B =
T IC . Note that

\T AI = \T AB +\BAI = (\C +\A)=2 = \ICA+\IAC = \T AI;


so T I = T A, and similarly T I = T B . Moreover, in the right triangle
AIC I , \AIC T = =2 \AIT = =2 \T AI = \T AIC , so T A =
T IC also.
We next show that the midpoint U of IB IC is also the midpoint of
the ar BAC . Note that the line IB IC bise ts the exterior angles of
ABC at A, so the line IB IC passes through the midpoint V of the
ar BAC . Considering the right triangles IB BIC and IB CIC , we
48

note BU = (IB IC )=2 = CU , so U lies on the perpendi ular bise tor


of BC , whi h su es to show U = V . (Note that IB and IC lie on
the same side of BC as A, so the same is true of U .)
Let E; F; G; H be the midpoints of the ar s AB; BC; CD; DA. Let
IA ; IB ; IC ; ID be the in enters of the triangles BCD; CDA; DAB ,
ABC , respe tively. Let AB ; AC ; AD be the ex enters of BCD opposite B; C; D, respe tively, and so on.
By the rst observation, IC ID CD DC is a re tangle with enter E ,
and the diagonals, whi h ontain the points C and D, have length
2EA = 2EB . Similarly, we obtain re tangles entered at F; G; H .
Now onsider the ex enters of the form XY , where X and Y are
opposite verti es in ABCD. We shall prove the laim with

K = fBC CB ; IC IB ; ID IA ; AD DA g; L = fAB BA ; IA IB ; IC ID ; CD DC g:
Consider the re tangle BC ID BA P , where P is an unknown point.
From the se ond observation above, the midpoint K of diagonal
BA BC is the midpoint of ar CDA, so it lies on the internal bise tor
BK of triangle ABC . Again by the rst observation, we on lude
M = DA , so DA lies on the lines BC CB and BA AB , and so on,
proving the laim.

5. Given a 2 R and f1 ; f2 ; : : : ; fn : R ! R additive fun tions su h that


f1(x)f2 (x)    fn (x) = axn for all x 2 R. Prove that there exists
b 2 R and i 2 f1; 2; : : : ; ng su h that fi (x) = bx for all x 2 R.
Solution: Let i = fi (1). Then for any integer x,
n
Y
i=1

fi (1 + mx) =

n
Y

[ i + mfi (x) = a(1 + mx)n :

i=1

First suppose a 6= 0, in whi h ase i 6= 0 for all i. Then we have an


equality of polynomials in T :
n
Y
i=1

[ i + fi (x)T = a(1 + xT )n ;

and so by unique fa torization, i + fi (x)T = bi (1+ xT ) for some real


number bi . Equating oe ients gives bi = i and fi (x) = bi x = i x
for all x.
49

Now suppose a = 0; we shall show that fi is identi ally zero for


some i. Assume on the ontrary that there exist ai for all i su h
that fi (ai ) 6= 0. Let

xm = a1 + ma2 +    + mn 1 am
for any integer m. Then
0=

n
Y
i=1

fi (xm ) =

n
Y
i=1

[fi (a1 ) + fi (a2 )m +    + fi (an )mn 1 :

Hen e for some i, the polynomial fi (a1 )+ fi(a2 )m +    + fi (an )mn 1


is identi ally zero, ontradi ting the fa t that fi (ai ) 6= 0. Thus for
some i, fi (x) = 0 for all x, proving the laim with b = 0.
6. The sequen e fangn2 is de ned as follows: if p1 ; p2 ; : : : ; pk are the
distin t prime divisors of n, then an = p1 1 + p2 1 + : : : + pk 1 . Show
that for any positive integer N  2,
N
X
n=2

a2 a3    an < 1:

Solution: It is easily seen that


n
X
k=2

ak =

n
X

k=2

 

X1 n
1
1
1
:
=
+ ++
p1 p2
pk
pn p p

On the other hand, we have the inequalities


 

1 n
pn p p
X

n
2
p
pn
!
1
1 X
1
< n +
4 k=1 (2k + 1)2
!
1
n X
n
1
<
= :
4 k=1 k(k + 1)
2

50

Thus
ity,

Pn

k=2 ak

< n=2 for all n  2. Now using the AM-GM inequal


a2 + a3 +    + an n 1
a2 a3    a n <
n 1


1 n 1
e
3
1
< n 1 < n 1:
< n 1 1+
2
n 1
2
2

Adding these inequalities,




1
X
1 1 1
1
1
1
a2    a n <
+ + + +3 5 + 6 +
2 6 12 60
2
2
n=2


1
46 6
46 3
1 + +    = + < 1:
+
=
60 25
2
60 32
7. Let n  3 be an integer and x1 ; x2 ; : : : ; xn
su h that

x1 + x2 + : : : + xn
x1 + 2x2 + : : : + (n 1)xn

1
1

nonnegative integers

= n
= 2n 2:

Find the minimum of the sum

F (x1 ; : : : ; xn 1 ) =

nX1
k=1

kxk (2n k):

Solution: The desired sum an be written as 2n(2n 2)

Now note
nX1
k=1

k2 xk =

nX1
k=1

xk +(k 1)(k+1)xk  n+n

k=1

Pn

k=1 k
1

xk .

n 1(k 1)xk = n+n(2n 2 n) = n2 n:

Hen e the quantity in question is at most 2n(2n 2) (n2 n) =


3n2 3n, with equality for x1 = n 1; x2 =    = xn 2 = 0; xn 1 = 1.
8. Let n; r be positive integers and A a set of latti e points in the plane,
su h that any open dis of radius r ontains a point of A. Show that
for any oloring of the points of A using n olors, there exist four
points of the same olor whi h are the verti es of a re tangle.
51

Solution: Consider a square of side length L = 4nr2 with sides par-

allel to the oordinate axes. One an draw (2nr)2 = 4n2r2 disjoint


disks of radius r inside the square, hen e su h a square ontains at
least 4n2 r2 points of A. The latti e points in A lie on L 1 = 4nr2 1
verti al lines; by the pigeonhole prin iple, some verti al line ontais
n + 1 points of A. Again by the pigeonhole prin iple, two of these
points are olored in the same olor.
Now onsider an in nite horizontal strip made of ribbons of side
length L; some two of them have two points in the same position in
the same olor, and these four points form the verti es of a re tangle.
9. Find all prime numbers p; q for whi h the ongruen e

3pq  (mod 3pq)


holds for all integers .
Solution: Without loss of generality assume p  q ; the unique solution will be (11; 17), for whi h one may he k the ongruen e using
the Chinese Remainder Theorem. We rst have 23pq  2 (mod 3),
whi h means p and q are odd. In addition, if is a primitive root
mod p, then 3pq 1  1 (mod p) implies that p 1 divides 3pq 1 as
well as 3pq 1 3q(p 1) = 3q 1, and onversely that q 1 divides
3p 1. If p = q, we now dedu e p = q = 3, but 427  1 (mod 27), so
this fails. Hen e p < q.
Sin e p and q are odd primes, q  p + 2, so (3p 1)=(q 1) < 3.
Sin e this quantity is an integer, and it is learly greater than 1, it
must be 2. That is, 2q = 3p + 1. On the other hand, p 1 divides
3q 1 = (9p + 1)=2 as well as (9p + 1) (9p 9) = 10. Hen e
p = 11; q = 17.

10. Let n  3 be an integer and p  2n 3 a prime. Let M be a


set of n points in the plane, no three ollinear, and let f : M !
f0; 1; : : : ; p 1g be a fun tion su h that:
(i) only one point of M maps to 0, and
(ii) if A; B; C are distin t points in M and k is the ir um ir le of
52

the triangle ABC , then


X

P 2M \k

f (P )  0 (mod p):

Show that all of the points of M lie on a ir le.


Solution: Let X be the point mapping to 0. We rst show that if
every ir le through X and two points of M ontains a third point
of M , then all of the points of M lie on a ir le. Indeed, onsider
an inversion with enter at X . Then the image of M fX g has
the property that the line through any two of its points ontains a
third point; it is a standard result that this means the points are
ollinear. (Otherwise, nd a triangle ABC minimizing the length of
the altitude AH ; there is another point N on BC , but then either
ABN or ACN has a shorter altitude than AH , ontradi tion.)
Now suppose the points of M do not lie on a ir le. By the above,
there exists a ir le passing through M and only two points A; B of
M . Let f (A) = i, so that by the hypothesis, f (B ) = p i. Let a be
the number of ir les passing through X; A and at least one other
point of M , let b be the number of ir les passing through X; B
and at least one other point of M , and let S be the sum of f (P )
over all P in M . By adding the relations obtained from the ir les
through X and A, we get S + (a 1)i  0 (mod p), and similarly,
S + (b 1)(p i)  0 (mod p). Therefore a + b 2  0 (mod p); sin e
a + b  2n +4 < p, we have a + b = 2 and so a = b = 1, ontradi ting
the assumption that the points do not all lie on a ir le.

11. Let x1 ; x2 ; : : : ; xn ; xn+1 be positive reals su h that x1 +x2 +  +xn =


xn+1 . Prove that
n p
X
i=1

xi (xn+i

xi ) 

v
u n
uX
t
xn+1 (xn+1

i=1

xi ):

Solution: First note that


n
X
i=1

xn+1 (xn+1

xi ) = nx2n+1 xn+1
53

n
X
i=1

xi = (n 1)xn+1 :

Hen e the given inequality may be rewritten as


n
X

xi

n 1 xn+1

i=1

xi

xn+1

1

On the other hand, by the arithmeti -geometry mean inequality, the


left side is at most
n
X

1 xi =xn+1 1 (n 1)
xi
+
= +
= 1:
2
x
2(n 1)
2 2(n 1)
i=1 n+i

12. Let x; y; z be real numbers. Prove that the following onditions are
equivalent.
(i) x; y; z > 0 and x1 + y1 + 1z  1.
(ii) For every quadrilateral with sides a; b; ; d, a2 x + b2y + 2z > d2 .
Solution: To show (i) implies (ii), note that

a x+b y+ z
2




1 1 1
(a x + b y + z )
+ +
x y x
(a + b + )2 > d2 ;
2

using Cau hy-S hwarz after the rst inequality.


To show (i) implies (ii), rst note that if x  0, we may take a
quadrilateral of sides a = n; b = 1; = 1; d = n and get y + z >
n2 (1 x), a ontradi tion for large n. Thus x > 0 and similarly
y > 0; z > 0. Now use a quadrilateral of sides 1=x; 1=y; 1=z and
1=x + 1=y + 1=z 1=n, where n is large. We then get


y
z
1 1 1
x
+ + >
+ +
x2 y2 z 2
x y z

1
n

2

Sin e this holds for all n, we may take the limit as n ! 1 and get
1 1 1
+ +
x y z

and hen e 1=x + 1=y + 1=z  1.


54

1 1 1
+ +
x y z

1
n

2

13. Let n be a positive integer and D a set of n on entri ir les


in the plane. Prove that if the fun tion f : D ! D satis es
d(f (A); f (B ))  d(A; B ) for all A; B 2 D, then d(f (A); f (B )) =
d(A; B ) for every A; B 2 D.
Solution: Label the ir les D1 ; : : : ; Dn in in reasing order of radius, and let ri denote the radius of Di . Clearly the maximum of
d(A; B ) o urs when A and B are antipodal points on D. Let ABCD
be the verti es of a square ins ribed in Dn ; then f (A) and f (C ) are
antipodal, as are f (B ) and f (D). In addition, ea h of the minor ar s
f (A)f (B ) and f (B )f (C ) must be at least a quarter ar , thus f (B )
bise ts one of the semi ir les bounded by f (A) and f (C ), and f (D)
bise ts the other. Now if P is any point on the minor ar AB , then
the ar s f (P )f (A) and f (P )f (B ), whi h are at least as long as the
ar s P A and P B , add up to the quarter ar f (P )f (B ). We on lude
f is isometri on Dn .
Sin e f is learly inje tive and is now bije tive on Dn , f maps D1 [
: : : [ Dn 1 into itself. Thus we may repeat the argument to show
that f is isometri on ea h Di . To on lude, it su es to show that
distan es between adja ent ir les, say D1 and D2 , are preserved.
This is easy; hoose a square ABCD on D1 and let A0 ; B 0 ; C 0 ; D0 be
the points on D2 losest to A; B; C; D, respe tively. Then A0 B 0 C 0 D0
also form a square, and the distan e from A to C 0 is the maximum
between any point on D1 and any point on D3 . Hen e the eight
points maintain their relative position under f , whi h su es to
prove isometry.
14. Let n  3 be an integer and X  f1; 2; : : : ; n3 g a set of 3n2 elements.
Prove that one an nd nine distin t numbers a1 ; : : : ; a9 in X su h
that the system
a1 x + a2 y + a3 z = 0
a4 x + a5 y + a6 z = 0
a7 x + a8 y + a9 z = 0
has a solution (x0 ; y0 ; z0) in nonzero integers.
Solution: Label the elements of X in in reasing order x1 <    <
x3n2 , and put
X1 = fx1 ; : : : ; xn2 g; X2 = fxn2 +1 ; : : : ; x2n2 g; X3 = fx2n2 +1 ; : : : ; x3n2 g:

55

De ne the fun tion f : X1  X2  X3 ! X  X as follows:

f (a; b; ) = (b a; b):
The domain of f ontains n6 elements. The range of f , on the other
hand, is ontained in the subset of X  X of pairs whose sum is at
most n3 , a set of ardinality
3 1
nX

k=1

k=

n3 (n3 1) n6
< :
2
2

By the pigeonhole prin iple, some three triples (ai ; bi ; i ) (i = 1; 2; 3)


map to the same pair, in whi h ase x = b1 1 ; y = 1 a1 ; z =
a1 b1 is a solution in nonzero integers. Note that ai annot equal bj
sin e X1 and X2 and so on, and that a1 = a2 implies that the triples
(a1 ; b1 ; 1 ) and (a2 ; b2 ; 2 ) are identi al, a ontradi tion. Hen e the
nine numbers hosen are indeed distin t.

56

1.14 Russia
1. Whi h are there more of among the natural numbers from 1 to
1000000, in lusive: numbers that an be represented as the sum
of a perfe t square and a (positive) perfe t ube, or numbers that
annot be?
There are more numbers not of this form. Let n =
k2 + m3 , where k; m; n 2 N and n  1000000. Clearly k  1000 and
m  100. Therefore there annot be more numbers in the desired
form than the 100000 pairs (k; m).

Solution:

2. The enters O1 ; O2 ; O3 of three noninterse ting ir les of equal radius


are positioned at the verti es of a triangle. From ea h of the points
O1 ; O2 ; O3 one draws tangents to the other two given ir les. It is
known that the interse tion of these tangents form a onvex hexagon.
The sides of the hexagon are alternately olored red and blue. Prove
that the sum of the lengths of the red sides equals the sum of the
lengths of the blue sides.
Solution: Let A; B; C; D; E; F be the verti es of the hexagon in
order, with A on the tangents to O1 , C on the tangents to O2 , and
E on the tangents to O3 . Sin e the given ir les have equal radius,

X1 O2 = O1 Y2 ; Y1 O3 = O2 Z2; Z1 O1 = O3 X2
or

X1 A + AB + BO2 = O1 B + BC + CY2 ;
Y1 C + CD + DO3 = O2 D + DE + EZ2 ;
Z1 E + EF + F O1 = O3 F + F A + AX2 :
Adding these equations and noting that

X1 A = AX2 ; Y1 C = CY2 ; Z1 E = EZ2 ;


(as these segments are tangents to a ir le from a single point) and

BO2 = O1 B; DO3 = O2 D; F O1 = O3 F
57

(sin e the ir les have equal radii), we get

AB + CD + EF = BC + DE + F A;
as desired.
Note: The analogous statement is also true in the ase where the
hexagon has re ex angles at B; D; F . In both ases, we also have
the equality AB  CD  EF = BC  DE  F A, or equivalently, the lines
AD; BE; CF on ur. Moreover, the latter statement remains true
even if the assumption of equal radii is removed, and this fa t leads
to a proof of Brian hon's Theorem.

3. Let x; y; p; n; k be natural numbers su h that

xn + yn = pk :
Prove that if n > 1 is odd, and p is an odd prime, then n is a power
of p.
Let m = g d(x; y). Then x = mx1 ; y = my1 and by
virtue of the given equation, mn (xn1 + y1n) = pk , and so m = p for
some nonnegative integer . It follows that

Solution:


xn1 + y1n = pk n :

(1)

Sin e n is odd,

xn1 + y1n
= x1n
x1 + y1

x1n 2 y1 + x1n 3 y12

   x yn
1 1

+ y1n 1:

Let A denote the right side of the equation. By the ondition p > 2,
it follows that at least one of x1 ; y1 is greater than 1, so sin e n > 1,
A > 1.

From (1) it follows that A(x1 + y1 ) = pk n , so sin e x1 + y1 > 1,
and A > 1, both of these numbers are divisible by p; moreover,
x1 + y1 = p for some natural number . Thus

A = x1n 1 x1n 2 (p
= nx1n 1 + Bp:

x1 ) +    x1 (p
58

x1 )n

+ (p

x1 )n

Sin e A is divisible by p and x1 is relatively prime to p, it follows


that n is divisible by p.
Let n = pq. Then xpq + ypq = pk or (xp )q +(yp )q = pk . If q > 1, then
by the same argument, p divides q. If q = 1, then n = p. Repeating
this argument, we dedu e that n = p` for some natural number `.
4. In the Duma there are 1600 delegates, who have formed 16000 ommittees of 80 persons ea h. Prove that one an nd two ommittees
having no fewer than four ommon members.
First Solution: Suppose any two ommittees have at most three
ommon members. Have two deputies ount the possible ways to
hoose a hairman for ea h of three sessions of the Duma. The rst
deputy assumes that any deputy an hair any session, and so gets
16003 possible hoi es. The se ond deputy makes the additional restri tion that all of the hairmen belong to a single ommittee. Ea h
of the 16000 ommittees yields 803 hoi es, but this is an over ount;
ea h of the 16000(16000 1)=2 pairs of ommittees give at most 33
overlapping hoi es. Sin e the rst deputy ounts no fewer possibilities than the se ond, we have the inequality

16003  16000  803

16000  15999 3
3:
2

However,
16000  803

16000  15999 42
16000  15999 3
3 > 16000  803
2
2
2
16000  43
+ 213  106 212  106
=
4
> 212  106 = 16003:

We have a ontradi tion.


Se ond Solution: Suppose we have N ommittees su h that no

two have more than three ommon members. For ea h deputy we


write down all of the unordered pairs of ommittees she belongs to.
If a person deputy to K ommittees, she gives rise to K (K 1)=2
pairs.
59

Let K1 ; : : : ; K1600 be the number of ommittees that deputies 1; : : : ; 1600


belong to (under some labeling of the deputies). The total number
of pairs written down is
K1(K1 1)
K (K
1)
+ : : : + 1600 1600
2
2
2
K1 + : : : + K1600
K12 + : : : + K1600
=
2
2


1 (K1 + : : : + K1600 )2
 2
(K1 + : : : + K1600 )
1600


1 (80N )2
1
=
80N = N (4N 80)
2 1600
2
sin e K1 + : : : + K1600 = 80N .
Sin e no two ommittees have more than three ommon members,
the total number of pairs written annot ex eed 3N (N 1)=2. Hen e
N (4N 80)=2  3N (N 1)=2, i.e. N  77. In parti ular, if N =
16000, this annot be the ase.
5. Show that in the arithmeti progression with rst term 1 and ratio
729, there are in nitely many powers of 10.
Solution: We will show that for all natural numbers n, 1081n

1
is divisible by 729. In fa t, 1081n 1 = (1081)n 1n = (1081 1)  A,
and
1081

1 = 9| :{z: : 9}
81

= 9| :{z: : 9}    10
: : 01} 10
: : 01} : : : 10
: : 01}
| :{z
| :{z
| :{z
9

= 9 |1 :{z: : 1}    10
: : 01} 10
: : 01} : : : 10
: : 01} :
| :{z
| :{z
| :{z
9

The se ond and third fa tors are omposed of 9 units, so the sum of
their digits is divisible by 9, that is, ea h is a multiple of 9. Hen e
1081 1 is divisible by 93 = 729, as is 1081n 1 for any n.
6. In the isos eles triangle ABC (AC = BC ) point O is the ir um enter, I the in enter, and D lies on BC so that lines OD and BI are
perpendi ular. Prove that ID and AC are parallel.
60

First Solution: If the given triangle is equilateral (i.e. O = I ) the

statement is obvious. Otherwise, suppose O lies between I and C .


Draw the altitude CE and note that
\EIB = 90 21 \ABC and \ODB = 90 21 \ABC;
so \OIB + \ODB = 180, that is, the points B; I; O; D lie on a
ir le. Thus \IDB = \IOB (both angles are ins ribed in ar IB ),
but \IOB = 21 \AOB = \ACB . Therefore \IDB = \ACB , and
so IDjjAC . The argument is similar in the ases where I lies between
O and C .
Se ond Solution: Extend the angle bise tor BI to meet the ir um ir le at E . Next extend the line ED to meet the ir um ir le at
F . Let G and K be the interse tions of ED with AF and the altitude
CH , respe tively. The line OD ontains the diameter perpendi ular
to EB , and so DE = DB , i.e. the triangle EDB is isos eles and
\DEB = \DBE . But then \DEB = \ABE , hen e EF jjAB and
EF ? CI . By ins ribed angles,

\CEF = \IEF = \CF E = \IF E;


so ECF I is a rhombus. Thus CK = KI , and (by the symmetry
of G and D a ross CH ) GK = KD. This means GKDI is also a
rhombus and CGjjDI .
7. Two piles of oins lie on a table. It is known that the sum of the
weights of the oins in the two piles are equal, and for any natural
number k, not ex eeding the number of oins in either pile, the sum
of the weights of the k heaviest oins in the rst pile is not more
than that of the se ond pile. Show that for any natural number x,
if ea h oin (in either pile) of weight not less than x is repla ed by a
oin of weight x, the rst pile will not be lighter than the se ond.
Solution: Let the rst pile have n oins of weights x  x 
    xn , and let the se ond pile have m oins of weights y 
y      ym , where x      xs  x  xs      xn
and y      yt  x  yt      ym . (If there are no
1

+1

+1

61

oins of weight greater than x, the result is lear.) We need to


show that xs + xs+1 +    + xn  xt + yt+1 +    + ym . Sin e
x1 +    + xn = y1 + : : : + ym = A, this inequality an be equivalently
written

xs + (A x1

   xm )  xt + (A y

: : : yt );

whi h in turn an be rewritten

x1 + : : : + xs + x(t s)  y1 + : : : + yt ;
whi h is what we will prove.
If t  s, then

x1 + : : : + xs + x(t s) = (x1 + : : : + xs ) + (x +    + x)
|

 (y

{z

t s

+ : : : + ys ) + (ys+1 + : : : + yt );

sin e x1 + : : : + xs  y1 + : : : + ys (from the given ondition) and


ys+1  : : :  yt  x.
If t < s, then x1 + : : : + xs + x(t s)  y1 + : : : + yt is equivalent to

x1 + : : : + xs  y1 + : : : + yt + (x + : : : + x) :
|

{z

t s

The latter inequality follows from the fa t that

x1 + : : : + xs  y1 + : : : + ys = (y1 + : : : + yt ) + (yt+1 + : : : + ys )
and ys  : : :  yt+1  x.

8. Can a 5  7 he kerboard be overed by L's ( gures formed from a


2  2 square by removing one of its four 1  1 orners), not rossing its
borders, in several layers so that ea h square of the board is overed
by the same number of L's?
No su h overing exists. Suppose we are given
a overing of a 5  7 he kerboard with L's, su h that every ell is
overed by exa tly k L's. Number the rows 1; : : : ; 5 and the olumns
1; : : : ; 7, and onsider the 12 squares lying at the interse tions of oddnumbered rows with odd-numbered olumns. Ea h of these ells is

First Solution:

62

overd by k L's, so at least 12k L's must be used in total. But these
over 3  12k > 35k ells in total, a ontradi tion.
Se ond Solution: Color the ells of the he kerboard alternately

bla k and white, so that the orners are all bla k. In ea h bla k
square we write the number 2, and in ea h white square 1. Note
that the sum of the numbers in the ells overed by ea h L is nonnegative, and onsequently if we are given a overing of the board
in k layers, the sum over ea h L of the numbers overed by that
L is nonnegative. But if this number is S and s is the sum of the
numbers on the board, then

S = ks = k( 2  12 + 23  1) = k < 0:
We have a ontradi tion.
Note: It is proved analogously that a overing of the desired form

does not exist if the he kerboard has dimensions 3  (2n + 1) or


5  5. The 2  3 board an be overed by one layer of two L's, the
5  9 by one layer of 15 L's, and the 2  2 by three layers using four
L's. Combining these three overings, it is not hard to show that all
remaining m  n boards (m; n  2) an be overed.
9. Points E and F are given on side BC of onvex quadrilateral ABCD
(with E loser than F to B ). It is known that \BAE = \CDF and
\EAF = \F DE . Prove that \F AC = \EDB .
Solution: By the equality of angles EAF and F DE , the quadri-

lateral AEF D is y li . Therefore \AEF + \F DA = 180. By the


equality of angles BAE and CDF we have
\ADC + \ABC = \F DA + \CDF + \AEF \BAE = 180:
Hen e the quadrilateral ABCD is y li , so \BAC = \BDC . It
follows that \F AC = \EDB .

10. On a oordinate plane are pla ed four ounters, ea h of whose enters


has integer oordinates. One an displa e any ounter by the ve tor
joining the enters of two of the other ounters. Prove that any two
63

presele ted ounters an be made to oin ide by a nite sequen e of


moves.
Solution:
Lemma 1

If three ounters lie on a line and have integer oordinates, then we an make any two of them oin ide.

Proof: Let A and B be the ounters between whi h the smallest


of the three pairwise distan es o urs, and let C be the other one.
By repeatedly moving C either by the ve tor AB or its reverse, we
an put C on the segment AB , thus de reasing the minimum of the
pairwise distan es. Sin e the points have integer oordinates, repeating this pro ess must eventually bring the minimum distan e down
to zero. If the desired ounters oin ide, we are done; otherwise,
the one that oin ides with the third ounter an be moved to the
lo ation of the other one. 2
Proje t the ounters onto one of the axes. The proje tions behave
like ounters, in that if a ounter is displa ed by a ve tor, its proje tion is displa ed by the proje tion of the ve tor. As des ribed
in the lemma, we an make the proje tions of our hosen ounters
oin ide, using one of the remaining ounters as the third ounter.
We an now make a third proje tion oin ide with these by treating
our hosen ounters as one. (That is, ea h time we displa e one,
we displa e the other by the same amount.) Now our two hosen
ounters and one more lie on a line perpendi ular to the axis, and
by the lemma we an make the desired ounters oin ide.
11. Find all natural numbers n, su h that there exist relatively prime
integers x and y and an integer k > 1 satisfying the equation 3n =
xk + yk .

Solution: The only solution is n = 2.


Let 3n = xk + yk , where x; y are relatively prime integers with x > y,
k > 1, and n a natural number. Clearly neither x nor y is a multiple
of 3. Therefore, if k is even, xk and yk are ongruent to 1 mod 3, so
their sum is ongruent to 2 mod 3, and so is not a power of 3.
If k is odd and k > 1, then 3n = (x + y)(xk 1 : : : + yk 1 ). Thus
x + y = 3m for some m  1. We will show that n  2m. Sin e 3jk

64

(see the solution to Russia 3), by putting x1 = xk=3 and y1 = yk=3


we may assume k = 3. Then x3 + y3 = 3m and x + y = 3n. To prove
the inequality n  2m, it su es to show that x3 + y3  (x + y)2 , or
x2 xy + y2  x + y. Sin e x  y + 1, x2 x = x(x 1)  xy, and
(x2 x + xy) + (y2 y)  y(y 1)  0, and the inequality n  2m
follows.
From the identity (x + y)3 (x3 + y3 ) = 3xy(x + y) it follows that
32m

3n m

= xy:

But 2m 1  1, and n m 1  n 2m  0. If stri t inequality


o urs in either pla e in the last inequality, then 32m 1 3n m 1 is
divisible by 3 while xy is not. Hen e n m 1 = n 2m = 0, and
so m = 1; n = 2 and 32 = 23 + 13 .
The inequality x2
shown by noting that

Note:

x2

xy + y2

xy + y2

 x + y an alternatively be

x y = (x y)2 + (x 1)(y 1) 1  0;

sin e (x y)2  1.
12. Show that if the integers a1 ; : : : ; am are nonzero and for ea h k =
0; 1; : : : ; m (n < m 1),

a1 + a2 2k + a3 3k + : : : + am mk = 0;
then the sequen e a1 ; : : : ; am ontains at least n + 1 pairs of onse utive terms having opposite signs.
Solution: We may assume am > 0, sin e otherwise we may multiply ea h of P
the numbers by 1. Consider the sequen e b1 ; : : : ; bm,
where bi = nj=0 j ij for an arbitrary sequen e of real numbers
0 ; : : : ; n . From the given ondition
m
X
i=1

ai bi =

m
X
i=1

ai

n
X
j =0

ij

j =

n
X
j =0

n
X
i=1

ai ij = 0:

Suppose now that the sequen e a1 ; : : : ; am has k pairs of neighbors


that di er in sign, where k < n + 1, and let i1 ; : : : ; ik be the indi es
65

of the rst members of these pairs. Let bi = f (i) = (i x1 )(i


x2 ) : : : (i xk ), where x` = i` + 1=2 (i = 1; 2; : : : ; k). The fun tion
f hanges sign only at the points x1 ; : : : ; xk , and so bi and bi+1
have di erent signs if and only one of the x` falls between them,
whi h means i = i` . We dedu e that the sequen es a1 ; : : : ; am and
b1; : : : ; bm have the same pairs of neighbors of opposite sign. Sin e
am and bm are positive, P
we have that ai and bi have the same sign
for i = 1; : : : ; m, and so m
i=1 ai bi > 0, a ontradi tion.
13. At the verti es of a ube are written eight pairwise distin t natural
numbers, and on ea h of its edges is written the greatest ommon
divisor of the numbers at the endpoints of the edge. Can the sum of
the numbers written at the verti es be the same as the sum of the
numbers written at the edges?
Solution: This is not possible. Note that if a and b are natural
numbers with a > b, then g d(a; b)  b and g d(a; b)  a=2. It
follows that if a 6= b, then g d(a; b)  (a + b)=3. Adding 12 su h
inequalities, orresponding to the 12 edges, we nd that the desired
ondition is only possible if g d(a; b) = (a + b)=3 in ea h ase. But in
this ase the larger of a and b is twi e the smaller; suppose a = 2b.
Consider the numbers and d assigned to the verti es of the other
endpoints of the other two edges oming out of the vertex labeled a.
Ea h of these is either half of or twi e a. If at least one is less than
a, it equals b; otherwise, both are equal. Either option ontradi ts
the assumption that the numbers are distin t.

14. Three sergeants and several solders serve in a platoon. The sergeants
take turns on duty. The ommander has given the following orders:
(a) Ea h day, at least one task must be issued to a soldier.
(b) No soldier may have more than two task or re eive more than
one tasks in a single day.
( ) The lists of soldiers re eiving tasks for two di erent days must
not be the same.
(d) The rst sergeant violating any of these orders will be jailed.
Can at least one of the sergeants, without onspiring with the others,
give tasks a ording to these rules and avoid being jailed?
66

Solution: The sergeant who goes third an avoid going to jail.


We all a sequen e of duties by the rst, se ond and third sergeants
in su ession a round. To avoid going to jail, the third sergeant on
the last day of ea h round gives tasks to pre isely those soldiers who
re eived one task over the previous two days. (Su h soldiers exist by
the third ondition.) With this strategy, at the end of ea h y le ea h
soldier will have re eived either two tasks or none, and the number
of the latter will have de reased. It will end up, at some point, that
all of the soldiers have re eived two tasks, and the rst sergeant will
go to jail.

15. A onvex polygon is given, no two of whose sides are parallel. For
ea h side we onsider the angle the side subtends at the vertex farthest from the side. Show that the sum of these angles equals 180.
Denote by Pa the vertex of the polygon farthest from
the line ontaining side a. Choose an arbitrary point O in the plane.
We all the two verti al angles, onsisting of all lines through O
and parallel to the segment Pa Q for some Q on side a, the angles
orresponding to side a.
We prove rst that the angles orresponding to di erent sides do
not overlap. Let a ray ` with vertex O lie inside one of the angles
orresponding to a. The line parallel to this ray passing through
Pa interse ts side a at some interior point A. Draw through Pa the
line b parallel to the line ontaining side a. From the onvexity
of the polygon and the de nition of Pa , it follows that the polygon
lies in the strip bounded by b and . Moreover, sin e the polygon
has no parallel sides, Pa is the only vertex of the polygon lying
on b. Therefore the segment Pa A is stri tly longer than any other
segment formed as the interse tion of the polygon with a line parallel
to `. If ` lay inside the angle orresponding to another side b, then
ontrary to this on lusion, the longest su h segment would be Pb B
for some B , and hen e this annot o ur. In other words, the angles
orresponding to a and b do not overlap.
We now prove that the angles we have onstru ted over the entire
plane. Suppose this were not the ase. Then there would exist some
angle with vertex O not overed by any of the angles onstru ted.
Solution:

67

Choose within this angle a ray m, not parallel to any side or diagonal
of the polygon. Of all of the segments formed by interse ting the
polygon with a line parallel to m, hoose the one of maximum length.
Clearly one of its verti es must be a vertex P of the polygon, while
the other lies on some side a. Draw the line through P parallel to
the line b ontaining a. If one of the sides adja ent to P did not lie
inside the strip bounded by b and , then we ould have found a line
parallel to m interse ting the polygon in a segment longer than P A.
Consequently, our polygon lies within the strip bounded by b and ,
from whi h we dedu e that P is the farthest vertex from the line b
ontaining side a. This means m lies in the angle orresponding to
a, ontradi ting our hoi e of m.
We thus on lude that our onstru ted angles over the plane without overlap, and hen e the sum of their measures is 360. To nish
the proof, simply note that the sum of the desired angles is half that
of the onstru ted angles.
16. Goodnik writes 10 numbers on the board, then Nogoodnik writes 10
more numbers, all 20 of the numbers being positive and distin t. Can
Goodnik hoose his 10 numbers so that no matter what Nogoodnik
writes, he an form 10 quadrati trinomials of the form x2 + px + q,
whose oe ients p and q run through all of the numbers written,
su h that the real roots of these trinomials omprise exa tly 11 values?
Solution: We will prove that Goodnik an hoose the numbers

1=4; 1=2; 1; 2; 5; 52; 54; 58 ; 516 ; 532:


(a) If a > 4 and a
has two distin t real roots.

Lemma 1

> b, then the

trinomial

x2 + ax + b

a < 4 and b > 0, then at least one of the trinomials x2 + ax +


b; x2 + bx + a does not have real roots.

(b) If

The rst part is obvious, sin e the dis riminant D = a2


4b > 4a 4b > 0. For the se ond part, note that if b  a, then
b2 4a < 0, while if b > a, then a2 4b < 0. 2
Proof:

68

Suppose 0 < a < b < < d and both of the trinomials


and x2 + x + b have two real roots. Then all four of
these roots are distin t.

Lemma 2

x2 + dx + a

Suppose the ontrary, that these trinomials have a ommon


root x0 . Then x20 + dx0 + a = 0 = x20 + x0 + b and onsequently
x0 = (b a)=(d ) > 0. But if x0 > 0, then x20 + dx0 + a > 0, a
ontradi tion. 2
Suppose Goodnik has written the aforementioned numbers. Consider all of Nogoodnik's numbers whi h are greater than 4. If there
are an odd number of them, add to them any of Nogoodnik's other
numbers. Call these numbers distinguished.
Add to the distinguished numbers members of the set f5; 52; 54, 58,
516; 532 g so that the total number of distinguished numbers is 12;
if the powers of 5 do not su e, add any of Nogoodnik's remaining
numbers to make a total of 12. From the unused powers of 5 make
trinomials x2 + px + q with p < q, whi h have negative dis riminant
and hen e no real roots.
Let n1 ; : : : ; n12 be the 12 distinguished numbers in in reasing order.
Now form from them the 6 trinomials x2 +n12 x+n1; : : : ; x2 +n7 x+n6.
By the onstru tion of the 12 distinguished numbers, at least 6 are
greater than 4. Hen e by Lemma 1, ea h of these trinomials has two
distin t real roots. By Lemma 2, all of these roots are distin t. Hen e
we have 12 distin t real roots of the \distinguished" trinomials.
Consider the trinomial x2 + 2x + 1, whose unique root is 1. If
this number o urs among the roots of the distinguished trinomials,
we de lare the orresponding trinomial \bad". If not, de lare an
arbitrary distinguised trinomial to be bad. Remove the bad trinomial, and from its oe ients and the numbers 1=2 and 1=4 form
(by Lemma 1) two trinomials without real roots. Now the number
of distin t real roots of the trinomials onstru ted so far is 11.
There may be some of Nogoodnik's numbers left; all ex ept possibly
one must be less than 4 (one may equal 4). By Lemma 1, we form
trinomials from these with no real roots.
Proof:

17. Can the number obtained by writing the numbers from 1 to n in


order (n > 1) be the same when read left-to-right and right-to-left?
69

Solution: This is not possible. Suppose N = 123    321 is an m-

digit symmetri number, formed by writing the numbers from 1 to


n in su ession. Clearly m > 18. Also let A and B be the numbers
formed from the rst and last k digits, respe tively, of N , where
k = bm=2 . Then if 10p is the largest power of 10 dividing A, then
n < 2  10p+1 , that is, n has at most p + 2 digits. Moreover, A and
B must ontain the fragments

: : : 0} 1 and 1 00
: : : 0} 1 99
: : : 9};
99
: : : 9} 1 00
| {z
| {z
| {z
| {z
p

respe tively, whi h is impossible.


18. Several hikers travel at xed speeds along a straight road. It is known
that over some period of time, the sum of their pairwise distan es
is monotoni ally de reasing. Show that there is a hiker, the sum of
whose distan es to the other hikers is monotoni ally de reasing over
the same period.
Solution: Let n be the number of hikers, who we denote P1 ; : : : ; Pn .
Let Vij be the rate of approa h between Pi and Pj (this is negative
if they are getting further apart). Note that Vij never in reases, and
an only de rease on e: it hanges sign if Pi and Pj meet.
By the given ondition, at the end of the period in question the sum
of the pairwise speeds must be positive:
X
1

i<jn

Vij > 0:

Sin e Vij = Vji , we have (putting Vii = 0)


n X
n
X
j =1 i=1

Vij = 2
1

i<jn

Vij > 0:

Hen e for some j , ni=1 Vij > 0. Sin e Vij annot in rease over time,
the sum of the distan es from Pj to the other hikers is de reasing
throughout the period.
70

19. Show that for n  5, a ross-se tion of a pyramid whose base is a


regular n-gon annot be a regular (n + 1)-gon.
Solution: Suppose the regular (n + 1)-gon B1 : : : Bn+1 is a ross-

se tion of the pyramid SA1 : : : An , whose base A1 : : : An is a regular


n-gon. We onsider three ases: n = 5, n = 2k 1 (k > 3) and
n = 2k (k > 2).
Sin e the pyramid has n + 1 fa es, one side of the se tion must lie
on ea h fa e. Therefore without loss of generality, we may assume
that the points B1 ; : : : ; Bn+1 lie on the edges of the pyramid.

(a) n = 5. Sin e in the regular hexagon B1 : : : B6 the lines B2 B3 ,


B5 B6 and B1 B4 are parallel, while the planes A2 SA3 and A1 SA5
pass through B2 B3 and B5 B6 , respe tively, the line ST (T =
A1 A5 \ A2 A3 ) along whi h these planes meet is parallel to these
lines, i.e. ST jjB1B4 . Draw the plane ontaining ST and B1 B4 .
This plane interse ts the plane of the base of the pyramid in
the line B1 A4 , whi h must pass through the interse tion of the
line ST with the plane of the base, that is, through T . Hen e
the lines A1 A5 ; A4 B1 and A2 A3 pass through a single point.
It is proved analogously that the lines A1 A2 ; A3 B6 and A4 A5
also meet in a point. From this it follows that A4 B1 and A3 B6
are axes of symmetry of the regular pentagon A1 : : : A5 , whi h
means their interse tion O is the enter of this pentagon. Now
note that if Q is the enter of the regular hexagon B1 : : : B6 ,
then the planes SA3 B6 , SA4 B1 and SB2 B5 interse t in the
line SQ. Consequently, the lines A3 B6 ; A4 B1 and A2 A5 must
interse t in a point, namely the interse tion of line SQ with the
plane of the base. This means the diagonal A2 A5 of the regular
pentagon A1 : : : A5 must pass through its enter O, whi h is
impossible.
(b) n = 2k 1 (k > 3). Analogously to the rst ase one shows that
sin e in the regular 2k-gon B1 : : : B2k the lines B1 B2 ; Bk+1 Bk+2 ,
and Bk Bk+3 are parallel, then the lines A1 A2 ; Ak+1 Ak+2 and
Ak Ak+3 must interse t in a point, whi h is impossible, sin e in
the regular (2k 1)-gon A1 : : : A2k 1 , Ak+1 Ak+2 jjAk Ak+3 , but
the lines A1 A2 and Ak+1 Ak+2 are not parallel.
( ) n = 2k (k > 2). Analogously to the pre eding ases, the lines
71

A1 A2 ; Ak+1 Ak+2 and Ak Ak+3 are parallel, and hen e the lines
B1 B2 ; Bk+1 Bk+2 and Bk Bk+3 must meet in a point, whi h is
impossible, sin e Bk+1 Bk+2 jjBk Bk+3 , while the lines B1 B2 and
Bk+1 Bk+2 are not parallel.
Note: For n = 3; 4, the statement of the problem is not true. For

examples, onsider a regular tetrahedron having a square as a rossse tion, and a square pyramid whose lateral fa es are equilateral
triangles, whi h has a regular pentagon as a ross-se tion.
Also, the presented solution may be more on isely expressed using
entral proje tion, and the property that under entral proje tion,
the images of lines passing through a single point (or parallel) are
lines passing through a single point (or parallel). It su es to proje t
the ross-se tion of the pyramid onto the plane of the base with
enter the vertex of the pyramid.

20. Do there exist three natural numbers greater than 1, su h that the
square of ea h, minus one, is divisible by ea h of the others?
Solution: Su h integers do not exist. Suppose a  b  satisfy the

desired ondition. Sin e a2 1 is divisible by b, the numbers a and


b are relatively prime. Hen e the number 2 1, whi h is divisible
by a and b, must be a multiple of ab, so in parti ular 2 1  ab.
But a  and b  , so ab  2 , a ontradi tion.

21. In isos eles triangle ABC (AB = BC ) one draws the angle bise tor
CD. The perpendi ular to CD through the enter of the ir um ir le
of ABC interse ts BC at E . The parallel to CD through E meets
AB at F . Show that BE = F D.
Solution: We use dire ted angles modulo  . Let O be the ir um ir le of ABC , and K the interse tion of BO and CD. From the
equality of the a ute angles BOE and DCA having perpendi ular
sides, it follows that \BOE = \KCE (CD being an angle bise tor), whi h means the points K; O; E; C lie on a ir le. From this it
follows that \OKE = \OCE ; but \OCE = \OBE , so OB = OC ,
and hen e \BKE = \KBE , or in other words BE = KE . Moreover, \BKE = \KBE = \KBA, and so KE jjAB . Consequently,

72

F EKD is a parallelogram and DF = KE . Therefore, DF = KE =


BE as desired.
22. Does there exist a nite set M of nonzero real numbers, su h that
for any natural number n a polynomial of degree no less than n with
oe ients in M , all of whose roots are real and belong to M ?
Su h a set does not exist. Suppose on the ontrary
that M = fa1 ; a2 ; : : : ; an g satis es the desired property. Let m =
minfja1 j; : : : ; jan jg and M = maxfja1j; : : : ; jan jg; the ondition implies M  m > 0.
Consider the polynomial P (x) = bk xk +    + b1 x + b0 , all of whose
oe ients b0 ; : : : ; bk and roots x1 ; : : : ; xk lie in M . By Vieta's theorem,

Solution:

bk 1
= x1 + : : : + xk
bk
b
x1 x2 + x1 x3 + : : : + xk 1 xk = k 2
bk
and so
It follows that

b2
x21 + : : : + x2k = k 2 1
bk

b2
km2  x21 + : : : + x2k = k 2 1
bk

b
2 k 2:
bk

b
2 k 2
bk

M
m

2
2

+2

M
:
m

Hen e k  M 2 =m4 + 2M=m3, ontradi ting the fa t that P may


have arbitrarily large degree.
23. The numbers from 1 to 100 are written in an unknown order. One
may ask about any 50 numbers and nd out their relative order.
What is the fewest questions needed to nd the order of all 100
numbers?
Solution: Five questions are needed. To determine the order
of a1 ; : : : ; a100 in the sequen e, it is ne essary that ea h of the pairs
(ai ; ai+1 ) (i = 1; :::; 99) o ur together in at least one question, or else

73

the two sequen es a1 ; : : : ; ai ; ai+1 ; : : : ; an and a1 ; : : : ; ai+1 ; ai ; : : : ; an


will give the same answers. We will show that for any two questions,
there an arise a situation where in luding all pairs of onse utive
numbers not already in luded requires at least three questions. Let
k1 ; : : : ; k50 be the sequen e (in order) of numbers about whi h the
0 the orresponding sequen e
rst question was asked, and k10 ; : : : ; k50
for the se ond question. We will onstru t a sequen e a1 ; : : : ; a100 for
whi h we annot, given two more questions, uniquely determine the
order of the terms. We onsider a situation where all of the numbers
named in the rst two questions appear in the answers in the very
same pla es.
For our desired sequen e we shall hoose a set with
ki ; k0 2 fa2i 1 ; a2i g; i = 1; : : : ; 50
i

and moreover, for ea h quadruple (a4m 3 ; a4m 2 ; a4m 1 ; a4m ) (m =


1; : : : ; 25), in the rst two questions there is no omparison of a
onse utive pair from this quadruple. We will show that su h a set
exists. Let X be the set of numbers not named in the rst two
questions. For ea h of the four ases
1 : k2m 1 = k20 m 1 ; k2m = k20 m
2 : k2m 1 = k20 m 1 ; k2m 6= k20 m
3 : k2m 1 6= k20 m 1 ; k2m 6= k20 m
4 : k2m 1 6= k0
; k2m = k 0 ;
2

we onstru t the quadruple (a4m 3 ; a4m 2 ; a4m 1 ; a4m ) in the following manner:
1:
2:
3:
4:

(k2m
(k2m
(k2m
(k2m

; ; ; k2m);
0
1 ; ; k2m ; k2m )
0
0
1 ; k2m 1 ; k2m ; k2m )
0
1 ; k2m 1 ; ; k2m );

where in pla e of a  we may hoose any number in X not o uring


in the previously onstru ted quadruples.
Hen e we have shown that after any two questions, a situation is
possible where no pair (ai ; ai+1 ) o urs for i not a multiple of 4.
74

Ea h of the 100 numbers o urs in at least one of the nonin luded


pairs, and so must appear in one of the remaining questions.
Suppose that in the given situation, all remaining pairs an be in luded in two questions; then ea h of the 100 numbers must appear
in exa tly one of these questions. Considering the quadruples of the
form (a4i 3 ; a4i 2 ; a4i 1 ; a4i ) (i = 1; : : : ; 25), we noti e that if one
of the numbers in the quadruple appears in some question, then the
remaining three numbers must also appear in the question (or else
not all of the pairs of onse utive numbers in the quadruple would be
in luded). But then the number of numbers in one question would
have to be a multiple of 4, whi h 50 is not, giving a ontradi tion.
Hen e 4 questions do not su e in general. We now show that 5
questions su e. We ask the rst question about M1 = f1; : : : ; 50g,
and the se ond about M2 = f51; : : : ; 100g. The set M3 onsists of
the 25 leftmost numbers from ea h of M1 and M2 , while M4 onsists
of the 25 rightmost numbers from ea h of M1 and M2. Clearly the
answer to the third question lo ates the rst 25 numbers, and the
answer to the fourth question lo ates the last 25. The fth question,
asked about the other 50 numbers, ompletely determines the order.

75

1.15 Spain
1. The natural numbers a and b are su h that
a+1 b+1
+
b
a
is an integer. Show
p that the greatest ommon divisor of a and b is
not greater than a + b.
Solution: Let d = g d(a; b) and put a = md and b = nd. Then we

have (md + 1)=nd + (nd + 1)=md = (m2 d + m + n2 d + n)=mnd is an


integer, so that in parti ular, d divides m2 d + m + n2 dp+ n and also
m
p + n. However, this means d  m + n, and so d  d(m + n) =
a + b.
2. Let G be the entroid of the triangle ABC . Prove that if AB + GC =
AC + GB , then ABC is isos eles.
Solution: Let a; b; be the lengths of sides BC; CA; AB , respe tively. By Stewart's theorem and the fa t that G trise ts ea h median
(on the side further from the vertex), we dedu e
9GB 2 = 2a2 + 2 2 b2 ;
9GC 2 = 2a2 + 2b2 2 :
Now assume b > . Assuming AB + GC = AC + GB , we have
p

2a2 + 2 2 b2
2
3(b2 2 )
p
= p 2
2a + 2b2 2 + 2a2 + 2 2 b2
3(b2 2 )
p
< p
2(b )2 + 2b2 2 + 2(b )2 + 2 2 b2
sin e a2 > (b )2 by the triangle inequality. However, 2(b )2 +
2b2 2 = (2b )2 , so we have
3(b2 2 )
3(b ) <
:
2b + j2 bj
If b  2 then the two sides are equal, a ontradi tion. If b > 2 we
get 9(b )2 < 3(b2 2 ); upon dividing o 3(b ) and rearranging,
we get 2b < 4 , again a ontradi tion. Thus we annot have b >
or similarly b < , so b = .
3(b ) =

2a2 + 2b2

76

3. Let a; b; be real numbers. Consider the fun tions

f (x) = ax2 + bx + ;

g(x) = x2 + bx + a:

Given that

jf (

1)j  1;

jf (0)j  1; jf (1)j  1;

show that for 1  x  1,

jf (x)j  54

and

jg(x)j  2:

We may assume a > 0, so that f is onvex; then


f ( 1); f (1)  1 implies f (x)  1 for 1  x  1, so it su es
to look at the point t where f takes its minimum. If t is not in
the interval, we have f (x)  1, so assume it is; without loss of
generality, we may assume t  0.
We now onsider two ases. First suppose t  1=2. In this ase
f ( 1)  f (1)  f (0), so it su es to impose the onditions f ( 1) 
1; f (0)  1. If we write f (x) = a(x t)2 + k, we have 2  f ( 1)
f (0) = a(2t +1), so a  2=(2t +1). Then f (0)  1 means at2 + k 
1, so
2t
2t2
= 1
;
k  1 at2  1
2t + 1
2 + 1=t
whi h is de reasing in t (the numerator of the fra tion is in reasing,
the denominator is de reasing and there is a minus sign in front).
Thus k  5=4.
Now suppose t  1=2. In this ase f ( 1)  f (0)  f (1), so the
relevant onditions are f ( 1)  1; f (1)  1. If we write f (x) =
a(x t)2 + k, we have 2  f ( 1) f (1) = 2at, so a  1=t. Then
f (1)  1 means a(1 t)2 + k  1, so
Solution:

k  1 a(1 t)2  1

(1 t)2
= 1
t

(1 t)
t=(1 t)

whi h is in reasing in t (similar reasoning). Thus k  5=4.


We move on to g. We assume > 0 and that the minimum of
g o urs in [0; 1. Assuming g( 1); g(1)  1, we again need only
77

determine the minimum of g. Writing g(x) = (x t)2 + k, we have


 1 and (1 t)2 + k  1, so

k  1 (1 t)2  1 (1 t)2  2:
4. Find all real solutions of the equation
p

x2

p + 2 x2

1=x

for ea h real value of p.


Solution: Squaring both sides, we get
p

x2 = 5x2 4 p + 4 (x2

p)(x2

1):

Isolating the radi al and squaring again, we get


16(x2

p)(x2

1) = (4x2

whi h redu es to (16 8p)x2 = p2


sum of two square roots), we have

x=

p 4)2 ;

8p + 16. Sin e x  0 (it is the

p 4j
pj16
8p

if a solution exists. We need only determine when this value a tually


satis es. Certainly we need p  2. In that ase
p plugging in our
laimed value of x and multiplying through by 16 8p gives

j3p

4j + 2jpj = 4 p:

If p  4=3 this be omes 6p = 8, or p = 4=3; if 0  p  4=3 this holds


identi ally; if p  0 this be omes 4p = 0, or p = 0. We on lude
there exists a solution if and only if 0  p  4=3, in whi h ase it is
the solution given above.
5. At Port Aventura there are 16 se ret agents. Ea h agent is wat hing
one or more other agents, but no two agents are both wat hing ea h
other. Moreover, any 10 agents an be ordered so that the rst is
wat hing the se ond, the se ond is wat hing the third, et ., and the
last is wat hing the rst. Show that any 11 agents an also be so
ordered.
78

Solution: We say two agents are partners if neither wat hes the

other. First note that ea h agent wat hes at least 7 others; if an agent
were wat hing 6 or fewer others, we ould take away 6 agents and
leave a group of 10 whi h ould not be arranged ina ir le. Similarly,
ea h agent is wat hed by at least 7 others. Hen e ea h agent is allied
with at most one other.
Given a group of 11 agents, there must be one agent x who is not
allied with any of the others in the group (sin e allies ome in pairs).
Remove that agen t and arrange the other 10 in a ir le. The removed
agent wat hes at least one of the other 10 and is wat hed by at least
one. Thus there exists a pair u; v of agents with u wat hing v,
u wat hing x and x wat hing v (move around the ir le until the
dire tion of the arrow to x hanges); thus x an be spli ed into the
loop between u and v.
6. A regular pentagon is onstru ted externally on ea h side of a regular
pentagon of side 1. This gure is then folded and the two edges
meeting at ea h vertex of the original pentagon but not belonging
to the original pentagon are glued together. Determine the volume
of water that an be poured into the resulting ontainer without
spillage.
Solution: The gure formed by the water is a prismatoid of height
equal to the verti al omponent of one of the glued edges. To determine
this omponent, introdu te a oordinate system entered at one of the
base verti es, su h that ( os 36; sin 36; 0) and ( os 36; sin 36; 0) are two
verti es. (All angles are measured in degrees.) The third vertex adja ent
to this one has oordinates (0; y; z ) for some y; z with z > 0, y2 + z 2 = 1
and y os 36 = os 108 (this being the dot produ t of the ve tors of the
two edges ). Therefore

5)=4
os 108 (1
p
=
y=
os 36 (1 + 5)=4

and z = 2  51=4 =(1 + 5).


Now we must determine the areas of the bases of the prismatoid. The
area of the lower base is the area of a regular pentagon of side 1, whi h is
5=4 ot 36. The area of the upper base is the area of a regular pentagon
79

in whi h the ir umradius has been in reased by y, namely 5=4 ot 36(1 +


y sin 36)2 . The volume is the height times the average of the bases, namely
5 5 =4
p ot 36(1 + os 108 tan 36)2  0:956207:
2(1 + 5)

80

1.16 Turkey
1. Let
1996
Y

n=1

1 + nx3

n

= 1 + a1 xk1 + a2 xk2 + : : : + am xkm ;

where a1 ; a2 ; : : : ; am are nonzero and k1 < k2 < : : : < km . Find


a1996 .
Solution: Note that ki is the number obtained by writing i in
base 2 and reading the result as a number in base 3, and ai is the
sum of the exponents of the powers of 3 used. In parti ular, 1996 =
210 + 29 + 28 + 27 + 26 + 23 + 22 , so

a1996 = 10 + 9 + 8 + 7 + 6 + 3 + 2 = 45:
2. In a parallelogram ABCD with \A < 90, the ir le with diameter
AC meets the lines CB and CD again at E and F , respe tively, and
the tangent to this ir le at A meets BD at P . Show that P; F; E
are ollinear.
Without loss of generality, suppose B; D; P o ur in
that order along BD. Let G and H be the se ond interse tions of
AD and AB with the ir le. By Menelaos's theorem, it su es to
show that
CE  BP  DF
= 1:
EB  P D  F C
First note that
BP AD sin \BAP sin \AP D sin \BAP
=
=
:
AB DP sin \AP B sin \DAP sin \DAP
Sin e AP is tangent to the ir le, \BAP = \HAP =  \HCA =
 \F AC ; similarly, \DAP = \GCA = \EAC . We on lude
BP AD sin \F AC F C
=
=
:
AB DP sin \EAC EC
Finally we note that DF=BE = DA=AB be ause the right triangles
AF D and AEB have the same angles at B and D and are thus
similar. This proves the laim.

Solution:

81

3. Given real numbers 0 = x1 < x2 < : : : < x2n < x2n+1 = 1 with
xi+1 xi  h for 1  i  2n, show that
n
1 h X
< x2i (x2i+1
2
i=1

x2i 1 ) <

1+h
:
2

Solution: The di eren e between the middle quantity and 1=2 is

the di eren e between the sum of the areas of the re tangles bounded
by the lines x = x2i 1 , x = x2i+1 , y = 0, y = x2i and the triangle
bounded by the lines y = 0; x = 1; x = y. The area ontained in the
re tangles but not the triangle is a union of triangles of total base
less than 1 and height at most h, as is the area ontained in the
triangle but not the re tangles. Hen e the sum di ers from 1=2 by
at most h=2, as desired.

4. In a onvex quadrilateral ABCD, triangles ABC and ADC have


the same area. Let E be the the interse tion of AC and BD,
and let the parallels through E to the lines AD; DC; CB; BA meet
AB; BC; CD; DA at K; L; M; N , respe tively. Compute the ratio of
the areas of the quadrilaterals KLMN and ABCD.
Solution: The triangles EKL and DAC are homotheti , so the
ratio of their areas equals (EK=AD)(EL=CD) = (BE=BD)2 = 1=4,
sin e B and D are equidistant from the line AC . Similarly the ratio
of the areas of EMN and BCA is 1=4, so the union of the triangles
EKL and EMN has area 1=4 that of ABCD.
As for triangle EKN , its base KN is parallel to BD and half as
long, so its area is one-fourth that of ABD. Similarly EML has
area one-fourth that of BCD, and so the union of the two triangles
EKN and EML has area one-fourth that of ABCD, and so the
quadrilateral KLMN has area one-half that of ABCD.

5. Find the maximum number of pairwise disjoint sets of the form


Sa;b = fn2 + an + b : n 2 Zg with a; b 2 Z.
Only two su h sets are possible, for example, with
(a; b) = (0; 0) and (0; 2) (sin e 2 is not a di eren e of squares). There
is no loss of generality in assuming a 2 f0; 1g by a suitable shift of

Solution:

82

n, and the sets generated by (0; a) and (1; b) have the ommon value
(a b)2 + a = (a b)2 + (a b) + b. Thus we have a = 0 or a = 1
universally.
First suppose a = 0. If b 6 2 (mod 4), then (0; b) and (0; ) give
a ommon value be ause b is a di eren e of squares; learly this
pre ludes having three disjoint sets. Now suppose a = 1. If b is
even, we an nd x; y su h that b = (x + y + 1)(x y), and so
x2 + x + b = y2 + y + ; again, this pre ludes having three disjoint
sets.
6. For whi h ordered pairs of positive real numbers (a; b) is the limit of
every sequen e fxn g satisfying the ondition
lim (axn+1

n!1

bxn ) = 0

zero?
Solution: This holds if and only if b < a. If b > a, the sequen e
xn = (b=a)n satis es the ondition but does not go to zero; if b = a,
the sequen e xn = 1 + 1=2 +    + 1=n does likewise. Now suppose
b < a. If L and M are the limit inferior and limit superior of the
given sequen e, the ondition implies M  (b=a)L; sin e L  M ,
we have M  (b=a)M , and so L; M  0. Similarly, the ondition
implies L  (b=a)M , and sin e M  L, we have L  (b=a)L, so
L; M  0; therefore L = M = 0 and the sequen e onverges to 0.

83

1.17 United Kingdom


1. Consider the pair of four-digit positive integers
(M; N ) = (3600; 2500):
Noti e that M and N are both perfe t squares, with equal digits in
two pla es, and di ering digits in the remaining two pla es. Moreover, when the digits di er, the digit in M is exa tly one greater
than the orresponding digit in N . Find all pairs of four-digit positive integers (M; N ) with these properties.
Solution: If M = m2 and N = n2 , then

(m + n)(m n) 2 f11; 101; 110; 1001; 1010; 1100g:


Sin e M and N are four-digit numbers, we must have 32  n < m 
99, and so 65  m + n  197. Moreover, m + n and m n are both
odd or both even, so 11, 110 and 1010 lead to no solutions. From
this we get exa tly ve a eptable fa torizations:
101
1001
1001
1001
1100

=
=
=
=
=

(m + n)(m
(m + n)(m
(m + n)(m
(m + n)(m
(m + n)(m

n) = 101  1
n) = 143  7
n) = 91  11
n) = 77  13
n) = 110  10

giving the solutions


(M; N ) = (2601; 2500); (5625; 4624); (2601; 1600); (2025; 1024); (3600; 2500):
2. A fun tion f de ned on the positive integers satis es f (1) = 1996
and
f (1) + f (2) +    + f (n) = n2 f (n) (n > 1):
Cal ulate f (1996).
Solution: An easy indu tion will show that

f (n) =
84

2  1996
:
n(n + 1)

Namely,
1

3992
3992
f (n) = 2
++
n 1 12
(n 1)n

3992
1 1 1
1
= 2
1
+
++
n 1
2 2 3
n 1


1
3992
1
=
(n + 1)(n 1)
n
3992
n 1
3992
=
=
:
(n + 1)(n 1) n
n(n + 1)
In parti ular, f (1996) = 2=1997:

1
n

3. Let ABC be an a ute triangle and O its ir um enter. Let S denote


the ir le through A; B; O. The lines CA and CB meet S again
at P and Q, respe tively. Prove that the lines CO and P Q are
perpendi ular.
Solution: The angles \P AB and \BQP are supplementary, so
\BQP = \CAB (as dire ted angles mod ). In other words, the

line P Q makes the same angle with the line CQ as the tangent to
the ir um ir le of ABC through C . Hen e P Q is parallel to the
tangent, so perpendi ular to OC .
4. De ne

n
q(n) = p
b n (n = 1; 2; : : :):
Determine all positive integers n for whi h q(n) > q(n + 1).
Solution: We have q (n) > q (n + 1) if and only if n + 1 is a perfe t

square. Indeed, if n + 1 = m2 , then



 2
 2
m
m 1
= m + 1;
q(n + 1) =
= m:
q(n) =
m 1
m

On the other hand, for n = m2 + d with 0  d  2m,


 2

 
m +d
d
q(n) =
=m+
m
m
whi h is nonde reasing.
85

5. Let a; b; be positive real numbers.


(a) Prove that 4(a3 + b3 )  (a + b)3 .
(b) Prove that 9(a3 + b3 + 3 )  (a + b + )3 .
Solution: Both parts follow from the Power Mean inequality: for
r > 1 and x1 : : : ; xn positive,
 r
x

+    + xrn
n

1=r

x

+    + xn
;
n

whi h in turn follows from Jensen's inequality applied to the onvex


fun tion xr .
6. Find all solutions in nonnegative integers x; y; z of the equation
2x + 3y = z 2 :
Solution: If y = 0, then 2x = z 2

1 = (z + 1)(z 1), so z + 1 and


z 1 are powers of 2. The only powers of 2 whi h di er by 2 are 4
and 2, so (x; y; z ) = (3; 0; 3).
If y > 0, then 2x is a quadrati residue modulo 3, hen e x is even.
Now we have 3y = z 2 2x = (z + 2x=2)(z 2x=2 ). The fa tors
are powers of 3, say z + 2x=2 = 3m and z 2x=2 = 3n, but then
3m 3n = 2x=2+1 . Sin e the right side is not divisible by 3, we must
have n = 0 and
3m 1 = 2x=2+1 :
If x = 0, we have m = 1, yielding (x; y; z ) = (0; 1; 2). Otherwise,
3m 1 is divisible by 4, so m is even and 2x=2+1 = (3m=2 +1)(3m=2 1).
The two fa tors on the right are powers of 2 di ering by 2, so they
are 2 and 4, giving x = 4 and (x; y; z ) = (4; 2; 5).
7. The sides a; b; and u; v; w of two triangles ABC and UV W are
related by the equations

u(v + w u) = a2 ;
v(w + u v) = b2 ;
w(u + v w) = 2 :
86

Prove that ABC is a ute, and express the angles U; V; W in terms


of A; B; C .
Solution: Note that

a2 + b2

2 = w2

u2 v2 + 2uv = (w + u v)(w u + v) > 0

by the triangle inequality, so os C > 0. By this reasoning, all of the


angles of triangle ABC are a ute. Moreover,

a2 + b2 2
2ab
r
(w + u v)(w u + v)
=
4uv
r
w2 u2 v2 + 2uv
=
4uv
1 p
= p 1 os U
2
from whi h we dedu e os U = 1 2 os2 A = os( 2A). Therefore
U =  2A, and similarly V =  2B , W =  2C .
os C =

8. Two ir les S1 and S2 tou h ea h other externally at K ; they also


tou h a ir le S internally at A1 and A2 , respe tively. Let P be one
point of interse tion of S with the ommon tangent to S1 and S2 at
K . The line P A1 meets S1 again at B1 , and P A2 meets S2 again at
B2 . Prove that B1 B2 is a ommon tangent to S1 and S2 .
Solution: It su es to show that \B2 B1 O1 = \B1 B2 O2 = =2,
where O1 and O2 are the enters of S1 and S2 , respe tively. By
power-of-a-point, P A1  P B1 = P K 2 = P A2  P B2 , so triangles
P A1 A2 and P B2 B1 are similar. Therefore \P B1 B2 = \P A2 A1 =
1
\P OA1 , where O is the enter of S .
2
Now note that the homothety at A1 arrying S1 to S takes O1 to
O and B1 to P , so \P OA1 = \B1 O1 A1 . From this we dedu e
\P B1 B2 = \B1 O1 N , where N is the midpoint of A1 B1 . Finally,

\B2 B1 O1 =  \P B1 B2 \O1 B1 N = =2;


as desired.
87

9. Find all solutions in positive real numbers a; b; ; d to the following


system of equations:

a + b + + d = 12
ab d = 27 + ab + a + ad + b + bd + d:
Solution: The rst equation implies ab d  81 by the arithmeti geometri mean inequality, with equality holding for a = b = =
d = 3. Again by AM-GM,

ab d  27 + 6(ab d)1=2 :
However, x2 6x 27  0 for x  3 or x  9, so (ab d)1=2  9,
hen e ab d  81. We on lude ab d = 81, and hen e a = b = =
d = 3.

88

1.18 United States of Ameri a

1. Prove that the average of the numbers n sin n (n = 2; 4; 6; : : : ; 180)


is ot 1.
All arguments of trigonometri fun tions will be in
degrees. We need to prove

Solution:

2 sin 2 + 4 sin 4 +    + 178 sin 178 = 90 ot 1;

(2)

whi h is equivalent to
2 sin 2  sin 1+2(2 sin 4  sin 1)+    +89(2 sin 178  sin 1) = 90 os 1: (3)
Using the identity 2 sin a  sin b = os(a b) os(a + b), we nd
2 sin 2  sin 1 + 2(2 sin 4  sin 1) +    + 89(2 sin 178  sin 1)
= ( os 1 os 3) + 2( os 3 os 5) +    + 89( os 177 os 179)
= os 1 + os 3 + os 5 +    + os 175 + os 177 89 os 179
= os 1 + ( os 3 + os 177) +    + ( os 89 + os 91) 89 os 179
= os 1 + 89 os 1 = 90 os 1;
so (1) is true.
Note: An alternate solution involves omplex numbers. One expresses sin n as (ein=180 e in=180 )=(2i) and uses the fa t that

x + 2x2 +    + nxn = (x +    + xn ) + (x2 +    + xn ) +    + xn


1
=
[(xn+1 x) + (xn+1 x2 ) +   
x 1
+ (xn+1 xn )
nxn+1 xn+1 x
:
=
x 1 (x 1)2
2. For any nonempty set S of real numbers, let (S ) denote the sum of
the elements of S . Given a set A of n positive integers, onsider the
olle tion of all distin t sums (S ) as S ranges over the nonempty
subsets of A. Prove that this olle tion of sums an be partitioned
89

into n lasses so that in ea h lass, the ratio of the largest sum to


the smallest sum does not ex eed 2.
Solution:
Let A = fa1; a2 ; : : : ; an g where a1 < a2 <    < an .
For i = 1; 2; : : : ; n let si = a1 + a2 +    + ai and take s0 = 0. All
the sums in question are less than or equal to sn , and if  is one of
them, we have
si 1 <   si
(1)
for an appropriate i. Divide the sums into n lasses by letting Ci
denote the lass of sums satisfying (1). We laim that these lasses
have the desired property. To establish this, it su es to show that
(1) implies
1
s <   si :
(2)
2 i
Suppose (1) holds. The inequality a1 + a2 +    + ai 1 = si 1 < 
shows that the sum  ontains at least one addend ak with k  i.
Then sin e then ak  ai , we have

si

 < si

si

= ai  ak  ;

whi h together with   si implies (2).


Note: The result does not hold if 2 is repla ed by any smaller
onstant . To see this, hoose n su h that < 2 2 (n 1) and
onsider the set f1; : : : ; 2n 1 g. If this set is divided into n subsets,
two of 1; : : : ; 2n 1; 1 +    + 2n 1 must lie in the same subset, and
their ratio is at least (1 +    + 2n 1 )=(2n 1 ) = 2 2 (n 1) > .

3. Let ABC be a triangle. Prove that there is a line ` (in the plane of
triangle ABC ) su h that the interse tion of the interior of triangle
ABC and the interior of its re e tion A0 B 0 C 0 in ` has area more
than 2=3 the area of triangle ABC .
First Solution: In all of the solutions, a; b; denote the lengths
of the sides BC; CA; AB , respe tively, and we assume without loss
of generality that a  b  .
Choose ` to be the angle bise tor of \A. Let P be the interse tion of
` with BC . Sin e AC  AB , the interse tion of triangles ABC and

90

A0 B 0 C 0 is the disjoint union of two ongruent triangles, AP C and


AP C 0 . Considering BC as a base, triangles AP C and ABC have
equal altitudes, so their areas are in the same ratio as their bases:
Area(AP C ) P C
=
:
Area(ABC ) BC
Sin e AP is the angle bise tor of \A, we have BP=P C = =b, so
PC
1
PC
=
=
:
BC BP + P C =b + 1
Thus it su es to prove
2
2
> :
(1)
=b + 1 3
But 2b  a + b > by the triangle inequality, so =b < 2 and thus
(1) holds.
Se ond Solution: Let the foot of the altitude from C meet AB

at D. We will use the notation [XY Z to denote the area of triangle


XY Z .
First suppose [BDC > (1=3)[ABC . In this ase we re e t through
CD. If B 0 is the image of B , then BB 0 C lies in ABC and the area
of the overlap is at least 2=3[ABC .
Now suppose [BDC  (1=3)[ABC . In this ase we re e t through
the bise tor of \A. If C 0 is the image of C , then triangle ACC 0 is
ontained in the overlap, and [ACC 0 > [ADC  2=3[ABC .
Note: Let F denote the gure given by the interse tion of the
interior of triangle ABC and the interior of its re e tion in `. Yet
another approa h to the problem involves nding the maximum attained for Area(F )=Area(ABC ) by taking ` from the family of lines
perpendi ular to AB . By hoosing the best alternative between the
angle bise tor at C and the optimal line perpendi ular to AB , one
an ensure
p
2
Area(F )
p
= 2( 2 1) = 0:828427; : : :
>
Area(ABC ) 1 + 2
and this onstant is in fa t the best possible.

91

4. An n-term sequen e (x1 ; x2 ; : : : ; xn ) in whi h ea h term is either 0


or 1 is alled a binary sequen e of length n. Let an be the number of
binary sequen es of length n ontaining no three onse utive terms
equal to 0, 1, 0 in that order. Let bn be the number of binary
sequen es of length n that ontain no four onse utive terms equal
to 0, 0, 1, 1 or 1, 1, 0, 0 in that order. Prove that bn+1 = 2an for all
positive integers n.
First Solution: We refer to the binary sequen es ounted by (an )

and (bn ) as \type A" and \type B", respe tively. For ea h binary
sequen e (x1 ; x2 ; : : : ; xn ) there is a orresponding binary sequen e
(y0 ; y1 ; : : : ; yn ) obtained by setting y0 = 0 and

yi = x1 + x2 +    + xi mod 2; i = 1; 2; : : : ; n:

(2)

(Addition mod 2 is de ned as follows: 0 + 0 = 1 + 1 = 0 and


0 + 1 = 1 + 0 = 1.) Then

xi = yi + yi

mod 2; i = 1; 2; : : : ; n;

and it is easily seen that (1) provides a one-to-one orresponden e between the set of all binary sequen es of length n and the set of binary
sequen es of length n + 1 in whi h the rst term is 0. Moreover, the
binary sequen e (x1 ; x2 ; : : : ; xn ) has three onse utive terms equal
to 0, 1, 0 in that order if and only if the orresponding sequen e
(y0 ; y1 ; : : : ; yn ) has four onse utive terms equal to 0, 0, 1, 1 or 1, 1,
0, 0 in that order, so the rst is of type A if and only if the se ond
is of type B. The set of type B sequen es of length n + 1 in whi h
the rst term is 0 is exa tly half the total number of su h sequen es,
as an be seen by means of the mapping in whi h 0's and 1's are
inter hanged.
Se ond Solution: The expression 2an

an ounts the number


1
of type A sequen es of length n 1 that do not remain of type A
when 0 is atta hed, or in other words the number of type A sequen es
of length n 1 ending in 0, 1. Sin e a type A sequen e followed by
1 is still of type A, this number in turn equals the number of type A
sequen es of length n 2 ending in 0. However, this number an be
viewed as the number of type A sequen es of length n 2, minus the
92

number of su h sequen es ending in 1. There are as many type A


sequen es of length n 2 ending in 1 as there are type A sequen es
of length n 3, again sin e every type A sequen e remains type A
when 1 is added. We on lude that 2an 1 an = an 2 an 3 , or
equivalently
an = 2an 1 an 2 + an 3 :
We also note the initial values a0 = 1; a1 = 2; a2 = 4.
Similarly, the expression 2bn 1 bn ounts the number of type B
sequen es of length n 1 ending in 0,0,1 or 1,1,0. Sin e adding 1 to
a type B sequen e ending in 0, or 0 to a type B sequen e ending in
1, yields another type B sequen e, this number is the same as the
number of type B sequen es of length n 2 ending in 0,0 or 1,1, or
equivalently the number not ending in 0,1 or 1,0. As in the type A
ase, we note that ea h type B sequen e of length n 3 generates a
unique type B sequen e of length n 2 ending in 0, 1 or 1, 0, and so
2bn 1 bn = bn 2 bn 3. Sin e b1 = 2; b2 = 4; b4 = 8, we on lude
by indu tion that bn+1 = 2an for all n.
5. Triangle ABC has the following property: there is an interior point P
su h that \P AB = 10 , \P BA = 20, \P CA = 30, and \P AC =
40. Prove that triangle ABC is isos eles.
First Solution: All angles will be in degrees. Let x = \P CB .
Then \P BC = 80 x. By the Law of Sines,
PA PB PC
1 =
PB PC PA
sin \P BA sin \P CB sin \P AC
=
sin \P AB sin \P BC sin \P CA
sin 20 sin x sin 40
4 sin x sin 40 os 10
=
=
:
sin 10 sin(80 x) sin 30
sin(80 x)

The identity 2 sin a  os b = sin(a b) + sin(a + b) now yields


1=

2 sin x(sin 30 + sin 50) sin x(1 + 2 os 40)


=
;
sin(80 x)
sin(80 x)

so
2 sin x os 40 = sin(80 x) sin x = 2 sin(40 x) os 40:
93

This gives x = 40 x and thus x = 20. It follows that \ACB =


50 = \BAC , so triangle ABC is isos eles.
Let D be the re e tion of A a ross the line
BP . Then triangle AP D is isos eles with vertex angle

Se ond Solution:

\AP D = 2(180 \BP A) = 2(\P AB + \ABP ) = 2(10 + 20) = 60;


and so is equilateral. Also, \DBA = 2\P BA = 40. Sin e \BAC =
50, we have DB ? AC .
Let E be the interse tion of DB with CP . Then

\P ED = 180 \CED = 180 (90 \ACE ) = 90 + 30 = 120


and so \P ED + \DAP = 180. We dedu e that the quadrilateral
AP ED is y li , and therefore \DEA = \DP A = 60.
Finally, we note that \DEA = 60 = \DEC . Sin e AC ? DE ,
we dedu e that A and C are symmetri a ross the line DE , whi h
implies that BA = BC , as desired.

6. Determine (with proof) whether there is a subset X of the integers


with the following property: for any integer n there is exa tly one
solution of a + 2b = n with a; b 2 X .
First Solution: Yes, there is su h a subset. If the problem is
restri ted to the nonnegative integers, it is lear that the set of integers whose representations in base 4 ontains only the digits 0 and 1
satis es the desired property. To a ommodate the negative integers
as well, we swit h
to \base 4". That is, we represent every integer
P
in the form ki=0 i ( 4)i , with i 2 f0; 1; 2; 3g for all i and k 6= 0,
and let X be the set of numbers whose representations use only the
digits 0 and 1. This X will again have the desired property, on e we
show that every integer has a unique representation in this fashion.
To show base 4 representations are unique, let f i g and fdi g be
two distin t nite sequen es of elements of f0; 1; 2; 3g, and let j be
the smallest integer su h that j 6= dj . Then
k
X
i=0

i ( 4)i 6

k
X
i=0

94

di ( 4)i (mod 4j );

so in parti ular the two numbers represented by f i g and fdi g are


distin t. On the other hand, to show that n admits a base 4
representation, nd an integer k su h
that 1 + 42 +    + 42k  n
P2k
2k 1
and express n + 4 +    + 4
as i=0 i 4i . Now set d2i = 2i and
P k
d2i 1 = 3 2i 1 , and note that n = 2i=0
di ( 4)i .
Se ond Solution: For any S  Z, let S  = fa +2bj a; b 2 S g. Call

a nite set of integers S = fa1 ; a2 ; : : : ; am g  Z good if jS  j = jS j2 ,


i.e., if the values ai + 2aj (1  i; j  m) are distin t. We rst
prove that given a good set and n 2 Z, we an always nd a good
superset T of S su h that n 2 T . If n 2 S  already, take T = S .
Otherwise take T = S [ fk; n 2kg where k is to be hosen. Then
put T  = S  [ Q [ R, where

Q = f3k; 3(n 2k); k + 2(n 2k); (n 2k) + 2kg


and

R = fk + 2ai ; (n 2k) + 2ai ; ai + 2k; ai + 2(n 2k)j 1  i  mg:


Note that for any hoi e of k, we have n = (n 2k) + 2k 2 Q  T .
Ex ept for n, the new values are distin t non onstant linear forms
in k, so if k is su iently large, they will all be distin t from ea h
other and from the elements of S  . This proves that T  is good.
Starting with the good set X0 = f0g, we thus obtain a sequen e
of sets X1 ; X2 ; X3 ; : : : su h that for ea h positive integer j , Xj is a
good superset of Xj 1 and Xj ontains the j th term of the sequen e
1; 1; 2; 2; 3; 3; : : : . It follows that
1
[
X = Xj
j =0

has the desired property.

95

1.19 Vietnam
1. Solve the system of equations:

7y 1

1
= 2
x+y

p
1
= 4 2:
x+y

3x 1 +

Solution: Let u = x; v = y, so the system be omes

2
u
=p
2
u +v
p3
v
4 2
v
= p :
u2 + v2
7
Now let z = u + vi; the system then redu es to the single equation

u+

p1 + 2p 2 i :
3
7

1
z+ =2
z

Let t denote the quantity inside the parentheses; then


p

z = t  t2 1
p 
p 
1 2 2
2
= p + p i  p + 2I
3
7
21
from whi h we dedu e


2
1
u= p p
3
21

2

!2

2 2 p
v= p  2
7

2. Let ABCD be a tetrahedron with AB = AC = AD and ir um enter O. Let G be the entroid of triangle ACD, let E be the
midpoint of BG, and let F be the midpoint of AE . Prove that OF
is perpendi ular to BG if and only if OD is perpendi ular to AC .
We identify points with their ve tors originating from
the ir um enter, so that A  B = A  C = A  D and A2 = B 2 =

Solution:

96

C 2 = D2 . Now
1
(A + E )  (B G)
2
1
= [(2A + B + G)  (B G)
4
1
=
[18A  B 6A  (A + C + D) + 9B 2
36
1
=
[2A  D 2C  D:
36
Therefore OF ? BF if and only if OD ? AC .
(O

F )  (B

G) =

(A + C + D)2

3. Determine, as a fun tion of n, the number of permutations of the set


f1; 2, : : : ; ng su h that no three of 1; 2; 3; 4 appear onse utively.
Solution: There are n! permutations in all. Of those, we ex lude
(n 2)! permutations for ea h arrangement of 1; 2; 3; 4 into an ordered
triple and one remaining element, or 24(n 2)! in all. However, we
have twi e ex luded ea h of the 24(n 3)! permutations in whi h all
four of 1; 2; 3; 4 o ur in a blo k. Thus the number of permutations
of the desired form is n! 24(n 2)! + 24(n 3)!.

4. Determine all fun tions f : N

! N satisfying (for all n 2 N )

f (n) + f (n + 1) = f (n + 2)f (n + 3) 1996:


Solution: From the given equation, we dedu e

f (n) f (n + 2) = f (n + 3)[f (n + 2) f (n + 4):


If f (1) > f (3), then by indu tion, f (2m 1) > f (2m +1) for all m >
0, giving an in nite de reasing sequen e f (1); f (3); : : : of positive
integers, a ontradi tion. Hen e f (1)  f (3), and similarly f (n) 
f (n + 2) for all n.
Now note that
0 = 1996 + f (n) + f (n + 1) f (n + 2)f (n + 3)
 1996 + f (n + 2) + f (n + 3) f (n + 2)f (n + 3)
= 1997 [f (n + 2) 1[f (n + 3) 1:
97

In parti ular, either f (n + 2) = 1 or f (n + 3)  1997, and vi e versa.


The numbers f (2m + 1) f (2m 1) are either all zero or all positive, and similarly for the numbers f (2m + 2) f (2m). If they are
both positive, eventually f (n + 2) and f (n + 3) both ex eed 1997, a
ontradi tion.
We now split into three ases. If f (2m) and f (2m + 1) are both
onstant, we have [f (2m) 1[f (2m + 1) 1 = 1997 and so either
f (2m) = 1 and f (2m + 1) = 1997 or vi e versa. If f (2m + 1)
is onstant but f (2m) is not, then f (2m + 1) = 1 for all m and
f (2m +2) = f (2m)+1997, so f (2m) = 1997(m 1)+ f (2). Similarly,
if f (2m) is not onstant, then f (2m) = 1 and f (2m + 1) = 1997m +
f (1).
5. Consider triangles ABC where BC = 1 and \BAC has a xed
measure > =3. Determine whi h su h triangle minimizes the
distan e between the in enter and entroid of ABC , and ompute
this distan e in terms of .
Solution: If we x B and C and for e A to lie above the line BC ,
then A is onstrained to an ar . The entroid of ABC is onstrained
to the image of that ar under a 1/3 homothety at the midpoint of
BC . On the other hand, the in enter subtends an angle of ( + )=2
at BC , so it is also onstrained to lie on an ar , but its ar passes
through B and C . Sin e the top of the in enter ar lies above the
top of the entroid ar , the ar s annot interse t (or else their ir les
would interse t four times). Moreover, if we dilate the entroid ar
about the midpoint of BC so that its image is tangent to the in enter
ar at its highest point, the image lies between the in enter ar and
BC .
In other words, the distan e from the in enter to the entroid is always at least the orresponding distan e for ABC isos eles. Hen e
we simply ompute the distan e in that ase. The in enter makes
an isos eles triangle of vertex angle ( + )=2, so its altitude is
1=2 ot( + )=4. Meanwhile, the distan e of the entroid to BC
is 1=3 that of A to BC , or 1=6 ot( =2). The desired distan e is thus

+
1
ot
2
4
98

1

ot :
6
2

6. Let a; b; ; d be four nonnegative real numbers satisfying the ondition


2(ab + a + ad + b + bd + d) + ab + abd + a d + b d = 16:
Prove that
2
a + b + + d  (ab + a + ad + b + bd + d)
3
and determine when equality o urs.
Solution: For i = 1; 2; 3, de ne si as the average of the produ ts
of the i-element subsets of fa; b; ; dg. Then we must show

3s2 + s3 = 4 ) s1  s2 :
It su es to prove the (un onstrained) homogeneous inequality
3s22 s21 + s3 s31  4s32;
as then 3s2 + s3 = 4 will imply (s1 s2 )3 + 3(s31 s32 )  0.
We now re all two basi inequalities about symmetri means of nonnegative real numbers. The rst is S hur's inequality:
3s31 + s3  4s1s2 ;
while the se ond,

s21  s2
is a ase of Ma laurin's inequality sii+1
prove the laim:
3s22 s21 + s3 s31  3s22s21 +

 sii
s22 s3
s1

+1

. These ombine to

 4s :
3
2

Finally, for those who have only seen S hur's inequality in three
variables, note that in general any inequality involving s1 ; : : : ; sk
whi h holds for n  k variables also holds for n + 1 variables, by
repla ing the variables x1 ; : : : ; xn+1 by the roots of the derivative of
the polynomial (x x1 )    (x xn+1 ).
99

1996 Regional Contests:


Problems and Solutions

2.1 Asian Pa i Mathemati s Olympiad


1. Let ABCD be a quadrilateral with AB = BC = CD = DA. Let
MN and P Q be two segments perpendi ular to the diagonal BD and
su h that the distan e between them is d > BD=2, with M 2 AD,
N 2 DC , P 2 AB , and Q 2 BC . Show that the perimeter of the
hexagon AMNCQP does not depend on the position of MN and
P Q so long as the distan e between them remains onstant.
Solution: The lengths of AM; MN; NC are all linear in the dis-

tan e between the segments MN and AC ; if this distan e is h, extrapolating from the extremes MN = AC and M = N = D gives
that
2AB AC
AM + MN + NC = AC +
h:
BD=2
In parti ular, if the segments MN and P Q maintain onstant total
distan e from AC , as they do if their distan e remains onstant, the
total perimeter of the hexagon is onstant.
2. Let m and n be positive integers su h that n  m. Prove that
2n n! 

(m + n)!
(m n)!

 (m

+ m)n :

Solution: The quantity in the middle is (m + n)(m + n 1)    (m

n + 1). If we pair o terms of the form (m + x) and (m + 1 x),


we get produ ts whi h do not ex eed m(m + 1), sin e the fun tion
f (x) = (m + x)(m + 1 x) is a on ave parabola with maximum
at x = 1=2. From this the right inequality follows. For the left, we
need only show (m + x)(m + 1 x)  2x for x  n; this rearranges
to (m x)(m + 1 + x)  0, whi h holds be ause m  n  x.

3. Let P1 ; P2 ; P3 ; P4 be four points on a ir le, and let I1 be the in enter


of the triangle P2 P3 P4 , I2 be the in enter of the triangle P1 P3 P4 , I3
be the in entre of the triangle P1 P2 P4 , and I4 be the in enter of the
100

triangle P1 P2 P3 . Prove that I1 ; I2 ; I3 and I4 are the verti es of a


re tangle.
Solution: Without loss of generality, assume P1 ; P2 ; P3 ; P4 o ur
on the ir le in that order. Let M12; M23 ; M34 ; M41 be the midpoints
of ar s P1 P2 ; P2 P3 ; P3 P4 ; P4 P1 , respe tively. Then the line P3 M1 is
the angle bise tor of \P2 P3 P1 and so passes through I4 . Moreover,
the triangle M12 P2 I4 is isos eles be ause

\I4 M12 P2 = \P3 P1 P2


=  2\P1 P2 I4 2\M12P2 P1
=  2\M12 P2 I4 :
Hen e the ir le entered at M passing through P1 and P2 also passes
through I4 , and likewise through I3 .
From this we determine that the angle bise tor of \P3 M12 P4 is the
perpendi ular bise tor of I3 I4 . On the other hand, this angle bise tor
passes through M34 , so it is simply the line M12 M34 ; by symmetry, it
is also the perpendi ular bise tor of I1 I2 . We on lude that I1 I2 I3 I4
is a parallelogram.
To show that I1 I2 I3 I4 is a tually a re tangle, it now su es to show
that M12 M34 ? M23 M41 . To see this, simply note that the angle between these lines is half the sum of the measures of the ar s M12 M23
and M34 M41 , but these ar s learly omprise half of the ir le.
4. The National Marriage Coun il wishes to invite n ouples to form
17 dis ussion groups under the following onditions:
(a) All members of the group must be of the same sex, i.e. they are
either all male or all female.
(b) The di eren e in the size of any two groups is either 0 or 1.
( ) All groups have at least one member.
(d) Ea h person must belong to one and only one group.
Find all values of n, n
your answer.

 1996, for whi h this is possible.

Justify

Solution: Clearly n  9 sin e ea h of 17 groups must ontain at


least one member. Suppose there are k groups of men and 17 k

101

groups of women; without loss of generality, we assume k  8. If m


is the minimum number of members in a group, then the number of
men in the groups is at most k(m +1), while the number of women is
at least (k + 1)m. As there are the same number as men as women,
we have k(m + 1)  (k + 1)m, so m  k  8, and the maximum
number of ouples is k(k + 1)  72. In fa t, any number of ouples
between 9 and 72 an be distributed: divide the men as evenly as
possible into 8 groups, and divide the women as evenly as possible
into 9 groups. Thus 9  n  72 is the set of a eptable numbers of
ouples.
5. Let a; b and be the lengths of the sides of a triangle. Prove that

a+b + b+ a+ +a b a+ b+

and determine when equality o urs.


Solution: By the triangle inequality, b +

positive. For any positive x; y, we have

a and + a b are

2(x + y)  x + y + 2 xy = ( x + y)2
by the AM-GM inequality, with equality for x = y. Substituting
x = a + b ; y = b + a, we get

a + b + b + a  2 a;

whi h added to the two analogous inequalities yields the desired


result. Inequality holds for a + b = b + a = + a b, i.e.
a = b = .

102

2.2 Austrian-Polish Mathemati s Competition


1. Let k  1 be an integer. Show that there are exa tly 3k 1 positive
integers n with the following properties:
(a)
(b)
( )
(d)

The de imal representation of n onsists of exa tly k digits.


All digits of k are odd.
The number n is divisible by 5.
The number m = n=5 has k odd (de imal) digits.

Solution: The multipli ation in ea h pla e must produ e an even

number of arries, sin e these will be added to 5 in the next pla e


and an odd digit must result. Hen e all of the digits of m must be
1; 5 or 9, and the rst digit must be 1, sin e m and n have the same
number of de imal digits. Hen e there are 3k 1 hoi es for m and
hen e for n.
2. A onvex hexagon ABCDEF satis es the following onditions:

(a) Opposite sides are parallel (i.e. AB k DE , BC k EF , CD k


F A).
(b) The distan es between opposite sides are equal (i.e.
d(AB; DE ) = d(BC; EF ) = d(CD; F A), where d(g; h) denotes
the distan e between lines g and h).
( ) The angles \F AB and \CDE are right.
Show that diagonals BE and CF interse t at an angle of 45.

Solution: The onditions imply that A and D are opposite verti es

of a square AP DQ su h that B; C; E; F lie on AP; P D; DQ; QA,


respe tively, and that all six sides of the hexagon are tangent to the
ins ribed ir le of the square. The diagonals BE and CF meet at
the enter O of the square. Let T; U; V be the feet of perpendi ulars
from O to AB; BC; CD; then \T OB = \BOU by re e tion a ross
OB , and similarly \UOC = \COV . Therefore =2 = 2\BOC ,
proving the laim.
3. The polynomials Pn (x) are de ned by P0 (x) = 0, P1 (x) = x and

Pn (x) = xPn 1 (x) + (1 x)Pn 2 (x)


103

n  2:

For every natural number n  1, nd all real numbers x satisfying


the equation Pn (x) = 0.
Solution: One shows by indu tion that

Pn (x) =

x
[(x 1)n 1:
x 2

Hen e Pn (x) = 0 if and only if x = 0 or x = 1 + e2ik=n for some


k 2 f1; : : : ; n 1g.
4. The real numbers x; y; z; t satisfy the equalities x + y + z + t = 0 and
x2 + y2 + z 2 + t2 = 1. Prove that
1  xy + yz + zt + tx  0:
Solution: The inner expression is (x + z )(y + t) = (x + z )2 , so

the se ond inequality is obvious. As for the rst, note that

1
1 = (x2 + z 2) + (y2 + t2 )  [(x + z )2 + (y + t)2  j(x + z )(y + t)j
2
by two appli ations of the power mean inequality.
5. A onvex polyhedron P and a sphere S are situated in spa e su h
that S inter epts on ea h edge AB of P a segment XY with AX =
XY = Y B = 13 AB . Prove that there exists a sphere T tangent to
all edges of P .
Solution: Let AB and BC be two edges of the polyhedron, so that
the sphere meets AB in a segment XY with AX = XY = Y B and
meets BC in a segment ZW with BZ = ZW = W C . In the plane
ABC , the points X; Y; Z; W lie on the ross-se tion of the sphere,
whi h is a ir le. Therefore BY  BX = BZ  BW by power-ofa-point; this learly implies AB = BC , and so the enter of S is
equidistant from AB and BC . We on lude that any two edges of
P are equidistant from S , and so there is a sphere on entri with
S tangent to all edges.

104

6. Natural numbers k; n are given su h that 1 < k < n. Solve the


system of n equations

x3i (x2i +    + x2i+k 1 ) = x2i

1in

in n real unknowns x1 ; : : : ; xn . (Note: x0 = xn ; x1 = xn+1 ; et .)


Solution: The only solution is x1 =    xn = k 1=3 . Let L and M

be the smallest and largest of the xi , respe tively. If M = xi , then

kM 3L2  x3i (x2i +    + x2i+k 1 ) = x2i

M

 1=(kL ). Similarly, if L = xj , then


kL M  xj (xj +    + xj k ) = xj  L
and so L  1=(kM ). Putting this together, we get

and so M

2
+

L

1
kM 2

 kL

and so L  k 1=3 ; similarly, M  k 1=3 . Obviously L  M , so we


have L = M = k 1=3 and x1 =    = xn = k 1=3 .
7. Show that there do not exist nonnegative integers k and m su h that
k! + 48 = 48(k + 1)m .
Solution: Suppose su h k; m exist. We must have 48jk !, so k  6;
one he ks that k = 6 does not yield a solution, so k  7. In that
ase k! is divisible by 32 and by 9, so that (k! + 48)=48 is relatively
prime to 6, as then is k + 1.
If k + 1 is not prime, it has a prime divisor greater than 3, but
this prime divides k! and not k! + 48. Hen e k + 1 is prime, and by
Wilson's theorem k!+1 is a multiple of k +1. Sin e k!+48 is as well,
we nd k + 1 = 47, and we need only he k that 46!=48 + 1 is not a
power of 47. We he k that 46!=48 + 1  29 (mod 53) (by an elling
as many terms as possible in 46! before multiplying), but that 47 has
order 13 modulo 53 and that none of its powers is ongruent to 29
modulo 53.

105

8. Show that there is no polynomial P (x) of degree 998 with real oe ients satisfying the equation P (x)2 1 = P (x2 + 1) for all real
numbers x.
The equation implies P (x) = P ( x); sin e P has
even degree, it must be an even polynomial, that is, P (x) = Q(x2 )
for some polynomial Q of degree 499. Then Q(t)2 1 = Q(t2 +2t +1)
for in nitely many values of t (namely t  0), so this equation is also
a polynomial identity. However, it implies that Q(t) = Q( 2 t);
if we put R(t) = Q(t 1), we have R(t) = R( t), so that R is an
odd polynomial. In parti ular, R(0) = 0, so Q( 1) = 0. But now
we nd Q(1) = 1; Q(4) = 0; Q(25) = 1; : : :; this pro ess produ es
in nitely many zeroes of Q, a ontradi tion.
Solution:

9. We are given a olle tion of re tangular bri ks, no one of whi h is


a ube. The edge lengths are integers. For every triple of positive
integers (a; b; ), not all equal, there is a su ient supply of a  b 
bri ks. Suppose that the bri ks ompletely tile a 10  10  10 box.
(a) Assume that at least 100 bri ks have been used. Prove that
there exist at least two parallel bri ks, that is, if AB is an edge
of one of the bri ks, A0 B 0 is an edge of the other and AB k A0 B 0 ,
then AB = A0 B 0 .
(b) Prove the same statement with 100 repla ed by a smaller number. The smaller the number, the better the solution.
Solution: We prove the laim with 97 bri ks. For ea h integer up

to 16, we tabulate the number of nonparallel bri ks of that volume


(disallowing ubi al bri ks and bri ks with a dimension greater than
10) and their total volume:
Volume 2 3 4 5 6 7 8 9 10 12 14 15 16
Number 3 3 6 3 9 3 9 6 9 15 6 6 12
Total
6 9 24 15 54 21 72 54 90 180 74 90 192
Assuming no two bri ks are parallel, the 90 smallest bri ks have total
volume 891. The 7 other bri ks ea h have volume at least 18, giving
a total volume of at least 1017, a ontradi tion.
106

We have not determined the optimal onstant (one an improve the


above bound to 96 easily), but we note that an arrangement with 73
nonparallel bri ks is possible.

107

2.3 Balkan Mathemati al Olympiad


1. Let O and G be the ir um enter and entroid, respe tively, of triangle ABC . If R is the ir umradius and r the inradius of ABC ,
show that
p
OG  R(R 2r):
Solution: Using ve tors with origin at O, we note that

1
2
1
OG2 = (A + B + C )2 = R2 + R2 ( os 2A + os 2B + os 2C ):
9
3
9
Hen e R2 OG2 = (a2 + b2 + 2 )=9. On the other hand, by the
standard area formula K = rs = ab =4R, we have 2rR = ab =(a +
b + ). We now note that
(a2 + b2 + 2 )(a + b + )  9ab
by two appli ations of the AM-GM inequality, so 2rR  R2 OG2 ,
proving the laim.
2. Let p > 5 be a prime number and X = fp n2 jn 2 N ; n2 < pg.
Prove that X ontains two distin t elements x; y su h that x 6= 1
and x divides y.
Write p = m2 + k with k  2m. If 1 < k < 2m and
k is either odd or a multiple of 4, we an write k = a(2m a) =
m2 (m a)2 , and then kjp2 (m a)2 . If k is even but not a
multiple of 4, write 2k = a(2m a) and pro eed as above, whi h
still works be ause 2k < m2 for p > 5.
We an't have k = 2m sin e m2 + 2m = m(m + 2) is omposite, so
the only ase left is p = m2 +1. In this ase let t = 2m = p (m 1)2
and write either t or 2t as a di eren e of squares (m 1)2 (m a)2 ;
this still works be ause 2t < (m 1)2 for p  7.
Solution:

3. Let ABCDE be a onvex pentagon, and let M; N; P; Q; R be the


midpoints of sides AB; BC; CD; DE; EA, respe tively. If the segments AP; BQ; CR; DM have a ommon point, show that this point
also lies on EN .
108

Solution: Let T be the ommon point, whi h we take as the origin

of a ve tor system. Then A  P = 0, or equivalently A  (C + D) = 0,


whi h we may write A  C = D  A. Similarly, we have B  D =
E  B , C  E = A  C , D  A = B  D. Putting these equalities
together gives E  B = C  E , or E  (B + C ) = 0, whi h means
the line EN also passes through the origin T .

4. Show that there exists a subset A of the set f1; 2; : : : ; 1996g having
the following properties:
(a) 1; 21996 1 2 A;
(b) every element of A, ex ept 1, is the sum of two (not ne essarily
distin t) elements of A;
( ) A ontains at most 2012 elements.
Solution: We state the problem a bit di erently: we want to
write down at most 2012 numbers, starting with 1 and ending with
21996 1, su h that every number written is the sum of two numbers
previously written. If 2n 1 has been written, then 2n (2n 1)
an be obtained by n doublings, and 22n 1 an be obtained in
one more step. Hen e we an obtain 22 1; 24 1; : : : ; 2256 1
in (1 + 1) + (2 + 1) +    + (128 + 1) = 263 steps. In 243 steps,
we turn 2256 1 into 2499 2243 . Now noti e that the numbers
2243 2115; 2115 251; 251 219; 219 23; 23 21; 21 1 have all been
written down; in 6 steps, we now obtain 2499 1. We make this into
2998 1 in 500 steps, and make 21996 1 in 999 steps. Adding 1 for
the initial 1, we ount

1 + 263 + 243 + 6 + 500 + 999 = 2012


numbers written down, as desired.

109

2.4 Cze h-Slovak Mat h

1. Let Z denote the set of nonzero integers. Show that an integer p > 3
is prime if and only if for any a; b 2 Z, exa tly one of the numbers
p 1
p+1
N1 = a + b 6ab +
; N2 = a + b + 6ab +
6
6

belongs to Z .
Solution: If N1 = 0, then p = (6a

1)(6b 1) is omposite; similarly, N2 = 0 implies p = (6a +1)(6b +1) is omposite. Conversely,


suppose that p is omposite. If p  0; 2; 3 or 4 (mod 6), then N1 and
N2 are not integers. Otherwise, all divisors of p are ongruent to
1 (mod 6), so there exist natural numbers ; d su h that

p = (6 + 1)(6d + 1)or(6 1)(6d 1)or(6 + 1)(6d 1):


In the rst ase, N2 is not an integer and N1 = 0 for a = ; b = d.
In the se ond ase, N2 is not an integer and N1 = 0 for a = ; b = d.
In the third ase, N1 is not an integer and N2 = 0 for a = ; b = d.

2. Let M be a nonempty set and  a binary operation on M . That is,


to ea h pair (a; b) 2 M  M one assigns an element a  b. Suppose
further that for any a; b 2 M ,
(a  b)  b = a

and

a  (a  b) = b:

(a) Show that a  b = b  a for all a; b 2 M .


(b) For whi h nite sets M does su h a binary operation exist?
Solution:

(a) First note that [a  (a  b)  (a  b) = a by the rst rule. By


the se ond rule, we may rewrite the left side as b  (a  b), so
b  (a  b) = a and so b  a = b  [b  (a  b). By the se ond rule,
this equals a  b, so a  b = b  a.
(b) Su h sets exist for all nite sets M . Identify M with f1; : : : ; ng
and de ne

a  b = , a + b +  0 (mod n):

It is immediate that the axioms are satis ed.


110

3. A pyramid  is given whose


p base is a square of side 2a and whose
lateral edges have length a 17. Let M be a point in the interior of
the pyramid, and for ea h fa e of , onsider the pyramid similar to 
whose vertex is M and whose base lies in the plane of the fa e. Show
that the sum of the surfa e areas of these ve pyramids is greater
than or equal to one- fth the surfa e area of , and determine for
whi h M equality holds.
Solution: All fa es of  have the same area S1 = 4a2 . The
segments onne ting the point M with the verti es of the pyramid
partition  into a quadrilateral pyramid and four tetrahedra. Let
v1 ; : : : ; v5 denote the heights of these ve bodies from vertex M ;
then the volumes of the bodies sum to the volume of , whi h means

1
1
S1 vi = S1 v:
3
3
i=1

5
X

In other words, vi = v. On the other hand, the vi are also the


heights of the small pyramids similar to , so
5
X
vi

i=1

=1=

5
X

i=1

ki =

5
X

i=1

Si
;
S

where S denotes the surfa e area of , Si that of the i-th small


pyramid, and ki the oe ient of similarity between the i-th pyramid
and . We on lude

S=

5
X
p

i=1

!2

Si

5

5
X

i=1

Si

by the power mean (or Cau hy-S hwarz) inequality. Equality holds
only when all of the Si are equal, as are the vi , whi h o urs when
M is the enter of the ins ribed sphere of the pyramid.

4. Determine whether there exists a fun tion f : Z ! Z su h that for


ea h k = 0; 1; : : : ; 1996 and for ea h m 2 Z the equation f (x) + bx =
m has at least one solution x 2 Z.
Solution: Ea h integer y an be written uniquely as 1997m + k
with m 2 Z and k 2 f0; : : : ; 1996g. De ne the fun tion f by f (y) =

111

m ky; then f (x) + kx = m has the solution x = 1997m + k, so the


ondition is satis ed.
5. Two sets of intervals A; B on a line are given. The set A ontains
2m 1 intervals, every two of whi h have a ommon interior point.
Moreover, ea h interval in A ontains at least two disjoint intervals
of B . Show that there exists an interval in B whi h belongs to at
least m intervals from A.
Solution: Let i = [ai ; bi (i = 1; : : : ; 2m 1) be the intervals,
indexed so that a1  a2  : : :  a2m 1 . Choose k 2 fm; : : : ; 2m 1g
to minimize bk . By assumption, the interval k ontains two disjoint
intervals from B , say 1 = [ 1 ; d1 and 2 = [ 2 ; d2 . Without loss of
generality, assume

ak  1 < d1 < 2 < d2  bk :


If d1  bi for i = 1; 2; : : : ; m, then 1  i for i = 1; 2; : : : ; m,
so 1 satis es the desired property. Otherwise, d1 > bs for some
s 2 f1; 2; : : : ; mg. By assumption, 2 > d1 > bs . Sin e no two of the
are disjoint, we have bs  ai for all i, so 2 > ai . On the other
hand, by the hoi e of k, bk  bi for i = m; : : : ; 2m1. Therefore
ai < 2 < d2  bk  bi for ea h i 2 fm; m + 1; : : : ; 2m 1g, and so
2 has the desired property.
6. The points E and D lie in the interior of sides AC and BC , respe tively, of a triangle ABC . Let F be the interse tion of the lines
AD and BE . Show that the area of the triangles ABC and ABF
satis es
SABC jAC j jBC j
=
+
1:
SABF jAE j jBDj
Solution: Let the line parallel to BC through F meet AB at K
and AC at N ; let the line parallel to CA through F meet BC at
M and AB at P ; let the line parallel to AB through F meet BC
at L and CA at O. Let vC and vF be the distan es of C and F ,
respe tively, to the line AB . Then

SABC vC BC BL + LM + MC
=
=
=
:
SABF vF F K
FK
112

Under the homothety through B arrying F to E , the segment P M


maps to AC . Thus

LM F M EC AC
=
=
=
F K F P AC AE
and similarly

CM NF CD BC
=
=
=
1:
F K F K BD BD
The required assertion follows by putting this all together and noting
BL = F K .

113

2.5 Iberoameri an Olympiad


1. Let n be a natural number. A ube of side length n an be divided into 1996 ubes whose side lengths are also natural numbers.
Determine the smallest possible value of n.
Sin e 1996 > 123, we must have n  13, and we now
show n = 13 su es. Inside a ube of edge 13, we pla e one ube of
edge 5, one ube of length 4, and 2 of length 2, and ll the remainder
with ubes of edge 1. The number of ubes used is
Solution:

133 (53 1) (43 1) 2(23 1) = 2197 124 63 2(7) = 1996;


as desired.
2. Let M be the midpoint of the median AD of triangle ABC . The line
BM interse ts side AC at the point N . Show that AB is tangent to
the ir um ir le of NB if and only if the following equality holds:
BM BC 2
:
=
BN BN 2
Solution: First note that (by the Law of Sines in triangles ABM

and AMN )

BM sin \MAB sin \MNA


=
:
MN sin \ABM sin \NAM
Then note that (by the Law of Sines in triangles ABD and ADC )
sin \MAB BD sin \ABD
=
:
sin \NAM DC sin \DCA
By the Law of Sines in triangle BNC ,
BC 2 sin2 \BNC
;
=
BN 2 sin2 \BCN
therefore BM=MN = BC 2 =BN 2 if and only if
sin \ABD sin \BNC
=
;
sin \ABM sin \BCN
whi h if we put = \ABM; = \BCN;  = \NBC be omes
sin( + ) sin = sin( + ) sin :
114

Rewriting ea h side as a di eren e of osines and an elling, this


be omes
os( +  ) = os( + ):
Both angles in this equation are between  and , so the angles
are either equal or negatives of ea h other. The latter implies  = 0,
whi h is untrue, so we dedu e = , and so BM=MN = BC 2 =BN 2
if and only if \ABM = \BCN , that is, if AB is tangent to the
ir um ir le of BNC .
3. We have a square table of k2 k + 1 rows and k2 k + 1 olumns,
where k = p + 1 and p is a prime number. For ea h prime p, give
a method of distributing the numbers 0 and 1, one number in ea h
square of the table, su h that in ea h row and olumn there are
exa tly k zeroes, and moreover no re tangle with sides parallel to
the sides of the table has zeroes at all four orners.
Solution: The proje tive plane of order p is de ned as the set of
equivalen e lasses in the set (Z mod p)3 (0; 0; 0) where (a; b; ) is
equivalent to (ma; mb) whenever m is oprime to p. Label the rows
and olumns by elements of the proje tive plane, and pla e a 1 in
row (a; b; ) and olumn (d; e; f ) if ad + be + f = 0; then the desired
ondition is immediately veri ed.

This holds for n = 2, and we prove it in general by


indu tion on n. Assume the result for n 1. The fra tions newly
added are those with b = n and a relatively prime to n. Those
removed have a + b = n. Now note that the added fra tions 1=an
and 1=bn pre isely an el the removed fra tion 1=ab, so the sum
remains un hanged.
Solution:

4. Given a natural number n  2, onsider all of the fra tions of the


form ab1 , where a and b are relatively prime natural numbers su h
that a < b  n and a + b > n. Show that the sum of these fra tions
is 1=2.
5. Three ounters A; B; C are pla ed at the orners of an equilateral
triangle of side n. The triangle is divided into triangles of side length
1. Initially all lines of the gure are painted blue. The ounters move
115

along the lines, painting their paths red, a ording to the following
rules:
(i) First A moves, then B , then C , then A, and so on in su esion.
On ea h turn, ea h ounter moves the full length of a side of
one of the short triangles.
(ii) No ounter may retra e a segment already painted red, though
it an stop on a red vertex, even if another ounter is already
there.
Show that for all integers n > 0 it is possible to paint all of the
segments red in this fashion.
Solution: The ases n = 1; 2 are trivial; we use them as the base
ases for an indu tive proof. We des ribe the moves for A, understanding that the moves for B and C are the same moves rotated
by 2=3 and 4=3, respe tively. To x dire tions, imagine the triangle is oriented with one side parallel to the horizontal and the third
vertex above it, and suppose A starts at the bottom left. We rst
move A right for n 1 steps. We then alternate moving it up to the
left and down to the left for a total of 2n 5 steps. We then tra e
a path through the inner triangle of side n 2 using the indu tion
hypothesis, ending at another orner. Finally, we follow the unused
edges from that orner, ending three steps later.

6. In the plane are given n distin t points A1 ; : : : ; An , and to ea h point


Ai is assigned a nonzero real number i su h that (Ai Aj )2 = i + j
for all i 6= j . Show that
(a) n  4;
(b) If n = 4, then 11 + 12 + 13 + 14 = 0.

Solution: For any four points Ai ; Aj ; Ak ; Am , we have

Ai A2k

Ak A2j = Ai A2m Am A2j = i

j

and by an elementary lemma, this means Ai Aj is perpendi ular to


Ak Am . Sin e this holds for all permutations of i; j; k; m, we on lude
that Am is the ortho enter of triangle Ai Aj Ak , so that in parti ular
no other points an be given, hen e n  4.
116

Now suppose n = 4; without loss of generality, assume that A1 A2 A3


form an a ute triangle inside whi h A4 lies. Note that
21 = A1 A22 + A1 A23

A1 A23 = 2A1 A2  A1 A3 os \A3 A1 A2 :

Using this and analogous formulae, and that \A1 A3 A4 = \A1 A2 A4


and \A3 A1 A2 =  \A3 A4 A2 , we get

1 2
(A1 A2  A1 A3 os \A3 A1 A2 )(A1 A2  A2 A4 os \A1 A2 A4 )
=
3 4
(A3 A1  A3 A4 os \A1 A3 A4 )(A2 A4  A3 A4 os \A2 A4 A3 )
A1 A22 2
 +
=
= 1 2:
A3 A4
3 + 4
Therefore 1=1 + 1=2 + 1=3 + 1=4 = 0, as desired.

117

2.6 St. Petersburg City Mathemati al Olympiad


1. Several one-digit numbers are written on a bla kboard. One an
repla e any one of the numbers by the last digit of the sum of all
of the numbers. Prove that the initial olle tion of numbers an be
re overed by a sequen e of su h operations.
Solution: At ea h step, erase the leftmost number and write the

sum on the right, so that (x1 ; : : : ; xn ) be omes (x2 ; : : : ; xn 1 ; x1 +

   + xn ). Sin e there are nitely many n-tuples, some n-tuple must

repeat, and the rst su h must be the original on guration, sin e


ea h n-tuple (y1 ; : : : ; yn) omes from a unique n-tuple, namely (yn
y1    yn 1 ; y1 ; : : : ; yn 1).

2. Fifty numbers are hosen from the set f1; : : : ; 99g, no two of whi h
sum to 99 or 100. Prove that the hosen numbers must be 50; 51; : : : ;
99.
Solution: In the sequen e

99; 1; 98; 2; 97; 3; : : :; 51; 49; 50;


any two adja ent numbers sum to 99 or 100, so both annot o ur.
Grouping the numbers into 49 pairs plus one extra, we see at most
50 numbers an o ur, and 50 must be one of them. Sin e we must
step at least two terms along the list to make the next hoi e, the
numbers must indeed be 50; 51; : : : ; 99. Clearly we maximize the
number of hosen numbers by taking them two apart, and the list
has odd length, so taking 99; 98; : : : ; 50 is the only Draw a graph
with f1; : : : ; 99g as verti es, where two numbers are adja ent if they
sum to 99 or 100.
3. Let M be the interse tion of the diagonals of the trapezoid ABCD.
A point P su h that \AP M = \DP M is hosen on the base BC .
Prove that the distan e from C to the line AP is equal to the distan e
from B to the line DP .
Solution:

Sin e M lies on the internal angle bise tor of angle

\AP D, it lies at the same distan e from the lines AP and DP . The
ratio of this distan e to the distan e from C to AP is AM=AC , while
118

the ratio of this distan e to the distan e from B to DP is BM=MD.


But AM=MC = BM=MD by similar triangles, so the latter two
distan es are indeed the same.
4. In a group of several people, some are a quainted with ea h other
and some are not. Every evening, one person invites all of his a quaintan es to a party and introdu es them to ea h other. Suppose
that after ea h person has arranged at least one party, some two people are still una quainted. Prove that they will not be introdu ed at
the next party.
Solution: We laim that two people una quainted after ea h per-

son has held at least one party lie in di erent onne ted omponents
of the original (and nal) graph of a quaintan e. If two people are
onne ted by a path of length n, they will be onne ted by a path
of length n 1 after one person along the path (in luding either of
the two people at the ends) holds a party, by a path of length n 2
after two of them hold a party, and so on. After ea h person holds
a party, the two people on the ends will be a quainted.
5. Let M be the interse tion of the diagonals of a y li quadrilateral,
N the interse tions of the lines joining the midpoints of opposite
sides, and O the ir um enter. Prove that OM  ON .
Solution: We use ve tors. If A; B; C; D are the verti es of the
quadrilateral in order, then N = (A + B + C + D)=4; in parti ular, if
E and F are the midpoints of AC and BD, respe tively, then N is
the midpoint of EF . The ir le with diameter OM passes through E
and F , so OM  OE and OM  OF ; moreover, in any triangle, the
median to a side is no longer than the average of the other two sides
(rotate the triangle by  about the foot of the median, so twi e the
median be omes a diagonal of a parallelogram, and use the triangle
inequality). Hen e OM  ON .
6. Prove that for every polynomial P (x) of degree 10 with integer oe ients, there is an in nite (in both dire tions) arithmeti progression
whi h does not ontain P (k) for any integer k.

Sin e P is not linear, there exists x su h that P (x +


P (x) = n > 1. Sin e P (t + k)  P (t) (mod k), ea h value of

Solution:

1)

119

P is ongruent to one of P (x); P (x + 1); : : : ; P (x + n 1) modulo


n. However, sin e P (x + 1) P (x) = n, the values of P over at
most n 1 distin t residue lasses, and so there is an arithmeti
progression of di eren e n ontaining no values of P .
7. There are n parking spa es along a one-way road down whi h n
drivers are traveling. Ea h driver goes to his favorite parking spa e
and parks there if it is free; otherwise, he parks at the nearest free
pla e down the road. If there is no free spa e after his favorite, he
drives away. How many lists a1 ; : : : ; an of favorite parking spa es are
there whi h permit all of the drivers to park?
Solution: There are (n + 1)n

su h lists. To ea h list of preferen es (a1 ; : : : ; an ) whi h allows all drivers to park, asso iate the list
(b2 ; : : : ; bn ), where bi is the di eren e mod n + 1 between the numbers of the spa e driver i wants and the spa e the previous driver
took. Clearly any two lists give rise to di erent sequen es of bi .
We now argue that any list of bi omes from a list of preferen es.
Imagine that the n parking spa es are arranged in a ir le with an
extra phantom spa e put in at the end. Put the rst driver in any
spa e, then for i = 2; : : : ; n, put driver i in the rst available spa e
after the spa e bi away from the spa e taken by driver i 1; this
gives a list of preferen es if and only if the one spa e not taken at
the end is the phantom spa e. However, by shifting the position of
the rst driver, we an always ensure that the phantom spa e is the
spa e not taken.
Thus the sequen es of bi are equal in number to the lists of preferen es, so there are (n + 1)n 1 of ea h.
1

8. Find all positive integers n su h that 3n

+ 5n

divides 3n + 5n .

Solution: This only o urs for n = 1. Let sn = 3n + 5n and note

that

sn = (3 + 5)sn 1 3  5  sn 2
so sn 1 must also divide 3  5  sn 2 . If n > 1, then sn 1 is oprime
to 3 and 5, so sn 1 must divide sn 2 , whi h is impossible sin e
sn 1 > sn 2 .
120

9. Let M be the midpoint of side BC of triangle ABC , and let r1 and


r2 be the radii of the in ir les of triangles ABM and ACM . Prove
that r1 < 2r2 .
Re all that the area of a triangle equals its inradius
times half its perimeter. Sin e ABM and ACM have equal area, we
have
r1 AC + AM + CM
=
r2 AB + AM + BM
and it su es to show AC + AM + CM < 2AB + 2AM + 2BM ;
sin e BM = CM , this simpli es to AC < 2AB + AM + CM . In
fa t, by the triangle inequality, AC < AM + CM , so we are done.
Solution:

10. Several positive integers are written on a bla kboard. One an erase
any two distin t integers and write their greatest ommon divisor and
least ommon multiple instead. Prove that eventually the numbers
will stop hanging.
Solution: If a; b are erased and < d are written instead, we have
 min(a; b) and d  max(a; b); moreover, ab = d. From this we
may on lude a + b  + d by writing ab + a2 = d + a2  a + ad (the
latter sin e (d a)( a)  0) and dividing both sides by a. Thus
the sum of the numbers never de reases, and it is obviously bounded
(e.g. by n times the produ t of the numbers, where n is the number
of numbers on the board); hen e it eventually stops hanging, at
whi h time the numbers never hange.

11. No three diagonals of a onvex 1996-gon meet in a point. Prove that


the number of triangles lying in the interior of the 1996-gon and
having sides on its diagonals is divisible by 11.
Solution: There is exa tly one su h triangle for ea h hoi e of six
verti es of the 1996-gon: if A; B; C; D; E; F are the six verti es in
order, the orresponding triangle is formed
 by the lines AD; BE; CF .
Hen e the number of triangles is 1996
; sin e 1991 is a multiple of
6
11, so is the number of triangles.

12. Prove that for every polynomial x2 + px + q with integer oe ients,


there exists a polynomial 2x2 + rx + s with integer oe iets su h
121

that the sets of values of the two polynomials on the integers are
disjoint.
Solution: If p is odd, then x2 + px + q has the same parity as q for

all integers x, and it su es to hoose r even and s of the opposite


parity as q. If p = 2m is even, then x2 + px + q = (x + m)2 +(q m2 )
whi h is ongruent to q m2 or q m2 + 1 modulo 4. Now it su es
to hoose r even and s ongruent to q m2 + 2 modulo 4.
13. In a onvex pentagon ABCDE , AB = BC , \ABE + \DBC =
\EBD, and \AEB + \BDC = . Prove that the ortho enter of
triangle BDE lies on AC .
Solution: By the assumption \AEB + \BDC =  , there exists a point F on AC su h that \AF B = \AEB and \BF C =
\BDC ; this means F is the se ond interse tion of the ir um ir les
of BCD and ABE . The triangle ABC is isos eles, so \F CB =
( \ABC )=2. Hen e \F DB = \F CB = =2 \DBE by the assumption \ABE + \DBC = \EBD, and so DF ? BE . Similarly
EF ? BD, and so F is the ortho enter of BDE .

14. In a federation onsisting of two republi s, ea h pair of ities is linked


by a one-way road, and ea h ity an be rea hed from ea h other
ity by these roads. The Hamilton travel agen y provides n di erent
tours of the ities of the rst republi (visiting ea h ity on e and
returning to the starting ity without leaving the republi ) and m
tours of the se ond republi . Prove that Hamilton an o er mn su h
tours around the whole federation.
Solution: From ea h pair of tours, we onstru t a tour of both
republi s from whi h the two original tours an be re onstru ted
(so in parti ular, all su h pairs will be distin t). We rst look for
ities u; v in the rst republi and w; x in the se ond, su h that u
immediately pre edes v in the rst tour, x immediately pre edes w
in the se ond, and there are roads from u to w and from x to v. In
that ase, we may start from v, tour the rst republi ending at u,
go to w, tour the se ond republi ending at x, and return to v. Call
this a \dire t" tour; the two original tours an be determined from

122

a dire t tour by reading o the ities of either republi in the order


they appear.
Now suppose no dire t tour exists; without loss of generality, assume
the number of ities in the se ond republi does not ex eed the number in the rst republi . In this ase, whenever u is some number of
ities ahead of v in the rst tour, and x is the same number ahead
of w in the se ond tour, the \parallel" roads uw and vx must point
in the same dire tion (by repeated appli ation of the above observation). Sin e ea h ity is rea hable from ea h other ity, given a
vertex v, we an nd x immediately pre eding w in the se ond tour
su h that there are roads from w to v and from v to x. We now
form an \alternating" tour starting at v by alternately taking roads
parallel to vx and wv; on e the se ond republi is exhausted, we
tour the remaining ities of the rst republi in order, returning to
v. The two original tours an be determined from an alternating
tour by reading o the ities of the rst republi in the order they
appear, and those of the se ond republi in the reverse of the order
in whi h they appear.
15. Sergey found 11 di erent solutions to the equation f (19x 96=x) = 0.
Prove that if he had tried harder, he ould have found at least one
more solution.
Solution: The equation 19x

96=x = t an be rewritten 19x2


tx 96 = 0; sin e t2 + 19  96 > 0, it always has two real roots.
Therefore the number of zeroes of f (if nite) is an even integer, so
Sergey an nd at least one more zero.
16. The numbers 1; 2; : : : ; 2n are divided into two groups of n numbers.
Prove that the pairwise sums of numbers in ea h group (the sum of
ea h number with itself in luded) have the same remainders upon
division by 2n. (Note: ea h pair of distin t numbers should be added
twi e, and ea h remainder must o ur the same number of times in
the two groups.)
Solution:
P Let S and T be the groups, and let P (x) =

Q(x) =

j 2T

xj ;

the laim amounts to showing

P (x)2  Q(x)2 (mod x2n


123

1):

i
i2S x

and

This follows by noting that x 1jP (x) Q(x) sin e both groups have
the same number of elements, and that x2n 1 +    + 1jP (x) + Q(x)
sin e ea h of the numbers 1; : : : ; 2n o urs exa tly on e.
17. The points A0 and C 0 are hosen on the diagonal BD of a parallelogram ABCD so that AA0 k CC 0 . The point K lies on the segment
A0 C , and the line AK meets CC 0 at L. A line parallel to BC is
drawn through K , and a line parallel to BD is drawn through C ;
these meet at M . Prove that D; M; L are ollinear.
Solution: Let M 0 and M 00 be the interse tions of DM with MK

and MC , respe tively. Sin e M 0 K k DA, we have LM 0 =LD =


LK=LA. Sin e CK k C 0 A, we have LK=LA = LC=LC 0. Finally, sin e CM k C 0 D, we have LM 00=LD = LC=LC 0. Therefore
LM 0=LD = LM 00=LD and so M 0 = M 00 = M .
18. Find all quadruples of polynomials P1 (x); P2 (x); P3 (x); P4 (x) with
real oe ients su h that for ea h quadruple of integers x; y; z; t su h
that xy zt = 1, one has

P1 (x)P2 (y) P3 (z )P4 (t) = 1:


If P1 (1) = 0, then P3 (z )P4 (t) = 1 for ea h pair of
integers z; t, and so P3 and P4 are onstant fun tions; moreover,
P1 (x)P2 (y) = 0, so one of P1 and P2 is identi ally zero. Ignoring
su h ases, whi h are easily enumerated, we assume Pi (1) 6= 0 for all
i.
We rst note that P1 (x)P2 (1) = P1 (1)P2 (x) for all nonzero integers
x, so that P1 and P2 are equal up to a s alar fa tor; similarly, P3 and
P4 are equal up to a s alar fa tor. Now note that P1 (x)P2 (ay) =
P1 (ax)P2 (y) for all nonzero a; x; y, so that the di eren e between
the two sides is identi ally zero as a polynomial in a. In parti ular,
that means no term in P1 (x)P2 (y) has unequal exponent in x and
y, and the same is true of P1 (x)P1 (y). On the other hand, if P1 (x)
has terms of more than one degree, then P1 (x)P1 (y) ontains a term
with di erent degrees in x and y. Hen e P1 (x) = xk for some
integer k and some onstant , and similarly P2 (x) = dxk , P3 (x) =
exm ; P4 (x) = fxm .
Solution:

124

Thus we must determine when dxk yk efz mtm = 1 whenever xy


zt = 1 in integers. Clearly k = m sin e otherwise one of the two
terms on the left dominates the other, and d = 1 by setting x = y =
1 and z = t = 0, and similarly ef = 1. Now note that (xy)k (zt)k =
1 an only happen in general for k = 1, sin e for k > 1, there are no
onse utive perfe t k-th powers. We on lude P1 (x) = x; P2 (x) =
x= ; P3 (x) = ex; P4 (x) = x=e for some nonzero real numbers ; e.
19. Two players play the following game on a 100  100 board. The
rst player marks a free square, then the se ond player puts a 1  2
domino down overing two free squares, one of whi h is marked. This
ontinues until one player is unable to move. The rst player wins if
the entire board is overed, otherwise the se ond player wins. Whi h
player has a winning strategy?
Solution: The rst player has a winning strategy. Let us say
a position is stable if every square below or to the right of a free
square is free. Then we laim the rst player an always ensure that
on his turn, either the position is stable or there is a free square with
exa tly one free neighbor (or both).
Let us label the square in the i-th row and j -th olumn as (i; j ), with
(1; 1) in the top left. We all a free square a orner if is not below or
to the right of another free square. Let (a1 ; b1), (a2 ; b2), : : :, (ak ; bk )
be the orners from top to bottom.
First noti e that if (a; b) is a orner su h that both (a + 1; b 1) and
(a 1; b + 1) are nonfree (or o the board), then the rst player may
mark (a; b), and however the se ond player moves, the result will be a
stable position. More generally, if (a; b); (a +1; b 1);    ; (a + k; b k)
are orners and (a 1; b +1) and (a + k +1; b k 1) are both nonfree
or o the board, the rst player an be sure to return to a stable
position.
To show this, rst note that we annot have both a = 1 and b
k = 1, or else the number of nonfree squares would be odd, whi h
is impossible. Without loss of generality, assume that b k 6= 1
is not the nal orner. The rst player now marks (a; b). If the
se ond player overs (a; b) and (a; b + 1), the position is again stable.
Otherwise, the rst player marks (a + 1; b 1) and the se ond player
is for ed to over it and (a + 2; b 1). Then the rst player marks

125

(a+2; b 2) and the se ond player is for ed to over it and (a+3; b 2),
and so on. After (a+k; b k) is marked, the result is a stable position.
(Note that the assumption b k 6= 1 ensures that the moves des ribed
do not ross the edge of the board.)
To nish the proof, we need to show that su h a hain of orners must
exist. Write the labels (a1 ; b1 ); : : : ; (ak ; bk ) in a row, and join two
adja ent labels by a segment if they are of the form (a; b); (a+1; b 1).
If two adja ent labels (a; b); (a + i; b j ) are not joined by a segment,
then either i = 1 or j = 1 but not both. If i = 1, draw an arrow
between the labels pointing towards (a + i; b j ); otherwise draw
the arrow the other way. Also draw arrows pointing to (a1 ; b1 ) and
(ak ; bk ). There is now one more hain of orners (joined by segments)
than arrows, so some hain has two arrows pointing to it. That hain
satis es the ondition above, so the rst player an use it to reate
another stable position. Consequently, the rst player an ensure
vi tory.
20. Let BD be the bise tor of angle B in triangle ABC . The ir um ir le
of triangle BDC meets AB at E , while the ir um ir le of triangle
ABD meets BC at F . Prove that AE = CF .
Solution: By power-of-a-point, AE  AB = AD  AC and CF  CB =

CD  CA, so AE=CF = (AD=CD)(BC=AB ). However, AB=CB =


AD=CD by the angle bise tor theorem, so AE = CF .

21. A 10  10 table onsists of positive integers su h that for every ve


rows and ve olumns, the sum of the numbers at their interse tions
is even. Prove that all of the integers in the table are even.
Solution: We denote the rst ve entries in a row as the \head"
of that row. We rst show that the sum of ea h head is even. We
are given that the sum of any ve heads is even; by subtra ting two
su h sums overlapping in four heads, we dedu e that the sum of any
two heads is even. Now subtra ting two su h relations from a sum
of ve heads, we determine that the sum of any head is even.
By a similar argument, the sum of any ve entries in a row is even.
By the same argument as above, we dedu e that ea h entry is even.

126

22. Prove that there are no positive integers a and b su h that for ea h
pair p; q of distin t primes greater than 1000, the number ap + bq is
also prime.
Solution: Suppose a; b are so hosen, and let m be a prime greater

than a + b. By Diri hlet's theorem, there exist in nitely many primes


in any nonzero residue lass modulo m; in parti ular, there exists a
pair p; q su h that p  b (mod m); q  a (mod m), giving ap + bq
is divisible by m, a ontradi tion.
23. In triangle ABC , the angle A is 60. A point O is taken inside the
triangle su h that \AOB = \BOC = 120. The points D and E
are the midpoints of sides AB and AC . Prove that the quadrilateral
ADOE is y li .
Sin e \OBA = 60 \OAB = \OAC , the triangles
OAB and OCA are similar, so there is a spiral similarity about O
arrying OAB to OCA. This similarity preserves midpoints, so it
arries D to E , and therefore \AOD = \COE = 120 \AOE .
We on lude \DOE = 120 and so ADOE is y li .
Solution:

24. There are 2000 towns in a ountry, ea h pair of whi h is linked by


a road. The Ministry of Re onstru tion proposed all of the possible assignments of one-way tra to ea h road. The Ministry of
Transportation reje ted ea h assignment that did not allow travel
from any town to any other town. Prove that more of half of the
assignments remained.
Solution: We will prove the same statement for n  6 towns.
First suppose n = 6. In this ase there are 215 assignments, and an
assignment is reje ted only if either one town has road to all of the
others in the same dire tion, or if there are two sets of three towns,
su h that within ea h town the roads point in a ir le, but all of the
roads from one set to the other point in the same dire tion. There
are 5  211 bad assignments of the rst kind and 20  8 of the se ond
kind, so the fra tion of good assignments is at least 5=8.

127

For n  6, we laim that the fra tion of good assignments is at least




5 nY1
1
8 i=6

2i

We show this by indu tion on n: a good assignment on n 1 verti es an be extended to a good assignment on n verti es simply by
avoiding having all edges from the last vertex pointing in the same
dire tion, whi h o urs in 2 ases out of 2n 1.
Now it su es to show that the above expression is more than 1=2.
In fa t,

1
1 i 4
Y
X
1 1
1 i

1+
i
2
i=5
i=5 2
1 i+1
1 X
= 1+ 5
2 i=0 2i
1X
1 1
1 X
= 1+ 5
2 i=0 k=i 2i
1 1
1 X
= 1+ 5
2 i=0 2i 1
4 9
= 1+ 5 =
2
8
Thus the fra tion of good assignments is at least (5=8)(8=9) = 5=9 >
1=2.
25. The positive integers m; n; m; n are written on a bla kboard. A
generalized Eu lidean algorithm is applied to this quadruple as follows: if the numbers x; y; u; v appear on the board and x > y, then
x y; y; u + v; v are written instead; otherwise x; y x; u; v + u are
written instead. The algorithm stops when the numbers in the rst
pair be ome equal (they will equal the greatest ommon divisor of
m and n). Prove that the arithmeti mean of the numbers in the
se ond pair at that moment equals the least ommon multiple of m
and n.
Solution: Note that xv + yu does not hange under the operation, so it remains equal to 2mn throughout. Thus when the rst

128

two numbers both equal g d(m; n), the sum of the latter two is
2mn= g d(m; n) = 2l m(m; n).
26. A set of geometri gures onsists of red equilateral triangles and
blue quadrilaterals with all angles greater than 80 and less than
100. A onvex polygon with all of its angles greater than 60 is
assembled from the gures in the set. Prove that the number of
(entirely) red sides of the polygon is a multiple of 3.
Solution: We rst enumerate the ways to de ompose various angles

into sums of 60 angles (T ) and angles between 80 and 100 (Q):


60 < < 180
= T; 2T; T + Q; 2Q

= 180
= 3T; 2Q

= 360
= 6T; 3T + 2Q; 4Q
(The range for Q annot be in reased, sin e 3Q ranges from 240
to 300; even in luding the endpoints would allow for additional
ombinations above.)
The set of all of the verti es of all of the polygons an be divided
into three ategories, namely those whi h lie in the interior, on an
edge, or at a vertex of the large polygon. The above omputation
shows that the number of T angles at interior or edge verti es is a
multiple of 3; sin e the total number is three times the number of
triangles, we dedu e that the number of T angles at verti es of the
large polygon is also a multiple of 3.
Next note that every edge is entirely of one olor, sin e we annot
have both a T and a Q at a 180 angle. Additionally, no vertex of
the large polygon onsists of more than two angles, and a T annot
o ur by itself. All this means that the number of red sides is half
the number of T angles at the verti es, whi h is a multiple of 3.

27. The positive integers 1; 2; : : : ; n2 are pla ed in some fashion in the


squares of an n  n table. As ea h number is pla ed in a square, the
sum of the numbers already pla ed in the row and olumn ontaining
that square is written on a bla kboard. Give an arrangement of
the numbers that minimizes the sum of the numbers written on the
bla kboard.
129

Solution: Rather than des ribe the arrangement, we demonstrate

it for n = 4:

1
14
11
8

5
2
15
12

9
6
3
16

13
10
7
4

Note that the sum of the numbers written may also be omputed as
the produ t of ea h number with the number of empty spa es in its
row and olumn at the time it was pla ed.
We now simply note that the ontribution from rows is at least that
of the minimal arrangement, and analogously for olumns. This is
be ause we end up multiplying n numbers by ea h of 0; 1; : : : ; n 1.
By the rearrangement inequality, the total is minimizing by multiplying 1; : : : ; n by n 1, n + 1; : : : ; 2n by n 2, and so on.

130

1997 National Contests:


Problems

3.1 Austria
1. Solve the system
(x 1)(y2 + 6) = y(x2 + 1)
(y 1)(x2 + 6) = x(y2 + 1):
2. Consider the sequen e of positive integers whi h satis es an = a2n 1 +
a2n 2 + a2n 3 for all n  3. Prove that if ak = 1997 then k  3.
3. Let k be a positive integer. The sequen e an is de ned by a1 = 1, and
an is the n-th positive integer greater than an 1 whi h is ongruent
to n modulo k. Find an in losed form. What happens if k = 2?
4. Given a parallelogram ABCD, ins ribe in the angle \BAD a ir le
that lies entirely inside the parallelogram. Similarly, ins ribe a ir le
in the angle \BCD that lies entirely inside the parallelogram and
su h that the two ir les are tangent. Find the lo us of the tangen y
point of the ir les, as the two ir les vary.

131

3.2 Bulgaria
1. Find all real numbers m su h that the equation
(x2

2mx 4(m2 + 1))(x2

4x 2m(m2 + 1)) = 0

has exa tly three di erent roots.


2. Let ABC be an equilateral triangle with area 7 and let M; N be
points on sides AB; AC , respe tively, su h that AN = BM . Denote
by O the interse tion of BN and CM . Assume that triangle BOC
has area 2.
(a) Prove that MB=AB equals either 1=3 or 2=3.
(b) Find \AOB .
3. Let f (x) = x2 2ax a2
jf (x)j  1 for all x 2 [0; 1.

3=4. Find all values of a su h that

4. Let I and G be the in enter and entroid, respe tively, of a triangle


ABC with sides AB = , BC = a, CA = b.
(a) Prove that the area of triangle CIG equals ja bjr=6, where r
is the inradius of ABC .
(b) If a = + 1 and b = 1, prove that the lines IG and AB are
parallel, and nd the length of the segment IG.
5. Let n  4 be an even integer and A a subset of f1; 2; : : : ; ng. Consider
the sums e1 x1 + e2 x2 + e3 x3 su h that:

 x ; x ; x 2 A;
 e ; e ; e 2 f 1; 0; 1g;
 at least one of e ; e ; e
 if xi = xj , then ei ej 6=
1

is nonzero;
1.

The set A is free if all su h sums are not divisible by n.


(a) Find a free set of ardinality bn=4 .
(b) Prove that any set of ardinality bn=4 + 1 is not free.

132

6. Find the least natural number a for whi h the equation


os2 (a x) 2 os (a x) + os

3x  x  
os
+ +2=0
2a
2a 3

has a real root.


7. Let ABCD be a trapezoid (AB jjCD) and hoose F on the segment
AB su h that DF = CF . Let E be the interse tion of AC and BD,
and let O1 ; O2 be the ir um enters of ADF; BCF . Prove that the
lines EF and O1 O2 are perpendi ular.
8. Find all natural numbers n for whi h a onvex n-gon an be divided into triangles by diagonals with disjoint interiors, su h that
ea h vertex of the n-gon is the endpoint of an even number of the
diagonals.
9. For any real number b, let f (b) denote the maximum of the fun tion


sin x +


2
+ b
3 + sin x

over all x 2 R. Find the minimum of f (b) over all b 2 R.


10. Let ABCD be a onvex quadrilateral su h that \DAB = \ABC =
\BCD. Let H and O denote the ortho enter and ir um enter of
the triangle ABC . Prove that H; O; D are ollinear.
11. For any natural number n  3, let m(n) denote the maximum number of points lying within or on the boundary of a regular n-gon of
side length 1 su h that the distan e between any two of the points
is greater than 1. Find all n su h that m(n) = n 1.
12. Find all natural numbers a; b; su h that the roots of the equations

x2 2ax + b = 0
x2 2bx + = 0
x2 2 x + a = 0
are natural numbers.

133

13. Given a y li onvex quadrilateral ABCD, let F be the interse tion


of AC and BD, and E the interse tion of AD and BC . Let M; N
be the midpoints of AB; CD. Prove that


MN 1 AB CD
=
:
EF
2 CD AB
14. Prove that the equation

x2 + y2 + z 2 + 3(x + y + z ) + 5 = 0
has no solutions in rational numbers.

15. Find all ontinuous fun tions f : R ! R su h that for all x 2 R,




1
f (x) = f x2 + :
4
16. Two unit squares K1; K2 with enters M; N are situated in the plane
so that MN = 4. Two sides of K1 are parallel to the line MN , and
one of the diagonals of K2 lies on MN . Find the lo us of the midpoint of XY as X; Y vary over the interior of K1 ; K2, respe tively.
17. Find the number of nonempty subsets of f1; 2; : : : ; ng whi h do not
ontain two onse utive numbers.

18. For any natural number n  2, onsider the polynomial


   
 


n
n
n 2
n
Pn (x) =
+
x+
x ++
xk ;
2
5
8
3k + 2
where k = b n 3 2 .

(a) Prove that Pn+3 (x) = 3Pn+2 (x) 3Pn+1 (x) + (x + 1)Pn (x).
(b) Find all integers a su h that 3b (n 1)=2 divides Pn (a3 ) for all
n  3.
19. Let M be the entroid of triangle ABC .
(a) Prove that if the line AB is tangent to the ir um ir le of the
triangle AMC , then
2
sin \CAM + sin \CBM  p :
3
134

(b) Prove the same inequality for an arbitrary triangle ABC .


20. Let m; n be natural numbers and m + i = ai b2i for i = 1; 2; : : : ; n,
where ai and bi are natural numbers and ai is squarefree. Find all
values of n for whi h there exists m su h that a1 + a2 +    + an = 12.
21. Let a; b; be positive numbers su h that ab = 1. Prove that
1
1
1
+
+
1+a+b 1+b+ 1+ +a

 2 +1 a + 2 +1 b + 2 +1 :

22. Let ABC be a triangle and M; N the feet of the angle bise tors of
B; C , respe tively. Let D be the interse tion of the ray MN with
the ir um ir le of ABC . Prove that
1
1
1
=
+
:
BD AD CD
23. Let X be a set of ardinality n + 1 (n  2). The ordered ntuples (a1 ; a2 ; : : : ; an ) and (b1 ; b2 ; : : : ; bn ) of distin t elements of X
are alled separated if there exist indi es i 6= j su h that ai = bj .
Find the maximal number of n-tuples su h that any two of them are
separated.

135

3.3 Canada

1. How many pairs (x; y) of positive integers with x  y satisfy g d(x; y) =


5! and l m(x; y) = 50!?

2. Given a nite number of losed intervals of length 1, whose union


is the losed interval [0; 50, prove that there exists a subset of the
intervals, any two of whose members are disjoint, whose union has
total length at least 25. (Two intervals with a ommon endpoint are
not disjoint.)
3. Prove that

1
1 3
1997 1
<   
< :
1999 2 4
1998 44
4. Let O be a point inside a parallelogram ABCD su h that \AOB +
\COD = . Prove that \OBC = \ODC .
5. Express the sum
 

n
X

( 1)k
n
3
2 + 26k + 24 k
k
+
9
k
k=0

in the form p(n)=q(n), where p; q are polynomials with integer oef ients.

136

3.4 China
1. Let x1 ; x2 ; : : : ; x1997 be real numbers satisfying the following onditions:
(a)
(b)

p
 xi  3 for i = 1; 2; : : : ; 1997;
p
x +x ++x
= 318 3.
p13

1997

12
1
Determine the maximum value of x12
1 + x2 +    + x1997 2.

2. Let A1 B1 C1 D1 be a onvex quadrilateral and P a point in its interior. Assume that the angles P A1 B1 and P A1 D1 are a ute, and similarly for the other three verti es. De ne Ak ; Bk ; Ck ; Dk as the re e tions of P a ross the lines Ak 1 Bk 1 ; Bk 1 Ck 1 ; Ck 1 Dk 1 ; Dk 1 Ak 1 .
(a) Of the quadrilaterals Ak Bk Ck Dk for k = 1; : : : ; 12, whi h ones
are ne essarily similar to the 1997th quadrilateral?
(b) Assume that the 1997th quadrilateral is y li . Whi h of the
rst 12 quadrilaterals must then be y li ?
3. Show that there exist in nitely many positive integers n su h that
the numbers 1; 2; : : : ; 3n an be labeled

a1 ; : : : ; a n ; b 1 ; : : : ; b n ; 1 ; : : : ; n
in some order so that the following onditions hold:
(a) a1 + b1 + 1 =    = an + bn + n is a multiple of 6;
(b) a1 +    + an = b1 +    + bn = 1 +    + n is also a multiple
of 6.
4. Let ABCD be a y li quadrilateral. The lines AB and CD meet at
P , and the lines AD and BC meet at Q. Let E and F be the points
where the tangents from Q meet the ir um ir le of ABCD. Prove
that points P; E; F are ollinear.
5. Let A = f1; 2; : : : ; 17g and for a fun tion f : A ! A, denote f [1 (x) =
f (x) and f [k+1 (x) = f (f [k (x)) for k 2 N . Find the largest natural
number M su h that there exists a bije tion f : A ! A satisfying
the following onditions:

137

(a) If m < M and 1  i  16, then

f [m(i + 1) f [m(i) 6 1 (mod 17):


(b) For 1  i  16,

f [M (i + 1) f [M (i)  1 (mod 17):


(Here f [k (18) is de ned to equal f [k (1).)
6. Let a1 ; a2 ; : : : ; be nonnegative integers satisfying

an+m  an + am (m; n 2 N ):
Prove that

an  ma1 +

for all n  m.

138

n

1 am

3.5 Colombia

1. We are given an m  n grid and three olors. We wish to olor ea h


segment of the grid with one of the three olors so that ea h unit
square has two sides of one olor and two sides of a se ond olor.
How many su h olorings are possible?
2. We play the following game with an equilaterial triangle of n(n +1)=2
pennies (with n pennies on ea h side). Initially, all of the pennies
are turned heads up. On ea h turn, we may turn over three pennies
whi h are mutually adja ent; the goal is to make all of the pennies
show tails. For whi h values of n an this be a hieved?

3. Let ABCD be a xed square, and onsider all squares P QRS su h


that P and R lie on di erent sides of ABCD and Q lies on a diagonal
of ABCD. Determine all possible positions of the point S .
4. Prove that the set of positive integers an be partitioned into an
in nite number of (disjoint) in nite sets A1 ; A2 ; : : : so that if x; y; z; w
belong to Ak for some k, then x y and z w belong to the same
set Ai (where i need not equal k) if and only if x=y = z=w.

139

3.6 Cze h and Slovak Republi s


1. Let ABC be a triangle with sides a; b; and orresponding angles
; ; . Prove that the equality = 3 implies the inequality (a2
b2)(a b) = b 2 , and determine whether the onverse also holds.
2. Ea h side and diagonal of a regular n-gon (n  3) is olored red
or blue. One may hoose a vertex and hange the olor of all of
the segments emanating from that vertex, from red to blue and vi e
versa. Prove that no matter how the edges were olored initially, it
is possible to make the number of blue segments at ea h vertex even.
Prove also that the resulting oloring is uniquely determined by the
initial oloring.
3. The tetrahedron ABCD is divided into ve onvex polyhedra so that
ea h fa e of ABCD is a fa e of one of the polyhedra (no fa es are divided), and the interse tion of any two of the ve polyhedra is either
a ommon vertex, a ommon edge, or a ommon fa e. What is the
smallest possible sum of the number of fa es of the ve polyhedra?
4. Show that there exists an in reasing sequen e fan g1
n=1 of natural
numbers su h that for any k  0, the sequen e fk + an g ontains
only nitely many primes.
5. For ea h natural number n  2, determine the largest possible value
of the expression

Vn = sin x1 os x2 + sin x2 os x3 +    + sin xn os x1 ;


where x1 ; x2 ; : : : ; xn are arbitrary real numbers.
6. A parallelogram ABCD is given su h that triangle ABD is a ute
and \BAD = =4. In the interior of the sides of the parallelogram,
points K on AB , L on BC , M on CD, N on DA an be hosen
in various ways so that KLMN is a y li quadrilateral whose ir umradius equals those of the triangles ANK and CLM . Find the
lo us of the interse tion of the diagonals of all su h quadrilaterals
KLMN .

140

3.7 Fran e
1. Ea h vertex of a regular 1997-gon is labeled with an integer, su h
that the sum of the integers is 1. Starting at some vertex, we write
down the labels of the verti es reading ounter lo kwise around the
polygon. Can we always hoose the starting vertex so that the sum
of the rst k integers written down is positive for k = 1; : : : ; 1997?
2. Find the maximum volume of a ylinder ontained in the interse tion
of a sphere with enter O radius R and a one with vertex O meeting
the sphere in a ir le of radius r, having the same axis as the one?
3. Find the maximum area of the orthogonal proje tion of a unit ube
onto a plane.
4. Given a triangle ABC , let a; b; denote the lengths of its sides and
m; n; p the lengths of its medians. For every positive real , let ( )
be the real number satisfying

a + b + = ( ) (m + n + p ):
(a) Compute (2).
(b) Determine the limit of ( ) as tends to 0.
( ) For whi h triangles ABC is ( ) independent of ?

141

3.8 Germany
1. Determine all primes p for whi h the system

p + 1 = 2x2
p2 + 1 = 2y2
has a solution in integers x; y.
2. A square Sa is ins ribed in an a ute triangle ABC by pla ing two
verti es on side BC and one on ea h of AB and AC . Squares Sb and
S are ins ribed similarly. For whi h triangles ABC will Sa ; Sb ; S
all be ongruent?
3. In a park, 10000 trees have been pla ed in a square latti e. Determine the maximum number of trees that an be ut down so that
from any stump, you annot see any other stump. (Assume the trees
have negligible radius ompared to the distan e between adja ent
trees.)
4. In the ir ular segment AMB , the entral angle \AMB is less than
90. FRom an arbitrary point on the ar AB one onstru ts the
perpendi ulars P C and P D onto MA and MB (C 2 MA, D 2
MB ). Prove that the length of the segemnt CD does not depend on
the position of P on the ar AB .
5. In a square ABCD one onstru ts the four quarter ir les having
their respe tive enters at A, B , C and D and ontaining the two
adja ent verti es. Inside ABCD lie the four interse tion points E ,
F , G and H , of these quarter ir les, whi h form a smaller square
S . Let C be the ir le tangent to all four quarter ir les. Compare
the areas of S and C .
6. Denote by u(k) the largest odd number that divides the natural
number k. Prove that
n

2
1 X
u(k)

2n k=1 k

 32 :

7. Find all real solutions of the system of equations

x3 = 2y 1
142

y3 = 2z 1
z 3 = 2x 1
8. De ne the fun tions

f (x) = x5 + 5x4 + 5x3 + 5x2 + 1


g(x) = x5 + 5x4 + 3x3 5x2 1:
Find all prime numbers p and for whi h there exists a natural number
0  x < p, su h that both f (x) and g(x) are divisible by p, and for
ea h su h p, nd all su h x.

143

3.9 Gree e
1. Let P be a point inside or on the sides of a square ABCD. Determine
the minimum and maximum possible values of

f (P ) = \ABP + \BCP + \CDP + \DAP:


2. Let f : (0; 1) ! R be a fun tion su h that
(a) f is stri tly in reasing;
(b) f (x) > 1=x for all x > 0;
( ) f (x)f (f (x) + 1=x) = 1 for all x > 0.
Find f (1).
3. Find all integer solutions of
13 1996
z
+ 2 =
:
x2
y
1997
4. Let P be a polynomial with integer oe ients having at least 13
distin t integer roots. Show that if n 2 Z is not a root of P , then
jP (n)j  7(6!)2 , and give an example where equality is a hieved.

144

3.10 Hungary
1. Ea h member of a ommittee ranks appli ants A; B; C in some order.
It is given that the majority of the ommittee ranks A higher than
B , and also that the majority of the ommitee ranks B higher than
C . Does it follow that the majority of the ommittee ranks A higher
than C ?
2. Let a; b; be the sides, ma ; mb ; m the lengths of the altitudes, and
da ; db ; d the distan es from the verti es to the ortho enter in an
a ute triangle. Prove that

ma da + mb db + m d =

a2 + b2 + 2
:
2

3. Let R be the ir umradius of triangle ABC , and let G and H be


its entroid and ortho enter, respe tively. Let F be the midpoint of
GH . Show that AF 2 + BF 2 + CF 2 = 3R2 .
4. A box ontains 4 white balls and 4 red balls, whi h we draw from
the box in some order without repla ement. Before ea h draw, we
guess the olor of the ball being drawn, always guessing the olor
more likely to o ur (if one is more likely than the other). What is
the expe ted number of orre t guesses?
5. Find all solutions in integers of the equation

x3 + (x + 1)3 + (x + 2)3 +    + (x + 7)3 = y3 :


6. We are given 1997 distin t positive integers, any 10 of whi h have the
same least ommon multiple. Find the maximum possible number
of pairwise oprime numbers among them.
7. Let AB and CD be noninterse ting hords of a ir le, and let K be
a point on CD. Constru t (with straightedge and ompass) a point
P on the ir le su h that K is the midpoint of the interse tion of the
part of the segment CD lying inside triangle ABP .
8. We are given 111 unit ve tors in the plane whose sum is zero. Show
that there exist 55 of the ve tors whose sum has length less than 1.

145

3.11 Iran
1. Suppose w1 ; : : : ; wk are distin t real numbers with nonzero sum.
Prove that there exist integers n1 ; : : : ; nk su h that n1 w1 +    +
nk wk > 0 and that for any permutation  of f1; : : : ; kg not equal to
the identity, we have n1 w(1) +    + nk w(k) < 0.
2. Suppose the point P varies along the ar BC of the ir um ir le
of triangle ABC , and let I1 ; I2 be the respe tive in enters of the
triangles P AB; P AC . Prove that
(a) the ir um ir le of P I1 I2 passes through a xed point;
(b) the ir le with diameter I1 I2 passes through a xed point;
( ) the midpoint of I1 I2 lies on a xed ir le.
3. Suppose f : R+ ! R+ is a de reasing ontinuous fun tion su h that
for all x; y 2 R+ ,

f (x + y) + f (f (x) + f (y)) = f (f (x + f (y))) + f (y + f (x)):


Prove that f (f (x)) = x.
4. Let A be a matrix of zeroes and ones whi h is symmetri (Aij = Aji
for all i; j ) su h that Aii = 1 for all i. Show that there exists a subset
of the rows whose sum is a ve tor all of whose omponents are odd.

146

3.12 Ireland
1. Find all pairs (x; y) of integers su h that 1 + 1996x + 1998y = xy.
2. Let ABC be an equilateral triangle. For M inside the triangle, let
D; E; F be the feet of the perpendi ulars from M to BC; CA; AB ,
respe tively. Find the lo us of points M su h that \F DE = =2.
3. Find all polynomials p(x) su h that for all x,
(x 16)p(2x) = 16(x 1)p(x):
4. Let a; b; be nonnegative real numbers su h that a + b +
Prove that a2 + b2 + 2  ab .

 ab .

5. Let S = f3; 5; 7; : : :g. For x 2 S , let (x) be the unique integer su h


that 2(x) < x < 2(x)+1. For a; b 2 S , de ne

a  b = 2(a) 1 (b 3) + a:
(a) Prove that if a; b 2 S , then a  b 2 S .
(b) Prove that if a; b; 2 S , then (a  b)  = a  (b  ).
6. Let ABCD be a onvex quadrilateral with an ins ribed ir le. If
\A = \B = 2=3, \D = =2 and BC = 1, nd the length of AD.
7. Let A be a subset of f0; 1; : : : ; 1997g ontaining more than 1000
elements. Prove that A ontains either a power of 2, or two distin t
integers whose sum is a power of 2.
8. Determine the number of natural numbers n satisfying the following
onditions:
(a) The de imal expansion of n ontains 1000 digits.
(b) All of the digits of n are odd.
( ) The absolute value of the di eren e between any two adja ent
digits of n is 2.

147

3.13 Italy
1. A re tangular strip of paper 3 entimeters wide is folded exa tly
on e. What is the least possible area of the region where the paper
overs itself?
2. Let f be a real-valued fun tion su h that for any real x,
(a) f (10 + x) = f (10 x);
(b) f (20 + x) = f (20 x).
Prove that f is odd (f ( x) = f (x)) and periodi (there exists
T > 0 su h that f (x + T ) = f (x)).
3. The positive quadrant of a oordinate plane is divided into unit
squares by latti e lines. Is it possible to olor some of the unit
squares so as to satisfy the following onditions:
(a) ea h square with one vertex at the origin and sides parallel to
the axes ontains more olored than un olored squares;
(b) ea h line parallel to the angle bise tor of the quadrant at the
origin passes through only nitely many olored squares?
4. Let ABCD be a tetrahedron. Let a be the length of AB and let
S be the area of the proje tion of the tetrahedron onto a plane
perpendi ular to AB . Determine the volume of the tetrahedron in
terms of a and S .
5. Let X be the set of natural numbers whose de imal representations
have no repeated digits. For n 2 X , let An be the set of numbers
whose digits are a permutation of the digits of n, and let dn be the
greatest ommon divisor of the numbers in An . Find the largest
possible value of dn .

148

3.14 Japan
1. Prove that among any ten points lo ated in a ir le of diameter 5,
there exist two at distan e less than 2 from ea h other.
2. Let a; b; be positive integers. Prove the inequality
(b + a)2
( + a b)2
(a + b )2
+
+
2
2
2
2
(b + ) + a
( + a) + b
(a + b)2 + 2

 35 ;

and determine when equality holds.


3. Let G be a graph with 9 verti es. Suppose given any ve points of
G, there exist at least 2 edges with both endpoints among the ve
points. What is the minimum possible number of edges in G?
4. Let A; B; C; D be four points in spa e not lying in a plane. Suppose
AX + BX + CX + DX is minimized at a point X = X0 distin t
from A; B; C; D. Prove that \AX0 B = \CX0 D.
5. Let n be a positive integer. Show that one an assign to ea h vertex
of a 2n -gon one of the letters A or B su h that the sequen es of n
letters obtained by starting at a vertex and reading ounter lo kwise
are all distin t.

149

3.15 Korea
1. Show that among any four points ontained
in a unit ir le, there
p
exist two whose distan e is at most 2.

! N be a fun tion satisfying


(a) For every n 2 N , f (n + f (n)) = f (n).
(b) For some n 2 N , f (n ) = 1.
Show that f (n) = 1 for all n 2 N .
p
P
p
Express nk b k in terms of n and a = b n .

2. Let f : N

3.

=1

4. Let C be a ir le tou hing the edges of an angle \XOY , and let


C1 be the ir le tou hing the same edges and passing through the
enter of C . Let A be the se ond endpoint of the diameter of C1
passing through the enter of C , and let B be the interse tion of
this diameter with C . Prove that the ir le entered at A passing
through B tou hes the edges of \XOY .
5. Find all integers x; y; z satisfying x2 + y2 + z 2

2xyz = 0.

6. Find the smallest integer k su h that there exist two sequen es fai g,
fbig (i = 1; : : : ; k) su h that
(a) For i = 1; : : : ; k, ai ; bi 2 f1; 1996; 19962; : : :g.
(b) For i = 1; : : : ; k, ai 6= bi .
( ) For i = 1; : : : ; k 1, ai  ai+1 and bi  bi+1 .
P
P
(d) ki=1 ai = ki=1 bi .

7. Let An be the set of all real numbers of the form 1+ p 12 + (p 2)2 2 +    +


p n , where j 2 f 1; 1g for ea h j . Find the number of elements
( 2)n
of An , and nd the sum of all produ ts of two distin t elements of
An .
8. In an a ute triangle ABC with AB 6= AC , let V be the interse tion
of the angle bise tor of A with BC , and let D be the foot of the
perpendi ular from A to BC . If E and F are the interse tions of the
ir um ir le of AV D with CA and AB , respe tively, show that the
lines AD; BE; CF on ur.
150

9. A word is a sequen e of 8 digits, ea h equal to 0 or 1. Let x and y be


two words di ering in exa tly three pla es. Show that the number
of words di ering from ea h of x and y in at least ve pla es is 188.
10. Find all pairs of fun tions f; g : R ! R su h that
(a) if x < y, then f (x) < f (y);
(b) for all x; y 2 R, f (xy) = g(y)f (x) + f (y).
11. Let a1 ; : : : ; an be positive numbers, and de ne
a +    + an
A = 1
n
G = (a1    an )1=n
n
H =
:
a1 1 +    + an 1


(a) If n is even, show that HA  1 + GA n .



(b) If n is odd, show that HA  nn 2 + 2(nn 1) GA n .
12. Let p1 ; : : : ; pr be distin t primes, and let n1 ; : : : ; nr be arbitrary
natural numbers. Prove that the number of pairs (x; y) of integers
satisfying x3 + y3 = pn1 1    pnr r is at most 2r 1.

151

3.16 Poland
1. The positive integers x1 ; : : : ; x7 satisfy the onditions

x6 = 144;

xn+3 = xn+2 (xn+1 + xn )

n = 1; 2; 3; 4:

Compute x7 .
2. Solve the following system of equations in real numbers x; y; z :
3(x2 + y2 + z 2 ) = 1
x y + y2 z 2 + z 2x2 = xyz (x + y + z )3 :
2 2

3. In a tetrahedron ABCD, the medians of the fa es ABD; ACD; BCD


from D make equal angles with the orresponding edges AB; AC; BC .
Prove that ea h of these fa es has area less than the sum of the areas
of the other two fa es.
4. The sequen e a1 ; a2 ; : : : is de ned by

a1 = 0;

an = abn=2 + ( 1)n(n+1)=2

n > 1:

For every integer k  0, nd the number of n su h that


2k  n < 2k+1 and an = 0:
5. Given a onvex pentagon ABCDE with DC = DE and \BCD =
\DEA = =2, let F be the point on segment AB su h that AF=BF =
AE=BC . Show that

\F CE = \F DE and \F EC = \BDC:
6. Consider n points (n  2) on a unit ir le. Show that at most n2 =3 of
the segments p
with endpoints among the n hosen points have length
greater than 2.

152

3.17 Romania
1. In the plane are given a line  and three ir les tangent to  and
externally tangent to ea h other. Prove that the triangle formed by
the enters of the ir les is obtuse, and nd all possible measures of
the obtuse angle.
2. Determine all sets A of nine positive integers su h that for any n 
500, there exists a subset B of A, the sum of whose elements is n.
3. Let n  4 be an integer and M a set of n points in the plane, no three
ollinear and not all lying on a ir le. Find all fun tions f : M ! R
su h that for any ir le C ontaining at least three points of M ,
X

P 2M \C

f (P ) = 0:

4. Let ABC be a triangle, D a point on side BC and ! the ir um ir le of ABC . Show that the ir les tangent to !; AD; BD and
to !; AD; DC , respe tively, are tangent to ea h other if and only if
\BAD = \CAD.
5. Let V A1    An be a pyramid with n  4. A plane  interse ts the
edges V A1 ; : : : ; V An at B1 ; : : : ; Bn , respe tively. Suppose that the
polygons A1    An and B1    Bn are similar. Prove that  is parallel
to the base of the pyramid.
6. Let A be the set of positive integers representable in the form a2 +2b2
for integers a; b with b 6= 0. Show that if p2 2 A for a prime p, then
p 2 A.

7. Let p  5 be a prime and hoose k 2 f0; : : : ; p 1g. Find the maximum length of an arithmeti progression, none of whose elements
ontain the digit k when written in base p.
8. Let p; q; r be distin t prime numbers and let A be the set

A = fpaqb r : 0  a; b;  5g:
Find the smallest integer n su h that any n-element subset of A
ontains two distin t elements x; y su h that x divides y.
153

9. Let ABCDEF be a onvex hexagon. Let P; Q; R be the interse tions


of the lines AB and EF , EF and CD, CD and AB , respe tively.
Let S; T; U be the interse tions of the lines BC and DE , DE and
F A, F A and BC , respe tively. Show that if AB=P R = CD=RQ =
EF=QP , then BC=US = DE=ST = F A=T U .
10. Let P be the set of points in the plane and D the set of lines
ithe plane. Determine whether there exists a bije tive fun tion
f : P ! D su h that for any three ollinear points A; B; C , the
lines f (A); f (B ); f (C ) are either parallel or on urrent.
11. Find all fun tions f : R ! [0; 1) su h that for all x; y 2 R,

f (x2 + y2 ) = f (x2

y2 ) + f (2xy):

12. Let n  2 be an integer and P (x) = xn + an 1 xn 1 +    + a1 x +1 be


a polynomial with positive integer oe ients. Suppose that ak =
an k for k = 1; 2; : : : ; n 1. Prove that there exist in nitely many
pairs x; y of positive integers su h that xjP (y) and yjP (x).
13. Let P (x); Q(x) be moni irredu ible polynomials over the rational
numbers. Suppose P and Q have respe tive roots and su h that
+ is rational. Prove that the polynomial P (x)2 Q(x)2 has a
rational root.
14. Let a > 1 be an integer. Show that the set

fa

+ a 1; a3 + a2

1; : : :g

ontains an in nite subset, any two members of whi h are relatively


prime.
15. Find the number of ways to olor the verti es of a regular dode agon
in two olors so that no set of verti es of a single olor form a regular
polygon.
16. Let be a ir le and AB a line not meeting . For any point P 2 ,
let P 0 be the se ond interse tion of the line AP with and let f (P )
be the se ond interse tion of the line BP 0 with . Given a point P0 ,
de ne the sequen e Pn+1 = f (Pn ) for n  0. Show that if a positive
integer k satis es P0 = Pk for a single hoi e of P0 , then P0 = Pk
for all hoi es of P0 .
154

3.18 Russia
1. Show that the numbers from 1 to 16 an be written in a line, but
not in a ir le, so that the sum of any two adja ent numbers is a
perfe t square.
2. On equal sides AB and BC of an equilateral triangle ABC are hosen
points D and K , and on side AC are hosen points E and M , so
that DA + AE = KC + CM = AB . Show that the angle between
the lines DM and KE equals =3.
3. A ompany has 50000 employees. For ea h employee, the sum of the
numbers of his immediate superiors and of his immediate inferiors is
7. On Monday, ea h worker issues an order and gives opies of it to
ea h of his immediate inferiors (if he has any). Ea h day thereafter,
ea h worker takes all of the orders he re eived on the previous day
and either gives opies of them to all of his immediate inferiors if he
has any, or otherwise arries them out himself. It turns out that on
Friday, no orders are given. Show that there are at least 97 employees
who have no immediate superiors.
4. The sides of the a ute triangle ABC are diagonals of the squares
K1; K2 ; K3 . Prove that the area of ABC is overed by the three
squares.
5. The numbers from 1 to 37 are written in a line so that ea h number
divides the sum of the previous numbers. If the rst number is 37
and the se ond number is 1, what is the third number?
6. Find all paris of prime numbers p; q su h that p3

q5 = (p + q)2 .
7. (a) In Mexi o City, to restri t tra ow, for ea h private ar are
designated two days of the week on whi h that ar annot be
driven on the streets of the ity. A family needs to have use of
at least 10 ars ea h day. What is the smallest number of ars
they must possess, if they may hoose the restri ted days for
ea h ar?
(b) The law is hanged to restri t ea h ar only one day per week,
but the poli e get to hoose the days. The family bribes the
poli e so that for ea h ar, they will restri t one of two days
hosen by the family. Now what is the smallest number of ars
the family needs to have a ess to 10 ars ea h day?
155

8. A regular 1997-gon is divided by noninterse ting diagonals into triangles. Prove that at least one of the triangles is a ute.
9. On a halkboard are written the numbers from 1 to 1000. Two
players take turns erasing a number from the board. The game ends
when two numbers remain: the rst player wins if the sum of these
numbers is divisible by 3, the se ond player wins otherwise. Whi h
player has a winning strategy?
10. 300 apples are given, no one of whi h weighs more than 3 times any
other. Show that the apples may be divided into groups of 4 su h
that no group weighs more than 11=2 times any other group.
11. In Robotland, a nite number of ( nite) sequen es of digits are forbidden. It is known that there exists an in nite de imal fra tion,
not ontaining any forbidden sequen es. Show that there exists an
in nite periodi de imal fra tion, not ontaining any forbidden sequen es.
12. (a) A olle tion of 1997 numbers has the property that if ea h number is subtra ted from the sum of the remaining numbers, the
same olle tion of numbers is obtained. Prove that the produ t
of the numbers is 0.
(b) A olle tion of 100 numbers has the same property. Prove that
the produ t of the numbers is positive.
13. Given triangle ABC , let A1 ; B1 ; C1 be the midpoints of the broken
lines CAB; ABC; BCA, respe tively. Let lA ; lB ; lC be the respe tive
lines through A1 ; B1 ; C1 parallel to the angle bise tors of A; B; C .
Show that lA; lB ; lC are on urrent.
14. The MK-97 al ulator an perform the following three operations on
numbers in its memory:
(a) Determine whether two hosen numbers are equal.
(b) Add two hosen numbers together.
( ) For hosen numbers a and b, nd the real roots of x2 + ax + b,
or announ e that no real roots exist.
The results of ea h operation are a umulated in memory. Initially
the memory ontains a single number x. How an one determine,
using the MK-97, whether x is equal to 1?
156

15. The ir les S1 and S2 interse t at M and N . Show that if verti es


A and C of a re tangle ABCD lie on S1 while verti es B and D lie
on S2 , then the interse tion of the diagonals of the re tangle lies on
the line MN .
16. For natural numbers m; n, show that 2n 1 divides (2m 1)2 if and
only if n divides m(2m 1).
17. Can three fa es of a ube of side length 4 be overed with 16 1  3
re tangles?
18. The verti es of triangle ABC lie inside a square K . Show that if
the triangle is rotated 180 about its entroids, at least one vertex
remains inside the square.
19. Let S (N ) denote the sum of the digits of the natural number N .
Show that there exist in nitely many natural numbers n su h that
S (3n)  S (3n+1 ).
20. The members of Congress form various overlapping fa tions su h
that given any two (not ne essarily distin t) fa tions A and B , the
omplement of A [ B is also a fa tion. Show that for any two fa tions
A and B , A [ B is also a fa tion.
21. Show that if 1 < a < b < , then
loga (loga b) + logb (logb ) + log (log a) > 0:
22. Do there exist pyramids, one with a triangular base and one with
a onvex n-sided base (n  4), su h that the solid angles of the
triangular pyramid are ongruent to four of the solid angles of the
n-sided pyramid?
23. For whi h does there exist a non onstant fun tion f : R ! R su h
that
f ( (x + y)) = f (x) + f (y)?
24. Let P (x) be a quadrati polynomial with nonnegative oe ients.
Show that for any real numbers x and y, we have the inequality

P (xy)2  P (x2 )P (y2 ):


157

25. Given a onvex polygon M invariant under a 90 rotation,


show that
p
there exist two ir les, the ratio of whose radii is 2, one ontaining
M and the other ontained in M .
26. (a) The Judgment of the Coun il of Sages pro eeds as follows: the
king arranges the sages in a line and pla es either a white hat
or a bla k hat on ea h sage's head. Ea h sage an see the olor
of the hats of the sages in front of him, but not of his own hat
or of the hats of the sages behind him. Then one by one (in an
order of their hoosing), ea h sage guesses a olor. Afterward,
the king exe utes those sages who did not orre tly guess the
olor of their own hat.
The day before, the Coun il meets and de ides to minimize the
number of exe utions. What is the smallest number of sages
guaranteed to survive in this ase?
(b) The king de ides to use three olors of hats: white, bla k and
red. Now what is the smallest number of sages guaranteed to
survive?
27. The lateral sides of a box with base ab and height (where a; b; are
natural numbers) are ompletely overed without overlap by re tangles whose edges are parallel to the edges of the box, ea h ontaining
an odd number of unit squares. Prove that if is odd, then the number of re tangles overing lateral edges of the box is even.
28. Do there exist real numbers b and su h that ea h of the equations
x2 + bx + = 0 and 2x2 + (b + 1)x + + 1 = 0 have two integer roots?
29. A lass onsists of 33 students. Ea h student is asked how many
other students in the lass have his rst name, and how many have
his last name. It turns out that ea h number from 0 to 10 o urs
among the answers. Show that there are two students in the lass
with the same rst and last name.
30. The in ir le of triangle ABC tou hes sides AB; BC; CA at M; N; K ,
respe tively. The line through A parallel to NK meets MN at D.
The line through A parallel to MN meets NK at E . Show that the
line DE bise ts sides AB and AC of triangle ABC .
31. The numbers from 1 to 100 are arranged in a 10  10 table so that
no two adja ent numbers sum to S . Find the smallest value of S for
whi h this is possible.
158

32. Find all integer solutions of the equation


(x2

y2 )2 = 1 + 16y:

33. An n  n square grid (n  3) is rolled into a ylinder. Some of the


ells are then olored bla k. Show that there exist two parallel lines
(horizontal, verti al or diagonal) of ells ontaining the same number
of bla k ells.
34. Two ir les interse t at A and B . A line through A meets the rst
ir le again at C and the se ond ir le again at D. Let M and N
be the midpoints of the ar s BC and BD not ontaining A, and let
K be the midpoint of the segment CD. Show that \MKN = =2.
(You may assume that C and D lie on opposite sides of A.)
35. A polygon an be divided into 100 re tangles, but not into 99. Prove
that it annot be divided into 100 triangles.
36. Do there exist two quadrati trinomials ax2 + bx + and (a + 1)x2 +
(b + 1)x + ( + 1) with integer oe ients, both of whi h have two
integer roots?
37. A ir le entered at O and ins ribed in triangle ABC meets sides
AC; AB; BC at K; M; N , respe tively. The median BB1 of the triangle meets MN at D. Show that O; D; K are ollinear.
38. Find all triples m; n; l of natural numbers su h that

m + n = g d(m; n)2 ; m + l = g d(m; l)2 ; n + l = g d(n; l)2 :


39. On an in nite (in both dire tions) strip of squares, indexed by the
natural numbers, are pla ed several stones (more than one may be
pla ed on a single square). We perform a sequen e of moves of one
of the following types:
(a) Remove one stone from ea h of the squares n 1 and n and
pla e one stone on square n + 1.
(b) Remove two stones from square n and pla e one stone on ea h
of the squares n + 1, n 2.
Prove that any sequen e of su h moves will lead to a position in whi h
no further moves an be made, and moreover that this position is
independent of the sequen e of moves.
159

40. An n  n  n ube is divided into unit ubes. We are given a losed


non-self-interse ting polygon (in spa e), ea h of whose sides joins
the enters of two unit ubes sharing a ommon fa e. The fa es of
unit ubes whi h interse t the polygon are said to be distinguished.
Prove that the edges of the unit ubes may be olored in two olors
so that ea h distinguished fa e has an odd number of edges of ea h
olor, while ea h nondistinguished fa e has an even number of edges
of ea h olor.
41. Of the quadrati trinomials x2 + px + q where p; q are integers and
1  p; q  1997, whi h are there more of: those having integer roots
or those not having real roots?
42. We are given a polygon, a line l and a point P on l in general position:
all lines ontaining a side of the polygon meet l at distin t points
di ering from P . We mark ea h vertex of the polygon whose sides
both meet the line l at points di ering from P . Show that P lies
inside the polygon if and only if for ea h hoi e of l there are an odd
number of marked verti es.
43. A sphere ins ribed in a tetrahedron tou hes one fa e at the interse tion of its angle bise tors, a se ond fa e at the interse tion of its
altitudes, and a third fa e at the interse tion of its medians. Show
that the tetrahedron is regular.
44. In an m  n re tangular grid, where m and n are odd integers, 1  2
dominoes are initially pla ed so as to exa tly over all but one of
the 1  1 squares at one orner of the grid. It is permitted to slide
a domino towards the empty square, thus exposing another square.
Show that by a sequen e of su h moves, we an move the empty
square to any orner of the re tangle.

160

3.19 South Afri a


1. From an initial triangle A0 B0 C0 a sequen e A1 B1 C1 ; A2 B2 C2 ; : : :
is formed su h that at ea h stage, Ak+1 ; Bk+1 ; Ck+1 are the points
where the in ir le of Ak Bk Ck tou hes the sides Bk Ck ; Ck Ak ; Ak Bk ,
respe tively.
(a) Express \Ak+1 Bk+1 Ck+1 in terms of \Ak Bk Ck .
(b) Dedu e that as k ! 1, \Ak Bk Ck ! 60.
2. Find all natural numbers with the property that, when the rst digit
is moved to the end, the resulting number is 3 12 times the original
one.
3. Find all fun tions f : Z ! Z whi h satisfy f (m + f (n)) = f (m) + n
for all m; n 2 Z.
4. A ir le and a point P above the ir le lie in a verti al plane. A
parti le moves along a straight line from P to a point Q on the ir le
under the in uen e of gravity. That is, the distan e traveled from P
in time t equals 21 gt2 sin , where g is a onstant and is the angle
between P Q and the horizontal. Des ribe (geometri ally) the point
Q for whi h the time taken to move from P to Q is minimized.
5. Six points are joined pairwise by red or blue segments. Must there
exist a losed path onsisting of four of the segments, all of the same
olor?

161

3.20 Spain
1. Cal ulate the sum of the squares of the rst 100 terms of an arithmeti progression, given that the sum of the rst 100 terms is 1 and
that the sum of the se ond, fourth, . . . , and the hundredth terms is
1.
2. Let A be a set of 16 latti e points forming a square with 4 points on
a side. Find the maximum number of points of A no three of whi h
form an isos eles right triangle.
3. For ea h parabola y = x2 + px + q meeting the oordinate axes in
three distin t points, a ir le through these points is drawn. Show
that all of the ir les pass through a single point.
4. Let p be a prime number. Find all k
positive integer.

2 Z su h that k

pk is a

5. Show that in any onvex quadrilateral of area 1,pthe sum of the


lengths of the sides and diagonals is at least 2(2 + 2).
6. The exa t quantity of gas needed for a ar to omplete a single loop
around a tra k is distributed among n ontainers pla ed along the
tra k. Prove that there exists a position starting at whi h the ar,
beginning with an empty tank of gas, an omplete a single loop
around the tra k without running out of gas. (Assume the ar an
hold unlimited quantities of gas.)

162

3.21 Taiwan
1. Let a be a rational number, b; ; d be real numbers, and f :
[ 1; 1 a fun tion satisfying

f (x + a + b) f (x + b) = bx + 2a + bx 2bx + a

bb + d

for ea h x 2 R. Show that f is periodi , that is, there exists p > 0


su h that f (x + p) = f (x) for all x 2 R.
2. Let AB be a given line segment. Find all possible points C in the
plane su h that in the triangle ABC , the altitude from A and the
median from B have the same length.
3. Let n  3 be an integer, and suppose that the sequen e a1 ; a2 ; : : : ; an
satis es ai 1 +ai+1 = ki ai for some sequen e k1 ; k2 ; : : : ; kn of positive
integers. (Here a0 = an and an+1 = a1 .) Show that
2n  k1 + k2 +    + kn  3n:
n

4. Let k = 22 + 1 for some positive integer n. Show that k is a prime


if and only if k is a fa tor of 3(k 1)=2 + 1.
5. Let ABCD be a tetrahedron. Show that
(a) If AB = CD, AD = BC , AC = BD, then the triangles
ABC; ACD; ABD; BCD are a ute;
(b) If ABC; ACD; ABD; BCD have the same area, then AB =
CD; AD = BC; AC = BD.
6. Let X be the set of integers of the form

a2k 102k + a2k 2 102k

+    + a2 102 + a0 ;

where k is a nonnegative integer and a2i 2 f1; 2; : : : ; 9g for i =


0; 2; : : : ; 2k. Show that every integer of the form 2p 3q , for p and q
nonnegative integers, divides some element of X .
7. Determine all positive integers k for whi h there exists a fun tion
f : N ! Z su h that
(a) f (1997) = 1998;
163

(b) for all a; b 2 N , f (ab) = f (a) + f (b) + kf (g d(a; b)).


8. Let ABC be an a ute triangle with ir um enter O and ir umradius
R. Let AO meet the ir um ir le of OBC again at D, BO meet the
ir um ir le of OCA again at E , and CO meet the ir um ir le of
OAB again at F . Show that OD  OE  OF  8R3.

9. For n  k  3, let X = f1; 2; : : : ; ng and let Fk be a family of kelement subsets of X su h that any two subsets in Fk have at most
k 2 ommon elements. Show that there exists a subset Mk of X
with at least blog2 n + 1 elements ontaining no subset in Fk .

164

3.22 Turkey
1. In a triangle ABC with a right angle at A, let H denote the foot of
the altitude from A. Show that the sum of the inradii of the triangles
ABC; ABH; ACH equals AH .
1
2. The sequen es fan g1
n=1 ; fbn gn=1 are de ned as follows: a1 = ,
b1 = , an+1 = an bn , bn+1 = an + bn for all n  1. How
many pairs ( ; ) of real numbers are there su h that a1997 = b1 and
b1997 = a1 ?
3. In a so er league, when a player moves from a team X with x players
to a team Y with y players, the federation re eives y x million
dollars from Y if y  x, but pays x y million dollars to X if x > y.
A player may move as often as he wishes during a season. The league
onsists of 18 teams, ea h of whi h begins a ertain season with 20
players. At the end of the season, 12 teams end up with 20 players,
while the other 6 end up with 16; 16; 21; 22; 22; 23 players. What is
the maximum amount the federation ould have earned during the
season?
4. The edge AE of a onvex pentagon ABCDE with verti es on a unit
ir le passes through the enter of the ir le. If AB = a, BC = b,
CD = , DE = d and ab = d = 1=4, ompute AC + CE in terms
of a; b; ; d.
5. Prove that for ea h prime p  7, there exists a positive integer n and
integers x1 ; : : : ; xn ; y1 ; : : : ; yn not divisible by p su h that
x21 + y12  x22 (mod p)
x22 + y22  x23 (mod p)
..
.

x2n + yn2

 x

2
1

(mod p):

6. Given an integer n  2, nd the minimal value of


x52
x5n
x51
+
++
x2 + x3 +    + xn x3 +    + xn + x1
x1 +    + xn 1
for positive real numbers x1 ; : : : ; xn subje t to the ondition x21 +
   + x2n = 1.
165

3.23 Ukraine
1. A re tangular grid is olored in he kerboard fashion, and ea h ell
ontains an integer. It is given that the sum of the numbers in ea h
row and the sum of the numbers in ea h olumn is even. Prove that
the sum of all numbers in bla k ells is even.
2. Find all solutions in real numbers to the following system of equations:

x1 + x2 +    + x1997 = 1997
x41 + x42 +    + x1997
= x31 + x32 +    + x31997 :
4
3. Let d(n) denote the greatest odd divisor of the natural number n. We
de ne the fun tion f : N ! N su h that f (2n 1) = 2n and f (2n) =
n + d2(nn) for all n 2 N . Find all k su h that f (f (   f (1)   )) = 1997,
where f is iterated k times.
4. Two regular pentagons ABCDE and AEKP L are situated in spa e
so that \DAK = 60. Prove that the planes ACK and BAL are
perpendi ular.
5. The equation ax3 + bx2 + x + d = 0 is known to have three distin t
real roots. How many real roots are there of the equation
4(ax3 + bx2 + x + d)(3ax + b) = (3ax2 + 2bx + )2 ?
6. Let Q + denote the set of positive rational numbers. Find all fun tions f : Q + ! Q + su h that for all x 2 Q + :
(a) f (x + 1) = f (x) + 1;
(b) f (x2 ) = f (x)2 .
7. Find the smallest integer n su h that among any n integers, there
exist 18 integers whose sum is divisible by 18.
8. Points K; L; M; N lie on the edges AB; BC; CD; DA of a (not ne essarily right) parallelepiped ABCDA1 B1 C1 D1 . Prove that the
enters of the ir ums ribed spheres of the tetrahedra A1 AKN ,
B1 BKL, C1 CLM , D1 DMN are the verti es of a parallelogram.
166

3.24 United Kingdom


1. (a) Let M and N be two 9-digit positive integers with the property
that if any one digit of M is repla ed by the digit of N in the
orresponding pla e, the resulting integer is a multiple of 7.
Prove that any number obtained by repla ing a digit of N with
the orresponding digit of M is also a multiple of 7.
(b) Find an integer d > 9 su h that the above result remains true
when M and N are two d-digit positive integers.
2. In a ute triangle ABC , CF is an altitude, with F on AB , and BM
is a median, with M on CA. Given that BM = CF and \MBC =
\F CA, prove that the triangle ABC is equilateral.
3. Find the number of polynomials of degree 5 with distin t oe ients
from the set f1; 2; : : : ; 9g that are divisible by x2 x + 1.

4. The set S = f1=r : r = 1; 2; 3; : : :g of re ipro als of the positive


integers ontains arithmeti progressions of various lengths. For instan e, 1=20; 1=8; 1=5 is su h a progression, of length 3 and ommon
di eren e 3=40. Moreover, this is a maximal progression in S of
length 3 sin e it annot be extended to the left or right within S
( 1=40 and 11=40 not being members of S ).
(a) Find a maximal progression in S of length 1996.
(b) Is there a maximal progression in S of length 1997?

167

3.25 United States of Ameri a


1. Let p1 ; p2 ; p3 ; : : : be the prime numbers listed in in reasing order,
and let x0 be a real number between 0 and 1. For positive integer
k, de ne

xk = 0 if xk

= 0;

pk
xk 1

if xk

6= 0;

where fxg = x bx denotes the fra tional part of x. Find, with


proof, all x0 satisfying 0 < x0 < 1 for whi h the sequen e x0 ; x1 ; x2 ; : : :
eventually be omes 0.
2. Let ABC be a triangle, and draw isos eles triangles BCD; CAE; ABF
externally to ABC , with BC; CA; AB as their respe tive bases. Prove
that the lines through A; B; C perpendi ular to the lines EF; F D; DE ,
respe tively, are on urrent.
3. Prove that for any integer n, there exists a unique polynomial Q
with oe ients in f0; 1; : : : ; 9g su h that Q( 2) = Q( 5) = n.
4. To lip a onvex n-gon means to hoose a pair of onse utive sides
AB; BC and to repla e them by the three segments AM; MN , and
NC , where M is the midpoint of AB and N is the midpoint of BC .
In other words, one uts o the triangle MBN to obtain a onvex
(n + 1)-gon. A regular hexagon P6 of area 1 is lipped to obtain a
heptagon P7 . Then P7 is lipped (in one of the seven possible ways)
to obtain an o tagon P8 , and so on. Prove that no matter how the
lippings are done, the area of Pn is greater than 1=3, for all n  6.
5. Prove that, for all positive real numbers a; b; ,
(a3 + b3 + ab )

+ (b3 + 3 + ab )

+ ( 3 + a3 + ab )

 (ab ) :
1

6. Suppose the sequen e of nonnegative integers a1 ; a2 ; : : : ; a1997 satis es


ai + aj  ai+j  ai + aj + 1
for all i; j  1 with i + j  1997. Show that there exists a real
number x su h that an = bnx for all 1  n  1997.

168

3.26 Vietnam
1. Determine the smallest integer k for whi h there exists a graph on
25 verti es su h that every vertex is adja ent to exa tly k others,
and any two nonadja ent verti es are both adja ent to some third
vertex.
2. Find the largest real number for whi h there exists an in nite
sequen e a1 ; a2 ; : : : of positive integers satisfying the following properties.
(a) For ea h n 2 N , an > 1997n.
(b) For every n  2, a n does not ex eed the greatest ommon
divisor of the set fai + aj : i + j = ng.
3. Let f : N

! Z be the fun tion de ned by

f (0) = 2; f (1) = 503; f (n + 2) = 503f (n + 1) 1996f (n):


For k 2 N , let s1 ; : : : ; sk be integers not less than k, and let pi be a
prime divisor of f (2si ) for i = 1; : : : ; k. Prove that for t = 1; : : : ; k,
k
X
i=1

pi j 2t

if and only if

k j 2t :

4. Find all pairs (a; b) of positive reals su h that for every n 2 N and
every real number x satisfying
4n2 x = log2 (2n2 x + 1);
we have ax + bx  2 + 3x.

5. Let n; k; p be positive integers su h that k  2 and k(p + 1)  n.


Determine the number of ways to olor n labeled points on a ir le
in blue or red, so that exa tly k points are olored blue, and any ar
whose endpoints are blue but ontains no blue points in its interior
ontains exa tly p red points.

169

1997 Regional Contests:


Problems

4.1 Asian Pa i Mathemati s Olympiad


1. Let

S =1+

1
1
1
+
++
;
1
1
1
1
1
1
1+ 3 1+ 3 + 6
1 + 3 + 6 +    + 1993006

where the denominators ontain partial sums of the sequen e of re ipro als of triangular numbers. Prove that S > 1001.
2. Find an integer n with 100  n  1997 su h that n divides 2n + 2.
3. Let ABC be a triangle and let
m
m
m
la = a ; lb = b ; l = ;
Ma
Mb
M
where ma ; mb ; m are the lengths of the internal angle bise tors and
Ma; Mb ; M are the lengths of the extensions of the internal angle
bise tors to the ir um ir le. Prove that
la
l
l
+ b +  3;
sin2 A sin2 B sin2 C
with equality if and only if ABC is equilateral.
4. The triangle A1 A2 A3 has a right angle at A3 . For n  3, let An+1
be the foot of the perpendi ular from An to An 1 An 2 .
(a) Show that there is a unique point P in the plane interior to the
triangles An 2 An 1 An for all n  3.
(b) For xed A1 and A3 , determine the lo us of P as A2 varies.
5. Persons A1 ; : : : ; An (n  3) are seated in a ir le in that order, and
ea h person Ai holds a number ai of obje ts, su h that (a1 +    +
an )=n is an integer. It is desired to redistribute the obje ts so that
ea h person holds the same number; obje ts may only be passed
from one person to either of her two neighbors. How should the
redistribution take pla e so as to minimize the number of passes?
170

4.2 Austrian-Polish Mathemati al Competition


1. Let P be the interse tion of lines l1 and l2 . Let S1 and S2 be two
ir les externally tangent at P and both tangent to l1 , and let T1
and T2 be two ir les externally tangent at P and both tangent to l2 .
Let A be the se ond interse tion of S1 and T1 , B that of S1 and T2,
C that of S2 and T1 , and D that of S2 and T2 . Show that the points
A; B; C; D are on y li if and only if l1 and l2 are perpendi ular.

2. Let m; n; p; q be positive integers, and onsider an mn he kerboard


with a he ker on ea h of its mn squares. A pie e an be moved from
(x; y) to (x0 ; y0 ) if and only if jx x0 j = p and jy y0 j = q. How many
ways an all of the pie es be moved simultaneously so that one pie e
ends up on ea h square?
3. On a bla kbroad are written the numbers 48=k with k = 1; 2; : : : ; 97.
At ea h step, two numbers a; b are erased and 2ab a b + 1 is
written in their pla e. After 96 steps, a single number remains on
the bla kboard. Determine all possible su h numbers.
4. In a onvex quadrilateral ABCD, the sides AB and CD are parallel,
the diagonals AC and BD interse t at E , and the triangles EBC
and EAD have respe tive ortho enters F and G. Prove that the
midpoint of GF lies on the line through E perpendi ular to AB .
5. Let p1 ; p2 ; p3 ; p4 be distin t primes. Prove there does not exist a
ubi polynomial Q(x) with integer oe ients su h that

jQ(p )j = jQ(p )j = jQ(p )j = jQ(p )j = 3:


1

6. Prove there does not exist f : Z ! Z su h that f (x + f (y)) = f (x) y


for all integers x; y.

7. (a) Prove that for all p; q 2 R, p2 + q2 + 1 > p(q + 1).


(b) Determine the largest real number x su h that p2 + q2 + 1 >
bp(q + 1) for all p; q 2 R.
( ) Determine the largest real number x su h that p2 + q2 + 1 >
bp(q + 1) for all p; q 2 Z.
8. Let n be a natural number and M a set with n elements. Find
the biggest integer k su h that there exists a k-element family of
three-element subsets of M , no two of whi h are disjoint.
171

9. Let P be a parallelepiped with volume V and surfa e area S , and


let L be the sum of the lengths of the edges of P . For t  0, let Pt
be the set of points whi h lie at distan e at most t from some point
of P . Prove that the volume of Pt is


4
V + St + Lt2 + t3 :
4
3

172

4.3 Cze h-Slovak Mat h


1. An equilateral triangle ABC is given. Points K and L are hosen
on its sides AB and AC , respe tively, su h that jBK j = jALj. Let
P be the interse tion of the segmentns BL and CK . Determine the
ratio jAK j : jKB j if it is known that the segments AP and CK are
perpendi ular.
2. In a ommunity of more than six people, ea h member ex hanges
letters with pre isely three other members of the ommunity. Prove
that the ommunity an be divided into two nonempty groups so
that ea h member ex hanges letters with at least two members of
the group he belongs to.
3. Find all fun tions f : R ! R su h that the equality

f (f (x) + y) = f (x2

y) + 4f (x)y

holds for all pairs of real numbers x; y.


4. Is it possible to pla e 100 solid balls in spa e so that no two of them
have a ommon interior point, and ea h of them tou hes at least
one-third of the others?
5. Several integers are given (some of them may be equal) whose sum
is equal to 1492. De ide whether the sum of their seventh powers
an equal
(a) 1996;
(b) 1998.
6. In a ertain language there are only two letters, A and B . The words
of this language obey the following rules:
(a) The only word of length 1 is A.
(b) A sequen e of letters X1 X2    Xn Xn+1 , where Xi 2 fA; B g for
ea h i, is a word if and only if it ontains at least one A but is
not of the form X1 X2    Xn A where X1 X2    Xn is a word.


Show that there are pre isely 3995


1 words whi h do not begin
1997
with AA and whi h are omposed of 1998 A's and 1998 B 's.
173

4.4 Hungary-Israel Mathemati s Competition


1. Is there an integer N su h that

( 1997

1996)1998 = N

1?

2. Find all real numbers with the following property: for any positive
integer n, there exists an integer m su h that


m 1
< :
n 3n

3. The a ute triangle ABC has ir um enter O. Let A1 ; B1 ; C1 be the


points where the diameters of the ir um ir le through A; B; C meet
the sides BC; CA; AB , respe tively. Suppose the ir umradius of
ABC is 2p for some prime number p, and the lengths OA1 ; OB1 ; OC1
are integers. What are the lengths of the sides of the triangle?
4. How many distin t sequen es of length 1997 an be formed using
ea h of the letters A; B; C an odd number of times (and no others)?
5. The three squares ACC1 A00 ; ABB10 A0 ; BCDE are onstru ted externally on the sides of a triangle ABC . Let P be the enter of BCDE .
Prove that the lines A0 C; A00 B; P A are on urrent.

174

4.5 Iberoameri an Mathemati al Olympiad


1. Let r  1 be a real number su h that for all m; n su h that m divides
n, bmr divides bnr . Prove that r is an integer.
2. Let ABC be a triangle with in enter I . A ir le entered at I meets
the segment BC at D and P (with D loser to B ), CA at E and
Q (with E loser to C ), and AB at F and R (with F loser to A).
Let S; T; U be the interse tions of the diagonals of the quadrilaterals
EQF R; F RDP; DP EQ, respe tively. Show that the ir um ir les
of the triangles F RT; DP U; EQS pass through a ommon point.
3. Let n  2 be an integer and Dn the set of points (x; y) in the plane
su h that x; y are integers with jxj; jyj  n.
(a) Prove that if ea h of the points in Dn is olored in one of three
olors, there exist two points of Dn in the same olor su h that
the line through them passes through no other point of Dn .
(b) Show that the points of Dn an be olored in four olors so that
if a line ontains exa tly two points of Dn , those two points have
di erent olors.
4. Let n be a positive integer. Let On be the number of 2n-tuples
(x1 ; : : : ; xn ; y1 ; : : : ; yn ) with values in 0 or 1 for whi h the sum x1 y1 +
   + xn yn is odd, and let En be the number of 2n-tuples for whi h
the sum is even. Prove that
On 2n 1
=
:
En 2n + 1
5. Let AE and BF be altitudes, and H the ortho enter, of a ute triangle ABC . The re e tion of AE a ross the interior angle bise tor
of A meets the re e tion of BF a ross the interior angle bise tor of
B meet in a point O. The lines AE and AO meet the ir um ir le
of ABC again at M and N , respe tively. Let P; Q; R be the interse tion of BC with HN , BC with OM , HR with OP , respe tively.
Show that AHSO is a parallelogram.
6. Let P = fP1 ; P2 ; : : : ; P1997 g be a set of 1997 points in the interior
of a ir le of radius 1, with P1 the enter of the ir le. For k =

175

1; : : : ; 1997, let xk be the distan e from Pk to the point of P losest


to Pk . Prove that

x21 + x22 +    + x21997  9:

176

4.6 Nordi Mathemati al Contest


1. For any set A of positive integers, let nA denote the number of triples
(x; y; z ) of elements of A su h that x < y and x + y = z . Find the
maximum value of nA given that A ontains seven distin t elements.
2. Let ABCD be a onvex quadrilateral. Assume that there exists
an internal point P of ABCD su h that the areas of the triangles
ABP; BCP; CDP; DAP are all equal. Prove that at least one of the
diagonals of the quadrilateral bise ts the other.
3. Assume that A; B; C; D are four distin t points in the plane. Three
of the segments AB; AC; AD; BC; BD; CD have length a. The other
three have length b > a. Find all possible values of the ratio b=a.
4. Let f be a fun tion de ned on f0; 1; 2; : : :g su h that

f (2x) = 2f (x); f (4x + 1) = 4f (x) + 3; f (4x 1) = 2f (2x 1) 1:


Prove that f is inje tive (if f (x) = f (y), then x = y).

177

4.7 Rio Plata Mathemati al Olympiad


1. Around a ir le are written 1996 zeroes and and one 1. The only
permitted operation is to hoose a number and hange its two neighbors, from 0 to 1 and vi e versa. Is it possible to hange all of the
numbers to 1? And what if we started with 1997 zeroes?
2. Show that one annot draw two triangles of area 1 inside a ir le of
radius 1 so that the triangles have no ommon point.
3. A bene t on ert is attended by 1997 people from Peru, Bolivia,
Paraguay and Venezuela. Ea h person paid for his ti ket an integer
number of dollars between 1 and 499, in lusive.
(a) Prove that at least two people of the same nationality paid the
same pri e.
(b) It is known that ea h possible pri e was paid at least on e, that
the maximum number of times a pri e was repeated was 10, and
that subje t to these onditions, the smallest amount of money
was olle ted. How many ti kets were sold at ea h pri e?
4. A 4  4 square is divided into 1  1 squares. A se ret number is
written into ea h small square. All that is known is that the sum
of the numbers in ea h row, ea h olumn, and ea h of the diagonals
equals 1. Is it possible to determine from this information the sum
of the numbers in the four orners, and the sum of the numbers in
the four entral squares? And if so, what are these sums?
5. What is the smallest multiple of 99 whose digits sum to 99 and whi h
begins and ends with 97?
6. A tourist takes a trip through a ity in stages. Ea h stage onsist of
three segments of length 100 meters separated by right turns of 60.
Between the last segment of one stage and the rst segment of the
next stage, the tourist makes a left turn of 60 . At what distan e
will the tourist be from his initial position after 1997 stages?

178

4.8 St. Petersburg City Mathemati al Olympiad (Russia)


1. The in ir le of a triangle is proje ted onto ea h of the sides. Prove
that the six endpoints of the proje tions are on y li .
2. Let a and b be integers. Prove that


a + b ab


a b

 1:

3. Prove that every positive integer has at least as many (positive)


divisors whose last de imal digit is 1 or 9 as divisors whose last digit
is 3 or 7.
4. Prove that opposite verti es of a 142  857 re tangle with verti es
at latti e points annot be joined by a ve-edge broken line with
verti es at latti e points su h that the ratio of the lengths of the
edges is 2 : 3 : 4 : 5 : 6.
5. Do there exist 100 positive integers su h that the sum of the fourth
powers of every four of the integers is divisible by the produ t of the
four numbers?
6. Let B 0 be the antipode of B on the ir um ir le of triangle ABC , let
I be the in enter of ABC , and let M be the point where the in ir le
tou hes AC . The points K and L are hosen on the sides AB and
BC , respe tively, so that KB = MC , LB = AM . Prove that the
lines B 0 I and KL are perpendi ular.
7. Can a 19971997 square be disse ted into squares whose side lengths
are integers greater than 30?
8. At ea h vertex of a regular 1997-gon is written a positive integer.
One may add 2 to any of the numbers and subtra t 1 from the numbers k away from it in either dire tion, for some k 2 f1; 2; : : : ; 1998g;
the number k is then written on a bla kboard. After some number
of operations, the original numbers reappear at all of the verti es.
Prove that at this time, the sum of the squares of the numbers written on the bla kboard is divisible by 1997.
179

9. The positive integers x; y; z satisfy the equation 2xx + yy = z z . Prove


that x = y = z .
10. The number N is the produ t of k diferent primes (k  3). Two
players play the following game. In turn, they write omposite divisors of N on a bla kboard. One may not write N . Also, there may
never appear two oprime numbers or two numbers, one of whi h
divides the other. The rst player unable to move loses. Does the
rst player or the se ond player have a winning strategy?
11. Let K; L; M; N be the midpoints of sides AB; BC; CD; DA, respe tively, of a y li quadrilateral ABCD. Prove that the ortho entres
of triangles AKN; BKL; CLM; DMN are the verti es of a parallelogram.
12. A 100  100 square grid is folded several times along grid lines. Two
straight uts are also made along grid lines. What is the maximum
number of pie es the square an be ut into?
13. The sides of a onvex polyhedron are all triangles. At least 5 edges
meet at ea h vertex, and no two verti es of degree 5 are onne ted
by an edge. Prove that this polyhedron has a side whose verti es
have degrees 5, 6, 6, respe tively.
14. Given 2n + 1 lines in the plane, prove that there are at most n(n +
1)(2n + 1)=6 a ute triangles with sides on the lines.
15. Prove that the set of all 12-digit numbers annot be divided into
groups of 4 numbers so that the numbers in ea h group have the
same digits in 11 pla es and four onse utive digits in the remaining
pla e.
16. A ir le is divided into equal ar s by 360 points. The points are
joined by 180 noninterse ting hords. Consider also the 180 hords
obtained from these by a rotation of 38 about the enter of the
ir le. Prove that the union of these 360 hords annot be a losed
(self-interse ting) polygon.
17. Can a 75  75 table be partitioned into dominoes (1  2 re tangles)
and rosses ( ve-square gures onsisting of a square and its four
neighbors)?
180

18. Prove that for x; y; z  2, (y3 + x)(z 3 + y)(x3 + z )  125xyz .


19. The ir les S1 ; S2 interse t at A and B . Let Q be a point on S1 .
The lines QA and QB meet S2 at C and D, respe tively, while the
tangents to S1 at A and B meet at P . Assume that Q lies outside
S2 , and that C and D lie outside S1 . Prove that the line QP goes
through the midpoint of CD.
20. Given a onvex 50-gon with verti es at latti e points, what is the
maximum number of diagonals whi h an lie on grid lines?
21. The number 99    99 (with 1997 nines) is written on a bla kboard.
Ea h minute, one number written on the bla kboard is fa tored into
two fa tors and erased, ea h fa tor is (independently) in reased or
diminished by 2, and the resulting two numbers are written. Is it
possible that at some point all of the numbers on the bla kboard
equal 9?
22. A devi e onsists of 4n elements, any two of whi h are joined by
either a red or a blue wire. The numbers of red and blue wires are
the same. The devi e is disabled by removing two wires of the same
olor onne ting four di erent elements. Prove that the number of
ways to disable the devi e by removing two blue wires is the same
as the number of ways by removing two red wires.
23. An Azte diamond of rank n is a gure onsisting of those squares of
a gridded oordinate plane lying entirely inside the diamond f(x; y) :
jxj + jyj  n +1g. For any overing of an Azte diamond by dominoes
(1  2 re tangles), we may rotate by 90 any 2  2 square overed by
exa tly two dominoes. Prove that at most n(n + 1)(2n + 1)=6 rotations are needed to transform an arbitrary overing into the overing
onsisting only of horizontal dominoes.

181

You might also like